Sunteți pe pagina 1din 81

American Board of Family Medicine

2013 IN-TRAINING EXAMINATION

CRITIQUE BOOK

This book contains the answers to each question in the In-Training Examination, as well as a critique that
provides a rationale for the correct answer. Bibliographic references are included at the end of each
critique to facilitate any further study you may wish to do in a particular area.

Copyright© 2013 The American Board of Family Medicine, Inc. All rights reserved.
Item 1

ANSWER: E

This patient with acute kidney injury (AKI) has clinical symptoms and signs consistent with
rhabdomyolysis, a known cause of AKI. Furthermore, she is taking a medication known to cause
rhabdomyolysis. The urinalysis with a positive dipstick for blood and no RBCs on the microscopic
examination is indicative of either hemolysis or rhabdomyolysis. Darkened, pigmented serum would be
expected with hemolysis, while rhabdomyolysis is associated with clear serum. Urine abnormalities found
in glomerulonephritis include proteinuria and RBC casts, while patients with allergic interstitial nephritis
may have eosinophils and possibly WBC casts. Pyelonephritis is associated with WBCs in the urine, and
if the dipstick is positive for blood there will be RBCs on the microscopic examination.

Ref: Rahman M, Shad F, Smith MC: Acute kidney injury: A guide to diagnosis and management. Am Fam Physician
2012;86(7):631-639.

Item 2

ANSWER: A

Tumor necrosis factor (TNF) inhibitors are currently approved by the U.S. Food and Drug Administration
(FDA) for the treatment of rheumatic diseases such as rheumatoid arthritis, ankylosing spondylitis,
psoriatic arthritis, and juvenile idiopathic arthritis. All drugs in this class carry an FDA black-box warning
about the potential for developing primary tuberculosis or reactivating latent tuberculosis. These drugs are
also associated with an increased risk for invasive fungal infections and opportunistic bacterial and viral
diseases. The FDA also warns of reports of lymphomas and other malignancies in children and adolescents
taking these drugs.

A PPD skin test should be performed prior to initiating PNF-inhibitor therapy. An induration of 5 mm or
greater with tuberculin skin testing should be considered a positive test result when assessing whether
treatment for latent tuberculosis is necessary prior to PNF-inhibitor use, even for patients previously
vaccinated with bacille Calmette-Guérin (BCG) (SOR B).

Ankylosing spondylitis patients may develop fibrosis of the upper lung fields with long-standing disease,
but esomeprazole, ondansetron, and meloxicam do not cause reactivation of tuberculosis. Amiodarone is
associated with a subacute cough and progressive dyspnea due to pulmonary toxicity (patchy interstitial
infiltrates).

Ref: Hauck FR, Neese BH, Panchal AS, El-Amin W: Identification and management of latent tuberculosis infection. Am Fam
Physician 2009;79(10):879-886. 2) Information for healthcare professionals: Tumor necrosis factor (TNF) blockers
(marketed as Remicade, Enbrel, Humira, Cimzia, and Simponi). US Food and Drug Administration, 2009. 3) Miller AV,
Ranatunga SK: Immunotherapies in rheumatologic disorders. Med Clin North Am 2012;96(3):475-496.

Item 3

ANSWER: D

To ensure that patients from other cultures understand instructions, it is helpful to ask them to repeat the
instructions in their own words. A website would probably not be specific or culturally sensitive to the
patient’s condition. The physician should speak in a normal tone to the patient, and not to the interpreter.
Family members may be used as convenient translators if necessary, but to maintain confidentiality and
reduce miscommunication it is best to use a trained medical interpreter.

1
Ref: Juckett G: Caring for Latino patients. Am Fam Physician 2013;87(1):48-54.

Item 4

ANSWER: B

An annular rash with a bright red outer border and partial central clearing is characteristic of erythema
migrans. It is important to remember that not all lesions associated with Lyme disease look this way, and
that some patients with Lyme disease may not have any skin lesions at all. Rocky Mountain spotted fever
causes scattered individual purple macules on the ankles and wrists. A dry, scaling, dark red rash in the
groin, with an active border and central clearing, is seen with tinea cruris. A diffuse eruption with clear
vesicles surrounded by reddish macules is found in chickenpox. A migratory pruritic, erythematous,
papular eruption is most consistent with urticaria.

Ref: Habif TP: Clinical Dermatology: A Color Guide to Diagnosis and Therapy, ed 5. Mosby Elsevier, 2010, pp 600-604.

Item 5

ANSWER: B

The recommendation for switching to warfarin in a patient treated with dabigatran is to start warfarin 3
days prior to stopping dabigatran. Bridging with a parenteral agent is not necessary. Dabigatran is known
to increase the INR, so the INR will not reflect warfarin’s effect until dabigatran has been withheld for at
least 2 days.

Ref: Spinler SA, Shafir V: New oral anticoagulants for atrial fibrillation. Circulation 2012;126(1):133-137.

Item 6

ANSWER: B

Average-risk adults should be screened for colon cancer starting at 50 years of age, and high-risk adults
either at age 40 or 10 years before the age at which colorectal cancer was diagnosed in the youngest
affected relative.

Ref: ACP releases best practice advice on colorectal cancer screening. Am Fam Physician 2012;86(12):1153-1154. 2) Qaseem
A, Denberg TD, Hopkins RH Jr; Clinical Guidelines Committee of the American College of Physicians: Screening for
colorectal cancer: A guidance statement from the American College of Physicians. Ann Intern Med 2012;156(5):378-386.

Item 7

ANSWER: B

Metformin has multiple mechanisms of action, but its main effect on serum glucose results from inhibition
of gluconeogenesis in the liver. Sulfonylureas and meglitinides stimulate insulin release from the pancreas,
and thiazolidinediones sensitize peripheral tissues to insulin. Carbohydrate absorption in the small intestine
is inhibited by the "-glucosidase inhibitors.

Ref: Goldman L, Schafer AI (eds): Goldman’s Cecil Medicine, ed 24. Elsevier Saunders, 2011, p e102. 2) Longo DL, Fauci
AS, Kasper DL, et al (eds): Harrison’s Principles of Internal Medicine, ed 18. McGraw-Hill, 2012, p 2996.

2
Item 8

ANSWER: D

Morphine effectively decreases the feeling of shortness of breath in hospice patients. Randomized,
controlled trials have shown significant improvements in symptoms without a significant change in oxygen
saturation. Haloperidol can be used for nausea and vomiting (SOR B) and delirium, but is not helpful in
the treatment of shortness of breath. Scopolamine is used to decrease the production of secretions but is
not helpful for treating dyspnea. Corticosteroids will not manage the sensation of shortness of breath in
a dying patient.

Ref: Reville B, Axelrod D, Maury R: Palliative care for the cancer patient. Prim Care 2009;36(4):781-810.

Item 9

ANSWER: B

Thyroid hormone requirements increase during pregnancy. Most women with hypothyroidism who become
pregnant require an increased levothyroxine dosage (SOR A). A common recommendation is to have
women on fixed daily doses of levothyroxine begin taking nine doses weekly (one extra dose on 2 days
of the week) as soon as the pregnancy is confirmed (SOR B). Thyroid function tests should be repeated
regularly throughout the pregnancy to guide additional dosage adjustments.

Ref: Alexander EK, Marqusee E, Lawrence J, et al: Timing and magnitude of increases in levothyroxine requirements during
pregnancy in women with hypothyroidism. N Engl J Med 2004;351(3):241-249. 2) Gaitonde DY, Rowley KD, Sweeney
LB: Hypothyroidism: An update. Am Fam Physician 2012;86(3):244-251.

Item 10

ANSWER: D

Family medicine journals, including American Family Physician, The Journal of Family Practice, and The
Journal of the American Board of Family Medicine utilize the Strength of Recommendation Taxonomy
(SORT) to label key recommendations in clinical review articles. These grades are assigned on the basis
of the quality and consistency of available evidence. The Cochrane Collaboration is an extensive database
of systematic reviews and clinical trials. A Cochrane review with a clear recommendation warrants a
strength of recommendation rating of A. This indicates consistent, good quality, patient-oriented evidence.
Consistent findings from at least two randomized, controlled studies or a systematic review/meta-analysis
of randomized, controlled trials are also assigned a level A strength of recommendation. Expert opinion
and consensus guidelines are assigned a level C strength of recommendation. SORT also includes a grade
of 1 to 3 for levels of evidence. Retrospective cohort studies are considered level 2.

Ref: Ebell MH: SORT: The strength-of-recommendation taxonomy. Am Fam Physician 2007;76(8):1095-1096. 2) Ebell MH:
How to find answers to clinical questions. Am Fam Physician 2009;79(4):293-296.

3
Item 11

ANSWER: A

Hypercalcemia due to malignancy has a poor prognosis. Up to 80% of cases are due to secretion of
parathyroid hormone–related protein. This is most common with squamous cell carcinomas. Breast cancer,
lymphomas, and multiple myeloma may cause hypercalcemia as a result of osteolytic activity at the site
of the metastasis.

Small cell carcinoma of the lung is a major cause of the syndrome of inappropriate secretion of antidiuretic
hormone (SIADH) and may also cause Cushing syndrome. Prostate cancer can also cause SIADH, and
thyroid cancer can cause Cushing syndrome.

Ref: Pelosof LC, Gerber DE: Paraneoplastic syndromes: An approach to diagnosis and treatment. Mayo Clin Proc
2010;85(9):838-854.

Item 12

ANSWER: C

Recent reports of epidemics of gastroenteritis on cruise ships are consistent with Norovirus infections due
to waterborne or foodborne spread. In the United States these viruses are responsible for about 90% of all
epidemics of nonbacterial gastroenteritis. The noroviruses are common causes of waterborne epidemics
of gastroenteritis, and have been shown to be responsible for outbreaks in nursing homes, on cruise ships,
at summer camps, and in schools. Symptomatic treatment by itself is usually appropriate.

Ref: Glass RI, Parashar UD, Estes MK: Norovirus gastroenteritis. N Engl J Med 2009;361(18):1776-1785. 2) Longo DL, Fauci
AS, Kasper DL, et al (eds): Harrison’s Principles of Internal Medicine, ed 18. McGraw-Hill, 2012, pp 1588-1590.

Item 13

ANSWER: E

Hyperparathyroidism is usually caused by a single adenoma of one of the four parathyroid glands. A
minority of cases (10%–15%) are associated with four-gland hyperplasia. Studies that localize the glands,
such as a technetium scan or ultrasonography, help surgeons who are familiar with this condition achieve
a cure rate of 95%–98%, with an estimated complication rate of 1%–3%. For patients <50 years old or
symptomatic patients, such as those with a fragility fracture, parathyroidectomy is the treatment of choice.
If a patient is older, is a poor surgical candidate, or has asymptomatic disease, long-term monitoring with
treatment focused on reducing bony complications can be considered (SOR C).

Ref: Marcocci C, Cetani F: Clinical practice. Primary hyperparathyroidism. N Engl J Med 2011;365(25):2389-2397. 2) Pallan
S, Rahman MO, Khan AA: Diagnosis and management of primary hyperparathyroidism. BMJ 2012;344:e1013.

4
Item 14

ANSWER: D

Venous thrombosis, both acute and recurrent, is associated with several hematologic abnormalities, in
addition to the well-known factors of trauma, surgery, malignancy, sepsis, and oral contraceptive use.
Notably, activated protein C resistance (factor V Leiden) has been found to be one of the most common
hereditary causes of thrombophilia.

Ref: Goldman L, Schafer AI (eds): Goldman’s Cecil Medicine, ed 24. Elsevier Saunders, 2011, pp 1149-1150.

Item 15

ANSWER: A

Asthma is typically associated with an obstructive impairment that is reversible with short-acting
bronchodilators. A reduced FEV1 and a decreased FEV1/FVC ratio indicates airflow obstruction. A
reduced FVC with a normal or increased FEV1/FVC ratio is consistent with a restrictive pattern of lung
function.

Ref: Expert Panel Report 3: Guidelines for the Diagnosis and Management of Asthma. National Asthma Education and
Prevention Program, 2007, pp 43-45. Available at http://www.nhlbi.nih.gov/guidelines/asthma/asthgdln.pdf.

Item 16

ANSWER: E

While some studies have shown mixed results, there is good evidence that oral acyclovir reduces the
incidence of herpetic neuralgia when given within 72 hours of the onset of the rash, and that it reduces the
duration of symptoms (SOR A). Acyclovir, valacyclovir, and famciclovir have also been shown to reduce
the formation of new lesions, reduce viral shedding, and hasten the resolution of lesions. The effect of
acyclovir on preventing neuralgia appears to be strongest in the first month. Oral or topical corticosteroids
can reduce the duration of the rash and pain in the acute phase. Tricyclic antidepressants and gabapentin
can be used to treat the pain of postherpetic neuralgia if it does develop (SOR A).

Ref: Li O, Chen N, Yang J, et al: Antiviral treatment for preventing postherpetic neuralgia. Cochrane Database Syst Rev
2009;(2):CD006866. 2) Fashner J, Bell AL: Herpes zoster and postherpetic neuralgia: Prevention and management. Am
Fam Physician 2011;83(12):1432-1437.

Item 17

ANSWER: C

When children show signs of dehydration from diarrhea, the first step is to assess its extent. In one study,
four factors predicted dehydration: a capillary refill time >2 seconds, the absence of tears, dry mucous
membranes, and an ill general appearance; the presence of two or more of these signs indicates a fluid
deficit of at least 5%. This child has two of the signs, but does not require intravenous fluids at this point.
Early oral rehydration therapy is recommended and can be started at home. This should be done using an
oral rehydration solution that is designed for children (SOR C). Adult oral rehydration solutions should
not be used in children.

5
Water and other clear liquids, even those with sodium, such as chicken broth, should not replace an oral
rehydration solution because they are hyperosmolar. These fluids do not adequately replace potassium,
bicarbonate, or sodium, and can sometimes cause hyponatremia. Antidiarrheal medications are usually not
recommended for use in children with acute gastroenteritis because they delay the elimination of infectious
agents from the intestines.

Ref: Canavan A, Arant BS Jr: Diagnosis and management of dehydration in children. Am Fam Physician 2009;80(7):692-696.
2) Churgay CA, Aftab Z: Gastroenteritis in children: Part II. Prevention and management. Am Fam Physician
2012;85(11):1066-1070.

Item 18

ANSWER: C

Acute kidney injury (AKI) is currently defined as either a rise in serum creatinine or a reduction in urine
output. Creatinine must increase by at least 0.3 mg/dL, or to 50% above baseline within a 24–48 hour
period. A reduction in urine output to 0.5 mL/kg/hr for longer than 6 hours also meets the criteria. Acute
interstitial nephritis is an intrinsic renal cause of AKI. These patients are often nonoliguric. A history of
recent medication use is key to the diagnosis, as cephalosporins and penicillin analogues are the most
common causes. Approximately one-third of patients present with a maculopapular rash, fever, and
arthralgias. Eosinophilia and sterile pyuria may also be seen in addition to eosinophiluria. Discontinuation
of the offending drug is the cornerstone of management.

Although up to 30% of patients with chronic hepatitis C infection have some kidney involvement, acute
interstitial nephritis is uncommon. Measuring postvoid residual urine volume is indicated if an obstructive
cause for the AKI is suspected. Starting an antibiotic to cover methicillin-resistant Staphylococcus aureus
(MRSA) is not indicated.

Ref: Rahman M, Shad F, Smith MC: Acute kidney injury: A guide to diagnosis and management. Am Fam Physician
2012;86(7):631-639. 2) Longo DL, Fauci AS, Kasper DL, et al (eds): Harrison’s Principles of Internal Medicine, ed 18.
McGraw-Hill, 2012, pp 2293-2308.

Item 19

ANSWER: A

$-Agonists activate potassium uptake by the cells. This includes bronchodilators and tocolytic agents. Other
agents that can induce hypokalemia include pseudoephedrine and insulin. Diuretics, particularly thiazides,
can also cause hypokalemia as a result of the renal loss of potassium.

Ref: Longo DL, Fauci AS, Kasper DL, et al (eds): Harrison’s Principles of Internal Medicine, ed 18. McGraw-Hill, 2012, pp
351-352.

6
Item 20

ANSWER: E

Infantile gonococcal infection is usually the result of exposure to infected cervical exudate during delivery
and manifests 2–5 days after birth. Ophthalmia neonatorum and sepsis are the most severe gonococcal
infections in newborns and immediate treatment is warranted based on the presumptive diagnosis. Topical
antibiotics are appropriate for prophylaxis, but not for treatment. Silver was used for prophylaxis at one
time, but is no longer available. Povidone-iodine has not been studied for prevention. A single dose of
25–50 mg/kg of ceftriaxone administered intravenously or intramuscularly is the recommended treatment.

Ref: Workowski KA, Berman S, CDC: Sexually transmitted diseases treatment guidelines, 2010. MMWR Recomm Rep
2010;59(RR-12):1-110. 2) Update to CDC’s sexually transmitted diseases treatment guidelines, 2010; Oral cephalosporins
no longer a recommended treatment for gonococcal infections. MMWR Morbid Mortal Wkly Rep 2012;61(31):590-594.

Item 21

ANSWER: B

Ipratropium is the only nasally inhaled anticholinergic recommended by the American College of Chest
Physicians for a cough caused by the common cold. One study showed that the nasal formulation decreases
rhinorrhea and sneezing, and a Cochrane review found that ipratropium bromide nasal spray improved
rhinorrhea but did not help nasal stuffiness (SOR B). Antihistamine monotherapy (either sedating or
nonsedating) such as diphenhydramine was no more effective than placebo (SOR A). Corticosteroids have
not been found to be effective for the symptoms of a common cold. Intranasal zinc should not be used
because it may result in the permanent loss of smell.

Ref: Albalawi ZH, Othman SS, Alfaleh K: Intranasal ipratropium bromide for the common cold. Cochrane Database Syst Rev
2011;(7):CD008231. 2) Fashner J, Ericson K, Werner S: Treatment of the common cold in children and adults. Am Fam
Physician 2012;86(2):153-159.

Item 22

ANSWER: C

The CT scan shows a subcapsular hematoma of the kidney. This is considered a grade I injury and does
not require surgical treatment.

Ref: Brunicardi FC (ed): Schwartz’s Principles of Surgery, ed 9. McGraw Hill Medical, 2010, pp 1465-1466. 2) Marx JA (ed):
Rosen’s Emergency Medicine: Concepts and Clinical Practice, ed 7. Mosby Elsevier, 2010, pp 446-450. 3) Tintinalli JE,
Kelen GD, Stapczynski JS (eds): Emergency Medicine: A Comprehensive Study Guide, ed 7. McGraw-Hill, 2011, pp
1773-1774.

Item 23

ANSWER: C

Rubella has been directly responsible for inestimable pregnancy wastage, as well as for severe congenital
malformations. Identification and vaccination of unimmunized women immediately after childbirth or
abortion is recommended. The use of (-globulin to prevent viremia in nonimmune subjects exposed to
rubella is not recommended. The vaccine should be avoided shortly before or during pregnancy since it
is an attenuated live virus. Because of herd immunity there is a very low likelihood that this patient will
be exposed to rubella.

7
Ref: National Center for Immunization and Respiratory Diseases: General recommendations on immunization—Recommendations
of the Advisory Committee on Immunization Practices. MMWR Recomm Rep 2011;60(2):26-27.

Item 24

ANSWER: E

An adult patient with a confirmed systolic blood pressure >139 mm Hg or a diastolic blood pressure >89
mm Hg is hypertensive. JNC-7 guidelines recommend the adoption of healthy lifestyles for all patients,
especially those with hypertension, and the addition of pharmacologic treatment as necessary to reach a
goal blood pressure <140/90 mm Hg. This goal blood pressure is further reduced to <130/80 mm Hg
for patients who also have diabetes mellitus or renal disease. The same guidelines note that elevated
systolic blood pressure is a much more important cardiovascular disease risk factor than diastolic blood
pressure in persons older than age 50.

Medication is commonly required to reach the goal blood pressure, and most patients will often require
two or more drugs. In the absence of compelling indications for use of a specific class of drugs,
thiazide-type diuretics are recommended for initial treatment. Examples of compelling indications include
ACE inhibitors for patients with heart failure, diabetes mellitus, or high coronary disease risk, or
$-blockers post myocardial infarction.

The selection of an agent with favorable side benefits is recommended. Thiazide-type diuretics are useful
in slowing demineralization from osteoporosis, making this the most appropriate choice for this patient.
Other examples of choosing drugs based on side benefits include $-blockers for patients with a history of
migraine or tachycardia, calcium channel blockers for patients with Raynaud’s syndrome, and "-blockers
for patients with benign prostatic hyperplasia.

Ref: JNC 7 Express: The Seventh Report of the Joint National Committee on Prevention, Detection, Evaluation, and Treatment
of High Blood Pressure. National Heart, Lung, and Blood Institute, National High Blood Pressure Education Program,
2003, NIH pub no 03-5233.

Item 25

ANSWER: A

This presentation is consistent with bronchiolitis, which is a response to a viral respiratory infection.
American Academy of Pediatrics guidelines for the management of bronchiolitis do not recommend routine
use of any treatment, recommending instead that the choice be based on the specific needs of the child.
If the child responds to a trial of albuterol, then treatment can be continued; otherwise, evidence shows
no benefit. Antibiotics are indicated for signs of bacterial infection. Oxygen is indicated if the O2 saturation
is <90%. Corticosteroids have not been shown to be of benefit.

Ref: Zorc JJ, Hall CB: Bronchiolitis: Recent evidence on diagnosis and management. Pediatrics 2010;125(2):342-349.

8
Item 26

ANSWER: C

Secondary prevention of fractures is an important component of care following a hip fracture. Options to
consider include bisphosphonates, calcium supplementation, and vitamin D supplementation.
Bisphosphonates, including zoledronic acid, can reduce rates of clinical fractures among patients who have
had a hip fracture (SOR A). While long-term use of bisphosphonates may increase the risk of jaw
osteonecrosis and anemia, a CBC is not necessary before initiating therapy. Contraindications to zoledronic
acid include hypocalcemia and a creatinine clearance <35 mL/min or other evidence of acute renal
impairment.

Ref: FDA Drug Safety Communication: New contraindication and updated warning on kidney impairment for Reclast (zoledronic
acid). US Food and Drug Administration, 2011. 2) Hung WW, Egol KA, Zuckerman JD, Siu AL: Hip fracture
management: Tailoring care for the older patient. JAMA 2012;307(20):2185-2194.

Item 27

ANSWER: A

ACE inhibitors such as lisinopril do not need to be discontinued unless baseline creatinine increases by
>30%. (This patient’s creatinine increased by 20%.) The current dosage of lisinopril is appropriate, as
the blood pressure meets the diabetic goal of <130/80 mm Hg. Small increases in creatinine have been
associated with long-term preservation of renal function, and may be a marker of changes in
intraglomerular pressure.

Ref: Bakris GL, Weir MR: Angiotensin-converting enzyme inhibitor–associated elevations in serum creatinine: Is this a cause
for concern? Arch Intern Med 2000;160(5):685-693. 2) Roett MA, Liegl S, Jabbarpour Y: Diabetic nephropathy—The
family physician’s role. Am Fam Physician 2012;85(9):883-889.

Item 28

ANSWER: E

Overly rapid correction of hyponatremia may cause osmotic demyelination syndrome, or central pontine
myelinolysis, sometimes resulting in permanent neurologic deficits after a brief improvement in neurologic
status. Signs and symptoms may include dysarthria, dysphagia, paresis, coma, and seizures. It is believed
that brain volume shrinks because it cannot assimilate the new electrolytes fast enough and water is lost
from the cells. Rapid correction of hypernatremia that has been present for a short time is relatively safe.
Hyperkalemia is a life-threatening condition that should be corrected promptly. Rapid correction of
hypoglycemia is not an issue. Overly rapid correction of hyperglycemia and subsequent cerebral edema
is unusual and is primarily seen in children.

Ref: Verbalis JG, Goldsmith SR, Greenberg A, et al: Hyponatremia treatment guidelines 2007: Expert panel recommendations.
Am J Med 2007;120(11 Suppl 1):S1-S21. 2) Goldman L, Schafer AI (eds): Goldman’s Cecil Medicine, ed 24. Elsevier
Saunders, 2011, p 731.

9
Item 29

ANSWER: A

Melasma is a very common condition in pregnancy and is due to hyperpigmentation related to normal
hormonal changes that accompany pregnancy. It can also be caused by oral contraceptives and is more
common in dark-skinned persons.

High-potency broad-spectrum sunscreens may help prevent melasma, or at least prevent worsening of the
condition (SOR C). Topical retinoids, hydroquinone, and corticosteroids can also be helpful, but are
usually reserved for postpartum use and require months of treatment. Other treatments include azelaic acid,
chemical peels, kojic acid, cryosurgery, and laser treatment (SOR B). Melasma usually improves
spontaneously after delivery, but it may be prolonged or worsened by oral contraceptive use. It does not
increase the risk of developing skin malignancies.

Ref: Tunzi M, Gray G: Common skin conditions during pregnancy. Am Fam Physician 2007;75(2):211-218. 2) Habif TP:
Clinical Dermatology: A Color Guide to Diagnosis and Therapy, ed 5. Mosby Elsevier, 2010, pp 772-774.

Item 30

ANSWER: D

Hypersensitivity pneumonitis can present in acute, subacute, or chronic forms. The case described includes
two episodes of the acute form. The patient was exposed to mold antigens in his flooded home. Within 4–8
hours of exposure, chills, cough, and shortness of breath will be noted, and at times will be dramatic. A
chest film can be normal, even with significant hypoxia. Pulmonary function tests will show restrictive
changes, as compared to the reversible obstructive changes of acute asthma. Blood tests often show an
elevated erythrocyte sedimentation rate. Serum IgG tests for the probable antigen confirm the diagnosis.

Symptoms of acute hypersensitivity pneumonitis resolve over several days, but will suddenly and violently
recur with repeated exposure to the offending antigen. The subacute form begins gradually over weeks or
months, causing a cough and increasing shortness of breath. The chronic form develops over years of
exposure, causing fibrotic changes to the lungs that will be evident on radiographs, as well as chronic
crackles on auscultation.

Asthma would be an unlikely diagnosis in this case, with the pulmonary function tests showing restrictive
changes rather than obstructive changes, and little improvement with albuterol. Also, the lack of a previous
history of asthma makes it less likely. Legionnaires’ disease is always possible, but is unlikely in this case
given the sudden onset, quick recovery over several days, and sudden recurrence. Pulmonary embolism
is ruled out by the negative D-dimer test.

Ref: Goldman L, Schafer AI (eds): Goldman’s Cecil Medicine, ed 24. Elsevier Saunders, 2011, pp 564-565.

10
Item 31

ANSWER: B

Hyperparathyroidism is present in more than half of patients who have a glomerular filtration rate <60
mL/min, and is independently associated with increased mortality and an increased prevalence of
cardiovascular disease. In patients with stage 4 chronic kidney disease, current guidelines recommend
monitoring of serum calcium and phosphate levels every 3–6 months and bone-specific alkaline
phosphatase activity every 6–12 months with the goal of normalizing these values. The other metabolic
abnormalities listed are less common than hyperparathyroidism.

Ref: Abboud H, Henrick WL: Stage IV chronic kidney disease. N Engl J Med 2010;362(1):56-65.

Item 32

ANSWER: C

Up through July of 2013, the Centers for Disease Control and Prevention had logged 624 cases of
hantavirus pulmonary syndrome in residents of 34 states. The infection killed more than a third of the
victims. The virus is usually spread by inhalation of dried aerosolized deer mouse urine or droppings.
Infected deer mice usually have few outward signs. Other hosts include the white-footed mouse, the rice
rat, and the cotton rat. Other rodents have not been shown to carry the virus.

Ref: Hantavirus. Centers for Disease Control website, 2012. http://www.cdc.gov/hantavirus 2) Centers for Disease Control
and Prevention: Facts about Hantaviruses: What you need to know to prevent the disease Hantavirus Pulmonary Syndrome
(HP). US Dept of Health and Human Services, nd.

Item 33

ANSWER: B

Erythema multiforme (EM) is an acute, often recurrent, inflammatory condition. While it is associated with
many different causes such as drugs, infections, and physical agents, it is most commonly associated with
acute upper respiratory infections, herpes simplex virus, and Mycoplasma pneumoniae.

EM typically occurs in persons 20–40 years of age, with urticarial papules or the classic bull’s-eye or
target lesions (as shown in this case). The distribution is primarily on the hands (both the dorsi and palms),
soles, and extensor aspects of the arms and legs. Lesions may also occur on mucosal surfaces, but in
typical EM these are milder than in the more severe Stevens-Johnson syndrome, which is more commonly
associated with drug exposure than with infectious agents.

The lesions of EM are usually pruritic, but not universally so. They evolve to the typical target forms over
24–48 hr. The central area is often dusky and may be superficially necrotic or vesicular. Partial target
lesions may resemble urticaria. The lesions usually heal without scarring over a 10- to 14-day period.

An id reaction is a papulovesicular eruption, classically on the sides of fingers, that occurs in response to
an intense inflammatory process such as a fungal infection or acute dermatitis in another anatomic area of
the body. When that condition resolves, so does the id reaction.

11
Discoid lupus lesions are irregular but roughly round in shape, sharply demarcated, and most commonly
found on the face or scalp. They usually develop an adherent thick scale. The duration may be months or
years.

Granuloma annulare manifests as a ring of firm, flesh-colored or red papules with a more prominent outer
ring forming due to central involution. These lesions may last for months to years. The distribution is
usually on the dorsolateral aspect of the hands or feet.

Pyoderma gangrenosum begins as a tender papule, usually on the lower leg, and evolves to a painful,
necrotic, inflammatory lesion that gradually enlarges up to 10 cm. It usually is a manifestation of an
underlying systemic inflammatory condition such as inflammatory bowel disease. The lesions last for
months to years and heal with scarring.

Ref: Habif TP: Clinical Dermatology: A Color Guide to Diagnosis and Therapy, ed 5. Mosby Elsevier, 2010, pp 109, 682,
710-714, 737-740, 976-977.

Item 34

ANSWER: E

The results of this urine test were that 10 people had the disease and tested positive (true positives); 8
people had the disease but tested negative (false negatives); 17 people did not have the disease (27 – 10)
but tested positive (false positive); 69 people did not have the disease (77 – 8) and tested negative (true
negative). The negative predictive value is determined by dividing the true negatives (69) by the total
number who tested negative (true plus false negatives = 77). The result is 89.6%, which rounded to a
whole number is 90%.

Ref: Gordis L: Epidemiology, ed 4. Saunders Elsevier, 2008, pp 85-108.

Item 35

ANSWER: A

This patient should be provided with a regular diet, which may promote weight gain in nursing-home
residents with unintentional weight loss. Malnutrition and unintentional weight loss are significant problems
in nursing-home residents and lead to multiple complications, including pressure ulcers and infections. The
American Dietetic Association recommends liberalizing diets to improve nutritional status and quality of
life in older adults. A small study demonstrated equivalent glycemic control in nursing-home residents who
ate a regular diet compared to those who ate a restricted American Diabetes Association diet (SOR C).
Low-salt and low-cholesterol diets are unpalatable and are often associated with protein-energy malnutrition
and postural hypotension in older persons. Special diets should be avoided whenever possible in
nursing-home patients.

Ref: Morley JE, Silver AJ: Nutritional issues in nursing home care. Ann Intern Med 1995;123(11):850-859. 2) Niedert KC;
American Dietetic Association: Position of the American Dietetic Association: Liberalization of the diet prescription
improves quality of life for older adults in long-term care. J Am Diet Assoc 2005;105(12):1955-1965. 3) Unwin BK,
Porvaznik M, Spoelhof GD: Nursing home care: Part II. Clinical aspects. Am Fam Physician 2010;81(10):1229-1237.

12
Item 36

ANSWER: D

Several disorders that are common in women with polycystic ovary syndrome are associated with an
increased risk for endometrial carcinoma, including obesity, hyperinsulinemia, diabetes mellitus,
anovulatory cycles, and high androgen levels.

Ref: Wilson JF: In the clinic. The polycystic ovary syndrome. Ann Intern Med 2011;154(3):ITC2-2-ITC2-15.

Item 37

ANSWER: C

Sinus bradycardia is almost universally present in patients with anorexia nervosa. It is hypothesized that
this is due to vagal hyperactivity resulting from an attempt to decrease the amount of cardiac work by
reducing cardiac output. It is also possible that the bradycardia can be accounted for by low serum T3
levels, a common finding in persons with chronic malnutrition. Sinus tachycardia may occur with refeeding
in patients with anorexia. Other arrhythmias may also occur but are less frequent.

Ref: Casiero D, Frishman WH: Cardiovascular complications of eating disorders. Cardiol Rev 2006;14(5):227–231. 2) Williams
PM, Goodie J, Motsinger CD: Treating eating disorders in primary care. Am Fam Physician 2008;77(2):187-195.

Item 38

ANSWER: C

The recommended treatment regimen for gonorrhea is ceftriaxone, 250 mg intramuscularly. The 125-mg
regimen is no longer recommended because of treatment failures and limited effectiveness in pharyngeal
infections. In addition, the patient should be given azithromycin, 1 g orally, because of the high incidence
of coinfection with Chlamydia, even if testing is negative, and to decrease the risk for cephalosporin
resistance.

Ref: Workowski KA, Berman S; Centers for Disease Control and Prevention (CDC): Sexually transmitted diseases treatment
guidelines, 2010. MMWR Recomm Rep 2010;59(RR-12):1-110. 2) Centers for Disease Control and Prevention (CDC):
Update to CDC’s sexually transmitted diseases treatment guidelines, 2010: Oral cephalosporins no longer a recommended
treatment for gonococcal infections. MMWR Morb Mortal Wkly Rep 2012;61(31):590-594.

Item 39

ANSWER: A

The herpes zoster vaccine is the only live-attenuated virus vaccine listed, and is therefore the one
contraindicated in immunodeficient patients. Human papillomavirus, Tdap, meningococcal, and
pneumococcal polysaccharide vaccines are not live or live-attenuated vaccines, and may be given to
immunocompromised patients.

Ref: Vaughn JA, Miller RA: Update on immunization in adults. Am Fam Physician 2011;84(9):1015-1020.

13
Item 40

ANSWER: B

Polypharmacy is common in the elderly population, but the use of numerous medications is necessary in
some elderly patients. However, some medications have been identified as having a considerably higher
potential to cause problems when prescribed to elderly patients.

In the case described, acetaminophen/diphenhydramine would be an appropriate medication to stop


initially. The older antihistamines cause many adverse CNS effects such as cognitive slowing and delirium
in older patients. These effects are more pronounced in elderly patients with some degree of preexisting
cognitive impairment. The anticholinergic properties of older antihistamines produce effects such as dry
mouth, constipation, blurred vision, and drowsiness. The sedative effect of older antihistamines also
increases the risk of falls. Hip fracture and subsequent death have been reported in patients who use older
antihistamines such as diphenhydramine.

Sertraline is an SSRI, a preferred class for the treatment of depression in the elderly compared to the
tricyclic antidepressants, which are associated with several side effects. Dipyridamole is associated with
hypotension in elderly patients, but it benefits some individuals by preventing strokes. It can be used in
the elderly, but patients should be monitored for side effects. Therefore, until further information is
obtained, it is appropriate to continue the dipyridamole/aspirin in this patient.

When used in elderly patients with heart failure, digoxin should be given in a dosage no greater than 0.125
mg daily; the low dosage used in this individual should not be considered inappropriate until the reason
for its use is clarified. While omeprazole can cause problems in the elderly with long-term use, 20 mg/day
is a relatively low dose and the decision to discontinue its use should be delayed until more history is
available.

Ref: Ramakrishnan K, Scheid DC: Treatment options for insomnia. Am Fam Physician 2007;76(4):517-526. 2) American
Geriatrics Society 2012 Beers Criteria Update Expert Panel: American Geriatrics Society updated Beers Criteria for
potentially inappropriate medication use in older adults. J Am Geriatric Soc 2012;60(4):616-631. 3) Pretorius RW, Gataric
G, Swedlund SK, Miller JR: Reducing the risk of adverse drug events in older adults. Am Fam Physician
2013;87(5):331-336.

Item 41

ANSWER: B

Regional analgesia in laboring patients increases the risk of vacuum- or forceps-assisted delivery (relative
risk = 1.42; 95% confidence interval, 1.28–1.57; 23 trials; n = 735). Multiple randomized, controlled
trials have compared regional analgesia with no analgesia. In a meta-analysis, no statistically significant
impact was found on the risk of cesarean delivery, maternal satisfaction with pain relief, long-term
backache, or immediate effect on neonatal status as determined by Apgar scores. Regional analgesia
provides better pain relief than opioid analgesia.

Ref: Schrock SD, Harraway-Smith C: Labor analgesia. Am Fam Physician 2012;85(5):447-454.

14
Item 42

ANSWER: B

This patient suffers from diarrhea-predominant irritable bowel syndrome (IBS). Her symptoms meet the
Rome III criteria for diagnosis with no alarm features. A CBC, serum chemistries, thyroid function
studies, stool testing for ova and parasites, and abdominal imaging are all low-yield tests that are not
recommended for routine diagnostic evaluation of IBS (SOR C). The association between IBS and bacterial
overgrowth in the small intestine is not clear, so routine hydrogen breath testing is not recommended.
Routine testing for celiac disease, however, should be considered in patients with diarrhea-predominant
or mixed-presentation IBS (SOR C). A systematic review that included more than 4000 patients found that
4% of those with diarrhea-predominant or mixed-presentation IBS had biopsy-proven celiac disease.

Ref: Wilkins T, Pepitone C, Biju A, Schade RR: Diagnosis and management of IBS in adults. Am Fam Physician
2012;86(5):419-426.

Item 43

ANSWER: A

The World Health Organization (WHO) publishes the medical eligibility criteria for contraceptive use as
a guideline for the appropriate use of contraceptives. There are four categories that define the
appropriateness of contraceptive use in women with certain medical problems:

Category 1: A condition for which there is no restriction for the use of the contraceptive method.

Category 2: A condition for which the advantages of using the method generally outweigh the theoretical
or proven risks.

Category 3: A condition for which the theoretical risk or proven risks usually outweigh the advantages of
using the method.

Category 4: A condition that represents an unacceptable health risk if the contraceptive method is used.

A history of migraine with aura is classified as category 4 for oral contraceptives. Women with a history
of migraines are 2–4 times as likely to have a stroke compared to women without migraines, and women
who have an aura associated with their migraines are at even higher risk. Migraine without aura is
classified as category 2 in women younger than 35 and category 3 in women 35 or older. Nonmigrainous
headaches are category 1, as is chronic hepatitis C.

Combined oral contraceptive use does not appear to increase the rate or severity of cirrhotic fibrosis and
there is no increased risk for hepatocellular carcinoma. Combined oral contraceptives are not
recommended for use in women with acute hepatitis C. Breastfeeding is considered category 2 by the CDC
and category 3 by WHO. There is conflicting evidence about the effects on the volume of breast milk in
women who are on combined oral contraceptives, but the concerns are mainly during the first month of
the postpartum period. There have not been any demonstrated adverse health effects in infants exposed to
combined oral contraceptives through breast milk. Laparoscopic banding weight-loss surgery is category
1 for combined oral contraceptive use. Evidence shows no significant decrease in the effectiveness of oral
contraceptives in women who have had this surgery. Combined oral contraceptive use in patients with
endometrial hyperplasia is category 1. Combined oral contraceptives have been used to decrease the risk
for endometrial cancer.

15
Ref: Medical Eligibility Criteria for Contraceptive Use, ed 4. World Health Organization, 2009, pp 11-12, 16-27. 2) US
Medical Eligibility Criteria for Contraceptive Use, 2010, June 18, 2010. MMWR Recomm Rep 2010;59(RR-4):11-18.

Item 44

ANSWER: C

This patient has secondary hyperparathyroidism, a common cause of hypercalcemia in patients with chronic
renal insufficiency. Sevelamer is a newer synthetic agent in the therapeutic class of phosphate binders,
which includes calcium acetate. Decreasing serum phosphate lowers the feedback stimulation of
parathyroid hormone secretion by the parathyroid gland, which is often excessive in chronic renal
insufficiency. Normalizing parathyroid levels improves serum calcium levels.

Ref: Longo DL, Fauci AS, Kasper DL, et al (eds): Harrison’s Principles of Internal Medicine, ed 18. McGraw-Hill, 2012, pp
3096-3120.

Item 45

ANSWER: D

Acute otitis externa is treated with topical antibiotics. Although no preparation has been shown to be most
effective, a fluoroquinolone does not create a risk of ototoxicity if the tympanic membrane is not intact.
Topical corticosteroids may hasten symptom reduction. Oral antibiotics are not appropriate unless the
infection has spread beyond the ear canal, or if the patient has diabetes mellitus or is immunocompromised.

Ref: Schaefer P, Baugh RF: Acute otitis externa: An update. Am Fam Physician 2012;86(11):1055-1061.

Item 46

ANSWER: D

Childhood obstructive sleep apnea syndrome has a prevalence rate of 5.7%. It is associated with growth,
cardiovascular, and neurobehavioral abnormalities. Adenotonsillectomy is the treatment of choice.
Although CPAP can be effective, compliance is poor and it is therefore not a first-line treatment. Intranasal
corticosteroids may also be helpful, but the benefit appears small.

Ref: Marcus CL, Brooks LJ, Draper KA, et al: Diagnosis and management of childhood obstructive sleep apnea syndrome.
Pediatrics 2012;130(3):e714-e755.

Item 47

ANSWER: D

Urine pH is an important factor in the production of kidney stones. Uric acid, cystine, and calcium oxalate
stones tend to form in acidic urine, whereas struvite (magnesium ammonium phosphate) and calcium
phosphate stones form in alkaline urine. Urine should be acidified for prevention of calcium phosphate and
struvite stones. Cranberry juice or betaine can lower urine pH.

Ref: Frassetto L, Kohlstadt I: Treatment and prevention of kidney stones: An update. Am Fam Physician
2011;84(11):1234-1242.

16
Item 48

ANSWER: B

Calcaneal apophysitis, also known as Sever disease, is the most common etiology of heel pain in children,
usually occurring between 5 and 11 years of age. It is thought that in these children the bones grow faster
than the muscles and tendons. A tight Achilles tendon then pulls on its insertion site at the posterior
calcaneus with repetitive running or jumping activities, causing microtrauma to the area. There may be
swelling and tenderness in this area and passive dorsiflexion may increase the pain. Radiography is usually
normal and therefore does not often aid in the diagnosis, but it may reveal a fragmented or sclerotic
calcaneal apophysis. Treatment involves decreasing pain-inducing activities, anti-inflammatory or analgesic
medication if needed, ice, stretching and strengthening of the gastrocnemius-soleus complex, and the use
of orthotic devices.

Plantar fasciitis and heel pad syndrome cause pain on the plantar surface of the heel rather than posteriorly.
Achilles tendinopathy causes tenderness to palpation of the Achilles tendon. Tarsal tunnel syndrome related
to compression of the posterior tibial nerve causes neuropathic pain and numbness in the posteromedial
ankle and heel.

Ref: Tu P, Bytomski JR: Diagnosis of heel pain. Am Fam Physician 2011;84(8):909-916.

Item 49

ANSWER: D

This patient’s age and his history of hypertension give him a CHADS2 score of 2, which implies that the
patient is at a higher risk of thromboembolism and should be anticoagulated with warfarin and not aspirin.
The patient should be fully anticoagulated for a minimum of 3 weeks prior to any attempt at cardioversion.

Ref: Gutierrez C, Blanchard DG: Atrial fibrillation: Diagnosis and treatment. Am Fam Physician 2011;83(1):61-68.

Item 50

ANSWER: B

This patient has a Revised Cardiac Risk Index (Goldman Index) score of 1, placing him in a low-risk group
for perioperative cardiac complications. Low-risk patients who are able to walk for 2 blocks or climb a
flight of stairs without stopping to rest (4 METS) do not need noninvasive cardiac testing. Patients in this
risk group who are already on a $-blocker should continue it, but adding one preoperatively may increase
risk. Stopping aspirin therapy in patients with coronary stents places them at risk for perioperative cardiac
events. Surgical bleeding is somewhat increased in patients on aspirin, but differences in the severity of
bleeding events and mortality in surgical patients on low-dose aspirin versus controls are minimal. Stopping
clopidogrel in patients who have recently undergone coronary stent placement (6 weeks for bare-metal
stents, 1 year for drug-eluting stents) markedly increases risk, but there is no need to start clopidogrel in
other patients. Perioperative statin therapy should be continued for all patients undergoing surgery. For
patients undergoing vascular therapy, statins have been associated with an improvement in postoperative
cardiac outcomes.

Ref: Holt NF: Perioperative cardiac risk reduction. Am Fam Physician 2012;85(3):239-246. 2) Drugs for lipids. Treat Guidel
Med Lett 2012;9(103):13-20.

17
Item 51

ANSWER: B

Comedones are noninflammatory acne lesions. Inflammatory lesions include papules, pustules, and
nodules. Grading acne based on the type of lesion and severity helps guide therapy. Topical retinoids
prevent the formation of comedones and reduce their number, and are indicated as monotherapy for
noninflammatory acne. Topical antibiotics are used primarily for the treatment of mild to moderate
inflammatory or mixed acne. Oral antibiotics are effective for the treatment of moderate to severe acne.
Oral isotretinoin is reserved for treatment of severe, recalcitrant acne.

Ref: Titus S, Hodge J: Diagnosis and treatment of acne. Am Fam Physician 2012;86(8):734-740.

Item 52

ANSWER: E

The most likely diagnosis for this patient is stable slipped capital femoral epiphysis (SCFE). This is more
frequent in males than in females, and is more common in African-Americans and Pacific Islanders than
in whites. Although some patients present with pain, many present with a painless limp or vague pain. The
average age of onset is 13.5 years for males and 12 years for females. Obesity is strongly associated with
SCFE.

The lack of systemic symptoms makes osteomyelitis, abscess, or a septic joint much less likely.
Malignancy is a possibility, but night pain would be more likely. Sacroiliitis is much less likely given a
negative FABER test. The patient’s age makes transient synovitis or Legg-Calvé-Perthes disease less
likely. Although muscle strain is a possibility, the physical examination findings of external rotation
deformity and limited internal rotation are more specific for SCFE.

Once the diagnosis of SCFE is made, the patient should not bear weight and should be referred promptly
for surgery to prevent complications.

Ref: Gholve PA, Cameron DB, Millis MB: Slipped capital femoral epiphysis update. Curr Opin Pediatr 2009;21(1):39-45. 2)
Sawyer JR, Kapoor M: The limping child: A systematic approach to diagnosis. Am Fam Physician 2009;79(3):215-224.
3) Peck D: Slipped capital femoral epiphysis: Diagnosis and management. Am Fam Physician 2010;82(3):258-262.

Item 53

ANSWER: C

Methimazole and propylthiouracil (PTU) are the two oral antithyroid medications available. However,
because of reports of severe hepatocellular damage, methimazole should be used instead of PTU unless
it is contraindicated. Radioactive iodine treatment (131I) is an option, especially for patients who do not
achieve remission with antithyroid medications. However, worsening of preexisting orbitopathy is a
well-recognized potential complication of 131I treatment, as well as a transient increase in thyroid hormone
levels that can precipitate thyroid storm. Thus, patients with elevated free T3 or free T4 levels should be
treated with methimazole prior to 131I administration. Thyroidectomy is most often recommended for
patients with thyroid nodules and those who are suspected of having cancer or who do not tolerate or refuse
alternative forms of therapy. However, antithyroid medication should be given to achieve a euthyroid state
prior to surgery in most patients.

18
Ref: McDermott MT: Hyperthyroidism. Ann Intern Med 2012;157(1):ITC1-ITC16.

Item 54

ANSWER: E

Obesity increases the risk of a variety of medical conditions, including type 2 diabetes mellitus,
hypertension, hyperlipidemia, pulmonary disease, coronary artery disease, gallstones, fatty liver disease,
obstructive sleep apnea, GERD, osteoarthritis, and a variety of forms of cancer. A weight loss of at least
10% for greater than 1 year leads to statistically significant improvement in lipid ratios, blood glucose
homeostasis, and coronary artery disease risk reduction.

The AAFP recommends screening for obesity and intensive counseling (more than 1 session per month
for more than 3 months) with behavior modification for obese patients. Counseling is ineffective by itself
and must be combined with lifestyle modification.

Dietary modification, increased physical activity, and behavior modification are effective for maintaining
modest weight loss for greater than 1 year (SOR B). However, there are few large, randomized, controlled
trials with subjects maintaining weight reductions of 10% for over 1 year, even when combining therapy,
exercise, and dietary restriction. Long-term pharmacotherapy can lead to weight loss, but regaining some
weight is typical.

Bariatric surgery leads to the most effective weight reduction and long-term maintenance in patients who
are morbidly obese (SOR A). Gastric bypass is effective, with a mean weight loss of 71.2% at 3 years;
with laparoscopic gastric banding the mean weight loss is 55.2% at 3 years. In one study, 94% of gastric
bypass patients maintained at least a 20% weight loss at 6 years. Bariatric surgery has also been shown
to significantly reduce fasting blood glucose, with resolution of diabetes mellitus in 31%–77% of lap band
patients and 72%–100% of gastric bypass patients. Bariatric surgery is a safe and effective means for
long-term weight loss and should be considered in adults with a BMI >40, or >35 with obesity-related
comorbidities.

Ref: Grief SN, Miranda RL: Weight loss maintenance. Am Fam Physician 2010;82(6):630-634. 2) Rao G: Office-based
strategies for the management of obesity. Am Fam Physician 2010;81(12):1449-1456. 3) Adams TD, Davidson LE, Litwin
SE, et al: Health benefits of gastric bypass surgery after 6 years. JAMA 2012;308(11):1122-1131.

Item 55

ANSWER: D

This patient has signs and symptoms of lateral epicondylitis, also known as tennis elbow, or alternatively
as lateral epicondylalgia to reflect the noninflammatory nature of the condition. This is an overuse
tendinopathy of the common extensor tendon origin of the lateral elbow. Conservative care that includes
offloading the involved tendons is the key to improving outcomes at 1 year, which would mean modifying
this patient’s work. Physical therapy can improve pain and function in the short term, but has not been
shown to improve long-term outcomes at 1 year in randomized trials (SOR A). The evidence is weaker
for bracing, with some studies showing improved pain and function at 3–6 weeks (SOR B). Recent
randomized, controlled trials have made it clear that while corticosteroid injections reduce acute pain for
up to 6 weeks, their use increases rates of poor long-term outcomes (SOR A).

19
Ref: Johnson GW, Cadwallader K, Scheffel SB, Epperly TD: Treatment of lateral epicondylitis. Am Fam Physician
2007;76(6):843-848. 2) Orchard J, Kountouris A: The management of tennis elbow. BMJ 2011;342:d2687. 3) Coombes
BK, Bisset L, Brooks P, et al: Effect of corticosteroid injection, physiotherapy, or both on clinical outcomes in patients with
unilateral lateral epicondylalgia: A randomized controlled trial. JAMA 2013;309(5):461-469.

Item 56

ANSWER: A

Many elderly patients with heart failure are hospitalized repeatedly at short intervals. As described in a
2012 Cochrane review of 25 randomized, controlled trials, there is now good evidence that case
management interventions led by a heart failure specialist nurse reduce heart failure readmissions, all-cause
readmissions, and all-cause mortality. Case management interventions include home care, telephone calls,
patient education, self-management, and face-to-face visits. It is not possible to say which specific
interventions were optimal, but telephone follow-up was a common component in most of these trials.
Multidisciplinary interventions may also be effective. For patients treated with lisinopril, 40 mg/day is the
maximum amount recommended for heart failure. While $-adrenergic blockers and aldosterone antagonists
are used in the treatment of heart failure, prophylactic amiodarone would not be expected to help and may
be harmful. Admission to a nursing home may reduce hospitalization, but it would also increase overall
costs of care.

Ref: Bonow RO, Mann DL, Zipes DP, Libby P (eds): Braunwald’s Heart Disease: A Textbook of Cardiovascular Medicine,
ed 9. Elsevier Saunders, 2011, pp 1039-1041. 2) Takeda A, Taylor SJ, Taylor RS, et al: Clinical service organisation for
heart failure. Cochrane Database Syst Rev 2012;9:CD002752.

Item 57

ANSWER: A

B-type natriuretic peptide (BNP) is a 32–amino acid polypeptide secreted from the cardiac ventricles in
response to ventricular volume expansion and pressure overload. The major source of BNP is the cardiac
ventricles, and because of the minimal presence of BNP in storage granules, its release is directly
proportional to ventricular dysfunction. A BNP test is simple and time efficient, and reliably predicts the
presence or absence of left ventricular dysfunction on an echocardiogram.

Ref: Krishnaswamy P, Lubien E, Clopton P, et al: Utility of B-natriuretic peptide levels in identifying patients with left
ventricular systolic or diastolic function. Am J Med 2001;111(4):274-279. 2) Bonow RO, Mann DL, Zipes DP, Libby P
(eds): Braunwald’s Heart Disease: A Textbook of Cardiovascular Medicine, ed 9. Elsevier Saunders, 2011, pp 505-516.

Item 58

ANSWER: D

Nonbenzodiazepine muscle relaxants such as cyclobenzaprine are beneficial for the relief of acute low back
pain for the first 7–14 days after the onset of symptoms. Patient education, physical therapy, and the
application of ice or heat may also help. Unsupported treatment options for acute low back pain include
oral corticosteroids, acupuncture, lumbar support, massage, chiropractic spinal manipulation, and traction.
Bed rest for acute low back pain is inadvisable.

Ref: Casazza BA: Diagnosis and treatment of acute low back pain. Am Fam Physician 2012;85(4):343-350.

20
Item 59

ANSWER: B

Many infections of the respiratory tract have a viral etiology, and when this is the case early antibiotic
treatment offers little to no benefit. Once the clinical course of a respiratory illness exceeds the expected
length for a viral illness, it may be proper to initiate antibiotic treatment for a suspected atypical or
secondary bacterial infection. Epiglottitis is one exception to this approach because of the possibility of a
bacterial infection, particularly with Haemophilus influenzae type b, that can produce a rapidly worsening,
potentially fatal airway compromise. When epiglottitis is suspected based on findings such as hoarseness,
dysphagia, stridor, drooling, fever, chills, and respiratory distress, intravenous antibiotic treatment should
be instituted immediately, ideally with a $-lactam drug that exhibits activity against methicillin-resistant
Staphylococcus aureus.

Ref: Zoorob R, Sidani M, Fremont RD, Kihlberg C: Antibiotic use in acute upper respiratory tract infections. Am Fam
Physician 2012;86(9):817-822.

Item 60

ANSWER: C

Some experts recommend inpatient observation for all children with apparent life-threatening events such
as this. Hospital admission is not always necessary, however, for a short, self-correcting episode associated
with feeding. Given the history of not breathing for 20–25 seconds, having a blue tongue and lips, and
being limp, admitting the child for observation is appropriate.

Although child abuse is a concern, referral to child protective services is not mandatory. Laryngoscopy
would not be routine, but might be appropriate in some cases depending on the history and physical
findings. Many groups recommend home apnea monitoring after discharge for patients with more severe
or undiagnosed cases.

Ref: Warren J, Biagioli F, Hamilton A, Smith PC: FPIN’s clinical inquiries: Evaluation of apparent life-threatening events in
infants. Am Fam Physician 2007;76(1):124-126. 2) Fu LY, Moon RY: Apparent life-threatening events: An update. Pediatr
Rev 2012;33(8):361-368.

Item 61

ANSWER: E

Cefazolin is appropriate for intrapartum prophylaxis against group B Streptococcus (GBS) in


penicillin-allergic patients who do not have a history of anaphylaxis, urticaria, angioedema, or respiratory
distress. Depending on the antibiotic sensitivity of the GBS organism, either vancomycin or clindamycin
is recommended for patients at higher risk for anaphylaxis.

Ref: Verani JR, McGee L, Schrag SJ, et al: Prevention of perinatal group B streptococcal disease—Revised guidelines from
CDC, 2010. MMWR Recomm Rep 2010;59(RR-10):1-36.

21
Item 62

ANSWER: D

The U.S. Preventive Services Task Force has concluded that the net benefit of daily aspirin is substantial
in women 55–79 years of age for whom the benefit of ischemic stroke prevention exceeds the harm of an
increased risk for gastrointestinal bleeding (SOR A). Aspirin use is recommended in men 45–79 years of
age for prevention of myocardial infarction when the potential benefit outweighs the potential harm of
gastrointestinal hemorrhage (SOR A).

Ref: Aspirin for the prevention of cardiovascular disease: Recommendation statement. US Preventive Services Task Force, 2009.
2) Aspirin for the prevention of cardiovascular disease: Recommendation statement. Am Fam Physician
2011;83(12):1464-1468.

Item 63

ANSWER: E

Pretibial myxedema is a complication of Graves disease, whether it presents as hypo- or hyperthyroidism.


It is a dermopathy that most often occurs in the lower legs and results from increased deposition of mucin
due to the endocrine abnormality. Diabetes mellitus can cause necrobiosis lipoidica, a lesion on the lower
extremities; hyperlipidemia can cause waxy papules; and collagen vascular and ischemic disease can cause
urticaria and/or ulceration.

Ref: Goldman L, Schafer AI (eds): Goldman’s Cecil Medicine, ed 24. Elsevier Saunders, 2011, pp e69-e70.

Item 64

ANSWER: B

Laryngoscopy should be performed to visualize the larynx and evaluate for vocal cord pathology in a
patient whose hoarseness does not resolve within 3 months (SOR C). If a serious condition is suspected
for some other reason, laryngoscopy should be performed regardless of the duration of symptoms. If there
is a recent history of upper respiratory infection or vocal abuse, then it would be appropriate to recommend
voice rest for 2 weeks. Laryngoscopy would then be indicated if the hoarseness did not improve or
recurred after voice rest. For patients with symptoms of gastroesophageal reflux, a trial of a proton pump
inhibitor is recommended (SOR B). Inhaled corticosteroids, especially fluticasone, may cause hoarseness.
Oral corticosteroids do not have a role in the management of hoarseness.

Ref: Feierabend RH, Shahram MN: Hoarseness in adults. Am Fam Physician 2009;80(4):363-370. 2) Schwartz SR, Cohen SM,
Dailey SH, et al: Clinical practice guideline: Hoarseness (dysphonia). Otolaryngol Head Neck Surg 2009;141(3 Suppl
2):S1-S31.

Item 65

ANSWER: D

The U.S. Preventive Services Task Force (USPSTF) recommends one-time screening for abdominal aortic
aneurysm (AAA) by ultrasonography in men age 65–75 who have ever smoked (Grade B
recommendation). The USPSTF recommends against routine screening for AAA in women (Grade D
recommendation).

22
Ref: US Preventive Services Task Force: Screening for Abdominal Aortic Aneurysm: Recommendation Statement. AHRQ pub
no 05-0569-A, 2005.

Item 66

ANSWER: B

Pneumonia caused by Legionella pneumophila is commonly preceded by nonspecific systemic symptoms


that may lead a clinician to consider other diagnoses. Symptoms may include high-grade fever, malaise,
myalgias, anorexia, and headache. Gastrointestinal and neurologic symptoms are also common and include
nausea, vomiting, abdominal pain, diarrhea, and confusion. Focal neurologic signs are less common, but
have been reported. Localizing respiratory symptoms will typically develop later, most often a dry cough
and dyspnea. From this point on the illness resembles a typical pneumonia with fever, productive cough,
pleuritic pain, and breathlessness.

Ref: Darby J, Buising K: Could it be Legionella? Aust Fam Physician 2008;37(10):812-815.

Item 67

ANSWER: D

$-Blockers are first-line antihypertensive medications for patients with coronary artery disease (CAD) and
have been shown to reduce the risk of death by 23% at 2 years. They should also be given to normotensive
patients with CAD if tolerated. Cardioselective ($1) $-blockers such as metoprolol and atenolol are
preferred, as they cause fewer adverse effects.

Ref: Pflieger M, Winslow BT, Mills K, Dauber IM: Medical management of stable coronary artery disease. Am Fam Physician
2011;83(7):819-826.

Item 68

ANSWER: B

Bronchiectasis is an illness of the bronchi and bronchioles involving obstructive and infectious processes
that injure airways and cause luminal dilation. In addition to daily viscid, often purulent sputum production
with occasional hemoptysis, wheezing and dyspnea occur in 75% of patients. Emphysema and chronic
bronchitis, forms of COPD, also cause a decreased FEV1/FVC ratio, but the sputum is generally mucoid
and luminal dilation of bronchi is not characteristically present. Bronchiolitis is usually secondary to
respiratory syncytial virus infection in young children. Asthma is not characterized by the sputum and CT
findings seen in this patient.

Ref: Barker AF: Bronchiectasis. N Engl J Med 2002;346(18):1383-1393. 2) Holmes RL, Fadden CT: Evaluation of the patient
with chronic cough. Am Fam Physician 2004;69(9):2159-2166, 2169. 3) Benich JJ, Carek PJ: Evaluation of the patient
with chronic cough. Am Fam Physician 2011;84(8):887-892. 4) Goldman L, Schafer AI (eds): Goldman’s Cecil Medicine,
ed 24. Elsevier Saunders, 2011, pp 548-550.

Item 69

ANSWER: B

Multiple myeloma can be asymptomatic, but it becomes symptomatic when there is organ damage or other
abnormalities, including renal insufficiency, elevated calcium, anemia, and bone disease.

23
The majority of patients have bone pain, but hypocalcemia is not common. Hypokalemia almost never
occurs, and both hepatic failure and insomnia are not usual signs of multiple myeloma. Anemia typically
occurs either because of renal failure or infiltration of the bone marrow by myeloma cells. Polycythemia
does not occur.

Ref: Palumbo A, Anderson K: Multiple myeloma. N Engl J Med 2011;363(11):1046-1058.

Item 70

ANSWER: C

For patients undergoing major orthopedic surgery, the American College of Chest Physicians recommends
outpatient thromboprophylaxis for a duration of up to 35 days. Older recommendations for 10–14 days of
prophylaxis were based on studies performed when this was the usual hospital stay. This is still
recommended as the minimum length for prophylaxis, but a longer period of time is preferred.

Ref: Guyatt GH, Akl EA, Crowther M, et al: Executive summary: Antithrombotic therapy and prevention of thrombosis, 9th
ed: American College of Chest Physicians Evidence-Based Clinical Practice Guidelines. Chest 2012;141(2 Suppl):7S-47S.

Item 71

ANSWER: D

Based on this patient’s history and physical examination, COPD is the most likely cause of his dyspnea.
Initial testing should include spirometry to diagnose airflow obstruction (SOR C). CT, a BNP level, a
D-dimer level, and arterial blood gas measurements would not be the best initial tests in the evaluation of
this patient’s dyspnea.

Ref: Wahls SA: Causes and evaluation of chronic dyspnea. Am Fam Physician 2012;86(2):173-180.

Item 72

ANSWER: A

During the treatment of severe preeclampsia with intravenous magnesium, the occurrence of apnea and
areflexia is most consistent with magnesium toxicity. In addition to hemodynamic support, calcium infusion
is recommended as an antidote. Calcium chloride can be used if a central line has been established.
Calcium gluconate would be safer with a peripheral intravenous site.

Lorazepam, phenytoin, and fosphenytoin are less useful in preventing eclamptic seizures than magnesium.
Labetalol is not indicated given the patient’s current blood pressure level. Dopamine, a pressor agent, is
not indicated in this scenario, and could aggravate the patient’s preeclampsia.

Ref: Vanden Hoek TL, Morrison LJ, Shuster M, et al: Part 12: Cardiac arrest in special situations: 2010 American Heart
Association Guidelines for Cardiopulmonary Resuscitation and Emergency Cardiovascular Care. Circulation 2010;122(18
Suppl 3):S829-S861. 2) Deak TM, Moskovitz JB: Hypertension and pregnancy. Emerg Med Clin North Am 2012;30(4):
903–917.

24
Item 73

ANSWER: E

This patient has cluster headaches. Most people with cluster headaches are male. These headaches typically
present with severe unilateral pain that lasts from 15 minutes to 3 hours. The pain is generally extremely
sharp, continuous, and incapacitating. In addition to the pain, the headaches are associated with at least
one of the following ipsilateral signs: conjunctival injection, lacrimation, nasal congestion, miosis or ptosis,
eye edema, and forehead and facial sweating. Patients may also have a sense of restlessness or agitation.
The headaches occur anywhere from every other day up to 8 times a day, often in cycles for 4–12 weeks.
Cluster headaches respond to most of the same medications as migraine headaches (DHE, ergotamines,
triptans). They also respond well to 100% oxygen therapy.

Paroxysmal hemicranias are very unusual and present with a similar type of pain, but the attacks are
usually short and they are more common in women. Medication rebound headaches tend to be diffuse,
bilateral, almost daily headaches. These occur in people who are overusing medications, and they tend to
get worse with physical or mental exertion. Temporal arteritis usually occurs in older adults. Migraines
are also often unilateral but they are usually pulsatile, and are associated with nausea and vomiting or
photophobia and phonophobia.

Ref: Rakel RE, Rakel DP (eds): Textbook of Family Medicine, ed 8. Elsevier Saunders, 2011, pp 962-965.

Item 74

ANSWER: C

The most common form of advance directive is a patient’s conversations with relatives and friends, and
these carry the same ethical and legal weight as written directives. Neither the hospital attorney nor the
patient’s personal attorney, in the absence of a previous discussion with the patient, has the ethical or legal
authority to make the decision. Since the patient has previously expressed his wishes, it is unnecessary to
have the court appoint a surrogate decision maker. Care should be transferred to another physician only
if the original physician has a philosophical or religious objection to carrying out the patient’s wishes.

Ref: Halter JB, Ouslander JG, Tinetti ME, et al (eds): Hazzard’s Geriatric Medicine and Gerontology, ed 6. McGraw-Hill
Medical, 2009, pp 388-389.

Item 75

ANSWER: A

Patients who require inhalation therapy with $2-adrenergic-receptor agonists more than twice weekly but
not daily have mild persistent asthma. Long-term control with inhaled corticosteroids is recommended for
adults with persistent asthma.

Ref: Expert Panel Report 3: Guidelines for the Diagnosis and Management of Asthma. National Asthma Education and
Prevention Program, NIH pub no 07-4051, 2007, pp 211-217.

25
Item 76

ANSWER: C

Inflammatory bowel disease is divided into two categories: Crohn’s disease and ulcerative colitis.
Noncontiguous or “skip” lesions that are transmural in nature and are found throughout the gastrointestinal
tract make a diagnosis of Crohn’s disease likely in this patient. Corticosteroids are more effective in
inducing remission than placebo and 5-ASA products (SOR A). A Cochrane review revealed no difference
between elemental and nonelemental diets with regard to symptom remission (SOR A). Anti-TNF agents
such as infliximab should be considered in patients with moderate to severe Crohn’s disease who do not
respond to initial corticosteroid or immunosuppressive therapy, but these are not recommended for initial
treatment. While antibiotics such as metronidazole are widely used for both their anti-inflammatory and
anti-infectious properties, controlled trials have not demonstrated their effectiveness. Surgical intervention
should be considered in patients with ulcerative colitis, but surgery is not indicated for Crohn’s disease.

Ref: Langan RC, Gotsch PB, Krafczyk MA, Skillinge DD: Ulcerative colitis: Diagnosis and treatment. Am Fam Physician
2007;76(9):1323-1330. 2) Wilkins T, Jarvis K, Patel J: Diagnosis and management of Crohn’s disease. Am Fam Physician
2011;84(12):1365-1375.

Item 77

ANSWER: A

Pharyngitis is a common complaint, and usually has a viral cause. The key factors in diagnosing
streptococcal pharyngitis are a fever over 100.4°F, tonsillar exudates, anterior cervical lymphadenopathy,
and absence of cough. The scenario described is consistent with a viral infection, with no risk factors to
make streptococcal infection likely; therefore, this patient should be offered symptomatic treatment only.
Testing for other infections is not indicated unless the patient worsens or does not improve.

Ref: Chan TV: The patient with sore throat. Med Clin North Am 2010;94(5):923-943.

Item 78

ANSWER: B

Many minority populations in the United States, including Hispanics, suffer health care disparities. Some
medical problems are also more common in certain populations. In 2009, Hispanics made up
approximately 16% of the U.S. population and were the largest minority. This group is projected to make
up 30% of the U.S. population by 2050. The rates of obesity and diabetes mellitus in Hispanics are
disproportionately higher than those of non-Hispanic whites. Hypertension is closely linked to obesity, and
Hispanics have higher rates of hypertension as well. Neurocysticercosis is the most common cause of
seizures in Hispanic immigrants. Risk factors for osteoporosis include female gender, non-Hispanic white
ethnicity, smoking, and low BMI.

Ref: Juckett G: Caring for Latino patients. Am Fam Physician 2013;87(1):48-54.

26
Item 79

ANSWER: B

While there is significant overlap in the symptoms of each condition, there are some signs and symptoms
that help the family physician determine whether a terminally ill patient is experiencing grief or has major
depression. This distinction is important because the terminally ill patient with depression would likely
benefit from antidepressant medication, whereas a patient with end-of-life grief is generally best managed
without psychotherapeutic medications.

The key clinical feature in distinguishing the two conditions is in the pervasiveness of symptoms in
depression, particularly the loss of pleasure or interest in all activities. Episodic feelings of guilt, anxiety,
and helplessness, and even thoughts of wanting to die can and do occur with grief reactions, but these
feelings are not constant and over time the symptoms gradually wane. Terminally ill patients with major
depression feel helplessly hopeless all the time, but they often respond to and significantly benefit from
antidepressant medication (SOR A).

Ref: Widera EW, Block SD: Managing grief and depression at the end of life. Am Fam Physician 2012;86(3):259-264.

Item 80

ANSWER: B

This patient has aortic stenosis, which is most likely of the degenerative, age-related type. An
echocardiogram should be included in this patient’s evaluation, but the diagnosis can still be made on the
basis of the auscultatory findings. The second heart sound (closure of the aortic valve then the pulmonic
valve) is normally split, consisting of two distinct components during inspiration. As the aortic valve
becomes more rigid with the progression of aortic stenosis, the valve closures occur simultaneously or even
become paradoxically split during expiration.

The murmur of aortic stenosis is a midsystolic ejection-type murmur, heard loudest at the right second
intercostal space parasternally. It may radiate to the right carotid, and to a lesser degree may also be heard
in the fourth intercostal space at the left lower sternal border. An S3 gallop is heard only late in the
progression of aortic stenosis and is associated with left ventricular dilatation and heart failure.

The murmur of aortic regurgitation is a diastolic descrescendo blowing-type murmur best heard at the left
fourth intercostal space at the lower left sternal border, with the patient sitting and leaning forward. Mitral
valve murmurs are typically located at the heart apex or even in the left infra-axillary area. Mitral valve
prolapse is characterized by a prominent systolic click, often with a brief mitral regurgitation murmur.

Ref: Maganti K, Rigolin VH, Serano ME, Bonow RO: Valvular heart disease: Diagnosis and management. Mayo Clin Proc
2010;85(5):483-500.

Item 81

ANSWER: C

Although patients with chronic mild liver disease may take NSAIDs, they should be avoided in all patients
with cirrhosis, due to the risk of precipitating hepatorenal syndrome. Pregabalin and gabapentin are not
metabolized by the liver and can be quite helpful. Acetaminophen, while toxic in high doses, can be used
safely in dosages of 2–3 g/day. Tramadol is also safe in patients with cirrhosis.

27
Ref: Chandok N, Watt KD: Pain management in the cirrhotic patient: The clinical challenge. Mayo Clin Proc
2010;85(5):451-458.

Item 82

ANSWER: B

Coral-red fluorescence on Wood’s light examination is typical of infection with Corynebacterium


minutissimum, a condition known as erythrasma. This organism commonly complicates intertrigo, often
in the groin or interdigital spaces. Erythromycin is the most effective treatment for this bacterial infection.

Ref: Janniger CK, Schwartz RA, Szepietowski JC, Reich A: Intertrigo and common secondary skin infections. Am Fam
Physician 2005;72(5):833-838. 2) Binic I, Jankovic A: Coral-red fluorescence. N Engl J Med 2011;364(13):e25.

Item 83

ANSWER: A

The U.S. Preventive Services Task Force (USPSTF) and the Centers for Disease Control and Prevention
(CDC) recommend that all adults age 65 years and under be screened for HIV regardless of risk factors.
The USPSTF does not recommend routine screening for ovarian cancer with a bimanual examination,
transvaginal ultrasonography, or CA-125 testing. The USPSTF recommends that women age 65 and older
be screened for osteoporosis with a DXA scan. Women younger than age 65 should be screened only if
their risk of fracture is equal to or greater than a 65-year-old white female with no additional risk factors.
The USPSTF recommends that women age 50–75 be screened for colorectal cancer with colonoscopy
every 10 years OR with flexible sigmoidoscopy every 5 years plus fecal occult blood testing (FOBT) every
3 years OR with FOBT annually.

Ref: Riley M, Dobson M, Jones E, Kirst N: Health maintenance in women. Am Fam Physician 2013;87(1):30-37.

Item 84

ANSWER: A

Acute stress disorder (ASD) is a condition that develops within 4 weeks of a traumatic experience. The
individual with ASD suffers dissociative symptoms including amnesia, a reduction in awareness of his or
her surroundings, numbness, and detachment. Patients who suffer from ASD are at higher risk for
developing PTSD, but appropriate treatment can reduce this risk. Cognitive-behavioral management
(muscle relaxation techniques, gradual introduction to avoided situations, restructuring beliefs about the
trauma) aids in the treatment of ASD (SOR A). Cognitive-behavioral therapy has been found to be more
effective than supportive counseling.

Although pharmacologic management can be helpful, the evidence does not show it to be broadly
beneficial. Imipramine, fluoxetine, risperidone, and propranolol have all been used for ASD, but are not
recommended for routine use (SOR C).

Critical incident stress debriefing is a group activity usually provided shortly after a traumatic event.
During the sessions, patients are invited to share their feelings and education is provided on coping with
the trauma. There has been no evidence of a positive effect of stress debriefing and it may actually be
harmful by interfering with the natural course of recovery (SOR C).

28
Ref: Kavan MG, Elsasser GN, Barone EJ: The physician’s role in managing acute stress disorder. Am Fam Physician
2012;86(7):643-649.

Item 85

ANSWER: B

The Centers for Disease Control and Prevention (CDC) recommends treatment for persons at higher risk
for complications from influenza. Pregnant women and postpartum women within 2 weeks of delivery are
considered to be at higher risk. Some data has shown that early antiviral treatment may shorten the duration
of symptoms and reduce the risk of complications, especially in these higher risk populations.

The CDC recommends against using adamantines due to increased resistance of influenza viruses. Other
antivirals such as acyclovir and famciclovir are not active against influenza A and B. The recommended
treatment of influenza in pregnancy is one of the neuraminidase inhibitors.

Ref: Fiore AE, Fry A, Shay D, et al: Antiviral agents for the treatment and chemoprophylaxis of influenza—Recommendations
of the Advisory Committee on Immunization Practices (ACIP). MMWR Recomm Rep 2011;60(1):1-24. 2) Influenza
antiviral medications: Summary for clinicians. Centers for Disease Control and Prevention, 2013.

Item 86

ANSWER: D

Patients with bone pain caused by Paget’s disease usually describe the pain as continuous. Unlike
osteoarthritis, the bone pain of Paget’s disease usually increases with rest, when the limbs are warmed,
and at night. A variety of deformities may occur, including bowing of the tibia, and alkaline phosphatase
is elevated. The case presented is not typical for osteoarthritis or osteoporosis, and the patient does not
have a fever, elevated WBC count, or other findings suggestive of osteomyelitis.

Seronegative spondyloarthritis (or spondyloarthropathy) is a family of inflammatory rheumatic diseases


that cause arthritis. The most common is ankylosing spondylitis, which affects mainly the spine. Others
include axial spondyloarthritis, which affects mainly the spine and pelvic joints; peripheral
spondyloarthritis, affecting mostly the arms and legs; reactive arthritis (formerly known as Reiter’s
syndrome); psoriatic arthritis; and enteropathic arthritis/spondylitis associated with inflammatory bowel
diseases (ulcerative colitis and Crohn’s disease). The main symptom in most patients with spondyloarthritis
is low back pain, which is most common in axial spondyloarthritis. In a minority of patients, the major
symptom is pain and swelling in the arms and legs. This type is known as peripheral spondyloarthritis.

Ref: Schneider D, Hofmann MT, Peterson JA: Diagnosis and treatment of Paget’s disease of bone. Am Fam Physician
2002;65(10):2069-2072. 2) Whyte MP: Paget’s disease of bone. N Engl J Med 2006;355(6):593-600. 3) Reveille JD:
Spondyloarthritis (spondyloarthropathy). American College of Rheumatology patient fact sheet, 2012. 4) Longo DL, Fauci
AS, Kasper DL, et al (eds): Harrison’s Principles of Internal Medicine, ed 18. McGraw-Hill, 2012, pp 3136-3139.

Item 87

ANSWER: D

The National Health and Nutrition Examination Survey (NHANES) found that 1.4% of adults over 40 have
an ankle-brachial index (ABI) >1.4; this group accounts for approximately 20% of all adults with
peripheral artery disease. An ABI >1.4 indicates noncompressible arteries (calcified vessels). In patients
with arterial calcification, such as diabetic patients, more reliable information is often obtained by using
toe pressures to calculate a toe-brachial index and from pulse volume recordings.

29
Vascular imaging should be used to confirm peripheral vascular disease. MR or CT arteriography, duplex
scanning, and hemodynamic localization are noninvasive methods for lesion localization and may be
helpful when symptoms or findings do not correlate with the ABI. Contrast arteriography is used for
definitive localization before intervention.

Ref: Resnick HE, Foster GL: Prevalence of elevated ankle-brachial index in the United States 1999 to 2002. Am J Med
2005;118(6):676-679. 2) Sontheimer DL: Peripheral vascular disease: Diagnosis and treatment. Am Fam Physician
2006;73(11):1971-1976. 3) Met R, Bipat S, Legemate DA, et al: Diagnostic performance of computed tomography
angiography in peripheral arterial disease: A systematic review and meta-analysis. JAMA 2009;301(4):415-424. 4) Hauk
L: ACCF/AHA update peripheral artery disease management guideline. Am Fam Physician 2012;85(10):1000-1001.

Item 88

ANSWER: A

The inhalation of asbestos fibers may lead to a number of respiratory diseases, including lung cancer,
asbestosis, pleural plaques, benign pleural effusion, and malignant mesothelioma. High-risk populations
for asbestos exposure include individuals who worked in construction trades or as boilermakers, shipyard
workers, or railroad workers, as well as U.S. Navy veterans. The occupational history helps to guide
clinical suspicion in these high-risk populations. This patient is a retired U.S. Navy veteran who spent his
entire career in ship maintenance and repair.

The patient history is not consistent with berylliosis, silicosis, or uranium exposure. Berylliosis is an
occupational disease related to mining and manufacturing. Silicosis is seen in sandblasters, miners, persons
who have worked with abrasives, and several other occupations. Uranium exposure occurs after nuclear
reactor leaks or blasts. Uranium compounds are also used in photography and as dyes or fixatures. The
chemical toxicity involves nonmalignant damage to alveolar cells. Iron oxide exposure is not known to be
related to lung disease.

Ref: O’Reilly KM, Mclaughlin AM, Beckett WS, Sime PJ: Asbestos-related lung disease. Am Fam Physician
2007;75(5):683-688. 2) State of Lung Disease in Diverse Communities, 2010. American Lung Association, 2010, pp 63-69.
3) Schraufnagel DE (ed): Breathing in America: Diseases, Progress, and Hope. American Thoracic Society, 2010, pp
131-144. 4) The National Institute for Occupational Safety and Health (NIOSH): Work-Related Lung Disease Surveillance
System (eWoRLD). Centers for Disease Control and Prevention, 2013.

Item 89

ANSWER: C

A lace-up ankle support reduces pain and recovery time after an ankle sprain (SOR B). The Ottawa Rules
state that radiography is required only if there is pain in the malleolar or midfoot zone and either bony
tenderness over an area of potential fracture (i.e., distal fibula or tibia, lateral or medial malleolus, base
of the fifth metatarsal, or navicular bone) or an inability to bear weight immediately after the injury and
when evaluated by a physician. This patient did not have those findings, and therefore would not need a
radiograph (SOR A). A cast is not necessary for an ankle sprain. An elastic compression wrap alone is not
as effective as a lace-up support. Taping of the ankle for future sports participation can reduce the risk of
ankle sprains during sports, but would not be appropriate for an acute injury.

Ref: Tiemstra JD: Update on acute ankle sprains. Am Fam Physician 2012;85(12):1170-1176.

30
Item 90

ANSWER: A

Acute mountain sickness is common in people traveling to altitudes higher than 8200 ft. Symptoms include
headache and at least one of the following: nausea or vomiting, anorexia, dizziness or lightheadedness,
fatigue or weakness, and difficulty sleeping. Slow ascent is the most effective way to prevent acute
mountain sickness. Acetazolamide or dexamethasone can be used for both prevention and treatment. Ataxia
and altered mental status are signs of cerebral edema and occur with end-stage acute mountain sickness.
This can progress to coma and death and requires prompt treatment and descent. High-altitude pulmonary
edema can occur without acute mountain sickness. Alcohol consumption on the first day at altitude can
exacerbate acute mountain sickness.

Ref: Fiore DC, Hall S, Shoja P: Altitude illness: Risk factors, prevention, presentation, and treatment. Am Fam Physician
2010;82(9):1103-1110.

Item 91

ANSWER: D

The hallmark of acute mesenteric artery ischemia is severe abdominal pain that is out of proportion to
physical findings. This is a life-threatening event, which often follows a myocardial infarction when a
mural thrombus occludes a superior mesenteric artery. Patients rapidly become acidotic and hypotensive,
and experience a high mortality rate.

Alcoholic withdrawal syndrome has a much more insidious onset, with tremors, agitation, and anxiety
being the prominent features. Abdominal pain is not a common prominent symptom or finding. Although
pulmonary embolism is possible in the patient described, the major symptoms are dyspnea and chest pain,
and abdominal pain and tenderness would not be a typical finding. A normal serum lipase level should
point to a nonpancreatic origin for this patient’s problem. A perforated gastric ulcer would typically have
a more insidious onset and the abdominal examination would usually demonstrate marked focal tenderness.

Ref: Oldenburg WA, Lau LL, Rodenberg TJ, et al: Acute mesenteric ischemia. Arch Intern Med 2004;164(10):1054-1062. 2)
Marx JA (ed): Rosen’s Emergency Medicine: Concepts and Clinical Practice, ed 7. Mosby Elsevier, 2010, pp 1188-1192.

Item 92

ANSWER: A

Maximal heart rate with exercise generally decreases with age. A frequently used formula for predicting
maximal heart rate is 220 minus age, with a correction factor of 0.85 often applied for females, who have
a lower peak heart rate and a more gradual decline.

Myocardial collagen and mass both increase with age. The increase in collagen may play a role in
decreasing left ventricular compliance. The resting heart rate, like the maximal exercising heart rate,
decreases with normal aging. Tachycardia at rest may suggest a pathologic state.

Ref: Halter JB, Ouslander JG, Tinetti ME, et al (eds): Hazzard’s Geriatric Medicine and Gerontology, ed 6. McGraw-Hill
Medical, 2009, pp 883-895. 2) Fillit HM, Rockwood K, Woodhouse K (eds): Brocklehurst’s Textbook of Geriatric
Medicine and Gerontology, ed 7. Saunders Elsevier, 2010, pp 91-97.

31
Item 93

ANSWER: E

ACE inhibitors and $-blockers improve mortality in heart failure (HF). Digoxin and furosemide improve
symptoms and reduce hospitalizations in systolic HF, and furosemide may decrease mortality.
Spironolactone, an aldosterone antagonist, reduces all-cause mortality and improves ejection fractions in
systolic HF. Verapamil, due to its negative inotropic effect, is associated with worsening heart failure and
an increased risk of adverse cardiovascular events.

Ref: Jessup M, Abraham WT, Casey DE, et al: 2009 focused update: ACCF/AHA Guidelines for the Diagnosis and Management
of Heart Failure in Adults: A report of the American College of Cardiology Foundation/American Heart Association Task
Force on Practice Guidelines. Circulation 2009;119(14):1977-2016.

Item 94

ANSWER: A

The diagnostic criteria for depression are the same for children and adults, although the manner in which
these symptoms present may be different. Adolescents with depression are more likely to experience
anhedonia, boredom, hopelessness, hypersomnia, weight change, alcohol or drug use, and suicide
attempts. Psychotherapy should always be included as part of a treatment plan for depression in
adolescents.

Cognitive-behavioral therapy and interpersonal therapy are two modalities that have been proven effective
in the treatment of adolescent depression. Medications should be considered for more severe depression
or depression failing to respond to psychotherapy. A Cochrane review found that fluoxetine was the only
agent with consistent evidence of effectiveness in decreasing depressive symptoms in adolescents.
Consensus guidelines recommend fluoxetine, citalopram, or sertraline as first-line treatments for moderate
to severe depression in children and adolescents. Escitalopram is also licensed for the treatment of
depression in adolescents age 12 or older. All antidepressants have a boxed warning regarding an increased
risk of suicide; therefore, close monitoring is recommended to assess for suicidality and other adverse
effects, such as gastrointestinal effects, nervousness, headache, and restlessness. Tricyclic antidepressants
were previously used to treat depression in children, but studies have shown little to no benefit in
adolescents and children.

Ref: Clark MS, Jansen KL, Cloy JA: Treatment of childhood and adolescent depression. Am Fam Physician 2012;86(5):442-448.

Item 95

ANSWER: A

While all of the medications listed can be used to treat osteoporosis, only calcitonin-salmon is useful in the
management of pain associated with acute or chronic vertebral fractures. Calcitonin is an antiresorptive
agent that has been shown to decrease the risk of vertebral fractures, but it is not considered a first-line
treatment for osteoporosis because there are more effective agents. However, it does have modest analgesic
properties that make it useful in the treatment of the pain associated with vertebral fractures.

Ref: Sweet MG, Sweet JM, Jeremiah MP, Galazka SS: Diagnosis and treatment of osteoporosis. Am Fam Physician
2009;79(3):193-200.

32
Item 96

ANSWER: C

This patient most likely has pseudogynecomastia due to increases in subareolar fat secondary to his obesity.
This is based upon clinical findings of symmetric adipose tissue in the breast region bilaterally and a lack
of firm, palpable glandular tissue in the nipple and areolar region. In gynecomastia, there is palpable, firm
glandular tissue in a concentric mass around the nipple-areola complex. Hard, immobile masses, masses
associated with skin changes, nipple retraction, nipple discharge, or enlarged lymph nodes would suggest
possible malignancy. Fat necrosis would involve a history of breast region trauma and would generally be
asymmetric. Mastitis would cause clinical signs of infection.

Ref: Dickson G: Gynecomastia. Am Fam Physician 2012;85(7):716-722.

Item 97

ANSWER: E

Among the oral antiglycemic drugs, the sulfonylurea agents are the most likely to cause hypoglycemia,
and glyburide is more likely to cause hypoglycemia than glipizide. Glyburide should rarely be used in the
elderly.

Ref: American Diabetes Association: Standards of medical care in diabetes—2013. Diabetes Care 2013;36(Suppl 1):S45.

Item 98

ANSWER: E

This patient meets the criteria for hypoactive sexual desire disorder (HSDD). The incidence of this
condition is variable based on the age, life stage, and culture of the patient, but is estimated to be present
in about 5%–15% of the adult female population. This diagnosis includes two components: (1) recurrent
deficiency or absence of sexual desire or receptivity to sexual activity, and (2) distress about such a
deficiency. In menstruating women, oral estrogen and oral sildenafil have not been shown to be superior
to placebo. Cognitive-behavioral therapy has been shown to be helpful for other sexual dysfunctions, but
not with HSDD. Topical testosterone, in either patch or gel form, has shown consistent improvements in
arousal, desire, fantasy, orgasm, and overall satisfaction in cases of HSDD.

Ref: Parish SJ: From whence comes HSDD? J Fam Pract 2009;58(7 Suppl Hypoactive):S16-S21. 2) American College of
Obstetricians and Gynecologists Committee on Practice Bulletins–Gynecology: ACOG Practice Bulletin No. 119: Female
sexual dysfunction. Obstet Gynecol 2011;117(4):996-1007. 3) Simon JA. Opportunities for intervention in HSDD. J Fam
Pract. 2009;58(7 Suppl Hypoactive):S26-30. Review.

Item 99

ANSWER: C

The appropriate management of a thrombosed hemorrhoid presenting within 72 hours of the onset of
symptoms is elliptical excision of the hemorrhoid and overlying skin under local anesthesia, such as 0.5%
bupivacaine hydrochloride in 1:200,000 epinephrine, infiltrated slowly with a 27-gauge needle.

33
Incision and clot removal may provide inadequate drainage, resulting in rehemorrhage and clot
reaccumulation. Most thrombosed hemorrhoids contain multilocular clots that may not be accessible
through a simple incision. Rubber band ligation is an excellent technique for management of internal
hemorrhoids, and infrared coagulation is also used for this purpose. Banding an external hemorrhoid would
cause exquisite pain.

If the pain is already subsiding or more time has elapsed, and if there is no necrosis or ulceration,
measures such as sitz baths, bulk laxatives, stool softeners, and local analgesia may be helpful. Some local
anesthetics carry the risk of sensitization. Counseling to avoid precipitating factors such as prolonged
standing/sitting, constipation, and delay of defecation is also appropriate.

Ref: Mounsey AL, Henry SL: Clinical inquiries. Which treatment works best for hemorrhoids? J Fam Pract 2009;58(9):492-493.
2) Bope ET, Kellerman RD (eds): Conn’s Current Therapy 2013. Elsevier Saunders, 2013, pp 525-528.

Item 100

ANSWER: C

A Cochrane review of 15 studies involving 922 patients showed that antidepressants had a beneficial effect
on the symptoms of irritable bowel syndrome (IBS). Both SSRIs and tricyclic antidepressants have shown
benefit. Another Cochrane review of 12 randomized, controlled trials did not show any benefit from the
use of fiber in any type of IBS. Antibiotics have been shown to have some beneficial effects, but neomycin
is used only in constipation-predominant IBS. Lubiprostone is a selective C-2 chloride channel activator
and can be used for patients with chronic constipation. Alosetron is a 5-hydroxytryptamine 3 antagonist
and is FDA approved to treat severe diarrhea-predominant IBS only in women who have not improved with
conventional therapy. Alosetron is associated with uncommon but serious adverse events (ischemic colitis,
constipation, death) and its use is restricted in the United States. Other potentially beneficial therapies for
IBS include peppermint oil, psychological treatments, exercise, and probiotics.

Ref: Wilkins T, Pepiton C, Alex B, Schade RR: Diagnosis and management of IBS in adults. Am Fam Physician
2012;86(5):419-426.

Item 101

ANSWER: E

This patient is experiencing the Jarisch-Herxheimer reaction—an acute, transient, febrile reaction that
occurs within the first few hours after treatment for syphilis. The condition peaks at 6–8 hours and
disappears within 12–24 hours after therapy. The temperature elevation is usually low grade, and there is
often associated myalgia, headache, and malaise. It is usually of no clinical significance and may be treated
with salicylates in most cases. The pathogenesis of the reaction is unclear, but it may be due to liberation
of antigens from the spirochetes.

Ref: Goldman L, Schafer AI (eds): Goldman’s Cecil Medicine, ed 24. Elsevier Saunders, 2011, pp e162-e163. 2) Mattei PL,
Beachkofsky TM, Gilson RT, Wisco OJ: Syphilis: A reemerging infection. Am Fam Physician 2012;86(5):433-440.

34
Item 102

ANSWER: E

Testing for diabetes mellitus should be considered in all asymptomatic adults who have a BMI ³25 kg/m2
and have one or more additional risk factors such as physical inactivity, a first degree relative with
diabetes, a high-risk ethnicity, hypertension, hyperlipidemia, or polycystic ovary syndrome. In
asymptomatic patients with no risk factors, screening should begin at age 45.

Ref: Ismail-Beigi F: Glycemic management of type 2 diabetes mellitus. N Engl J Med 2012;366(14):1319-1327. 2) American
Diabetes Association: Standards of medical care in diabetes—2013. Diabetes Care 2013;36(Suppl 1):S13.

Item 103

ANSWER: D

Myalgias and noninflammatory arthralgias are more likely with aromatase inhibitors. Venous
thromboembolism rarely occurs with these drugs. Endometrial cancer may occur with long-term use of
tamoxifen.

Ref: Khan QJ, O’Dea AP, Sharma P: Musculoskeletal adverse events associated with adjuvant aromatase inhibitors. J Oncol
2010:65438.

Item 104

ANSWER: D

Myelodysplastic syndrome is a hematologic malignancy with a predisposition to leukemic transformation.


It can present with findings of anemia, thrombocytopenia, neutropenia, or any combination of these.
Anemia occurs in 80%–85% of patients and is typically macrocytic.

Ref: Foran JM, Shammo JM: Clinical presentation, diagnosis, and prognosis of myelodysplastic syndromes. Am J Med
2012;125(7 Suppl):S6-S13.

Item 105

ANSWER: B

Education of asthmatic patients is critically important in their follow-up care. This includes informing
patients about the severity of their asthma in addition to instruction about appropriate treatment modalities.
The National Heart, Lung, and Blood Institute’s National Asthma Education and Prevention Program uses
the following definitions for asthma severity:

Intermittent: Symptoms less than or equal to twice weekly, nighttime awakenings £2 times/month,
short-acting $-agonist usage £2 days/week, no interference with daily activities, and normal FEV1 and
FEV1/FVC ratio at baseline

Mild Persistent: Symptoms >2 days/week but not daily, nighttime awakenings 3–4 times/month,
short-acting $-agonist usage >2 days/week but not more than once daily, minor limitation to daily
activities, FEV1 ³80% predicted, and normal FEV1/FVC ratio

35
Moderate Persistent: Daily symptoms, nighttime awakenings greater than once weekly but not nightly,
daily use of a short-acting $-agonist, some limitation to daily activity, FEV1 >60% but <80% of
predicted, and FEV1/FVC ratio reduced by 5%

Severe Persistent: Symptoms throughout the day, nighttime awakenings nightly, short-acting $-agonist
usage several times daily, extremely limited daily activities, FEV1 <60% of predicted, and FEV1/FVC
ratio reduced by >5%

Status asthmaticus is a medical emergency and requires emergent treatment in a hospital setting.

Ref: Expert Panel Report 3: Guidelines for the Diagnosis and Management of Asthma. National Asthma Education and
Prevention Program, NIH pub no 07-4051, 2007. 2) Pollart SM, Elward KS: Overview of changes to asthma guidelines:
Diagnosis and screening. Am Fam Physician 2009;79(9):761-767. 3) Pollart SM, Compton RM, Elward KS: Management
of acute asthma exacerbations. Am Fam Physicians 2011;84(1):40-47.

Item 106

ANSWER: B

The treatment of depression in pregnancy is determined by the severity of the symptoms and any past
history of treatment response. For women who have a new onset of mild or moderate depression, it may
be best to start with nonpharmacologic treatments such as supportive psychotherapy or cognitive-behavioral
therapy. These interventions may improve the depression enough that the patient will not need medications.
However, in situations where pharmacologic treatment is clearly indicated, SSRIs are thought to have the
best safety profile. Fluoxetine, sertraline, and citalopram have extensive data to support their safety in
pregnancy and should be considered first line. Paroxetine is the one SSRI that is thought to carry an
increased risk of congenital malformations with first-trimester exposure and should be avoided.

Tricyclic antidepressants are class D in pregnancy. SNRIs do not have as much safety data as SSRIs to
support their use in pregnancy and would be considered a second-line choice. MAOIs are known teratogens
and should be avoided in pregnancy. Stimulants are not first-line agents and should be avoided in
pregnancy.

Ref: Cohen LS, Wang B, Nonacs R, et al: Treatment of mood disorders during pregnancy and postpartum. Psychiatr Clin North
Am 2010;33(2):273-293. 2) Stewart DE: Depression during pregnancy. N Engl J Med 2011;365:1605-1611.

Item 107

ANSWER: D

The concept of Accountable Care Organizations (ACOs) was introduced in 2009 by the Centers for
Medicare and Medicaid Services (CMS) to encourage doctors, hospitals, and other health care providers
to work together to deliver high-quality care and spend health care dollars more wisely. The ACO concept,
together with a shared savings program, has had difficulty penetrating smaller practices and more rural
regions of the country. There is also concern that ACOs may allow larger systems to work as a monopoly
as an unintended consequence. For this reason the Department of Justice and the Federal Trade
Commission are monitoring these organizations as they develop.

36
This new strategy of shared savings through coordinated health care is an alternative to the Sustainable
Growth Rate (SGR) formula that CMS had previously hoped would contain health-care costs. The SGR
was created to prevent Medicare rates from growing faster than the GDP. The high-profile topic of
requiring individuals to carry health insurance has also been part of governmental reform initiatives but
is not directly related to ACOs.

Ref: Dove JT, Weaver WD, Lewin J: Health care delivery system reform: Accountable care organizations. J Am Coll Cardiol
2009;54(11):985-988. 2) Accountable Care Organizations. Centers for Medicare and Medicaid Services, 2013.

Item 108

ANSWER: E

Blood transfusions should be administered to patients with upper gastrointestinal bleeding who have a
hemoglobin level £7.0 g/dL (SOR C). According to the Rockall risk scoring system, this patient’s
mortality risk from gastrointestinal bleeding is low, based on the following: age <60, systolic blood
pressure ³100 mm Hg, heart rate <100 beats/min, no shock, and no major comorbidities. The
Mallory-Weiss tear adds no points to his total score, and the only stigmata of recent hemorrhage is a dark
spot in an otherwise clean ulcer base, which also adds no points. His only scored finding is the presence
of the ulcer, which adds a single point to his score.

Patients with low-risk peptic ulcer bleeding based on clinical and endoscopic criteria can be discharged
from the hospital on the same day as endoscopy (SOR C). Routine second-look endoscopy is not
recommended in patients with upper gastrointestinal bleeding who are not considered to be at high risk for
rebleeding (SOR C). Arteriography with embolization is indicated only in patients with persistent bleeding.

Ref: Wilkins T, Khan N, Nabh A, Schade RR: Diagnosis and management of upper gastrointestinal bleeding. Am Fam Physician
2012;85(5):469-476.

Item 109

ANSWER: A

Plain radiography should be the initial imaging modality to diagnose stress fractures (SOR C). One
algorithm advocates radiography 2 weeks after the onset of symptoms (if symptoms persist), with repeat
radiography the following week before performing more advanced imaging. An expert panel of the
American College of Radiology recommends that MRI be considered next if plain radiography is negative.

Ref: Patel DS, Roth M, Kapil N: Stress fractures: Diagnosis, treatment , and prevention. Am Fam Physician 2011;83(1):39-46.

Item 110

ANSWER: B

In general, when young children are found to be behind schedule in receiving recommended
immunizations, catch-up immunization is important. However, the rotavirus series should not be started
past 15 weeks of age, or continued past 8 months of age. This child should have received hepatitis B
vaccine at 6 months of age, and should be given a catch-up dose. The MMR and varicella vaccines are
recommended at the 12-month visit.

Ref: Loehr J: ACIP releases 2013 immunization schedules. Am Fam Physician 2013;87(3):204-210.

37
Item 111

ANSWER: D

In patients with type 2 diabetes mellitus, the single most important predictor of severe hypoglycemia is a
previous history of severe hypoglycemia that required external assistance. It is thought that hypoglycemia
reduces the body’s protective responses (glucagon and epinephrine) to subsequent episodes of
hypoglycemia (SOR C). Increased blood glucose level goals and increased self-monitoring of blood glucose
are the most important measures for avoiding further episodes.

Less significant risk factors for hypoglycemia include advanced age, use of five or more medications,
African-American ethnicity, and recent hospital discharge (SOR B). Diabetic autonomic neuropathy may
be a risk factor but this has not been definitively established.

Ref: McCall AL: Insulin therapy and hypoglycemia. Endocrinol Metab Clin North Am 2012;41(1):57-87.

Item 112

ANSWER: A

A persistent occiput posterior position is associated with a higher risk of cesarean delivery and assisted
vaginal delivery, and a lower chance of spontaneous vaginal delivery. Assisted vaginal deliveries are
associated with a higher rate of third- and fourth-degree perineal lacerations and postpartum hemorrhage.

Ref: Cunningham FG, Leveno KJ, Bloom SL, et al (eds): Williams Obstetrics, ed 23. McGraw-Hill Medical, 2010, p 480.

Item 113

ANSWER: E

The primary care physician can assess decision-making capacity based on the patient’s ability to reason,
communicate, understand the proposed treatment, and grasp the consequences of accepting or declining
the suggested treatment.

Formal mental status testing and determination of capacity are different functions. Accurate mental status
testing is helpful for assessing the capacity to make decisions, but there is not a specific score that
determines capacity. However, there is a certain level of cognitive impairment where a patient simply lacks
any ability to receive and process health information. At somewhat higher levels of cognition a patient
might lack specific mental abilities, but still be able to satisfy the requirements for making treatment
decisions. A recent meta-analysis showed that Mini-Mental State Examination (MMSE) scores below 20
increase the likelihood of incapacity (LR 6.3), scores of 20–24 have no effect (LR 0.87), and scores
greater than 24 significantly lower the likelihood of incapacity (LR 0.17).

Determination of capacity does not require legal intervention or psychiatric expertise. While there is no
specific test for decision-making capacity, there are instruments available to assist physicians with making
these assessments. The best validated of these is the Aid to Capacity Evaluation (ACE), which is free and
available online (http://www.jointcentreforbioethics.ca/tools/documents/ace.pdf). It can be administered
in 10–20 minutes.

38
Competence is a legal term in this situation. Decisions regarding competence are judicial determinations
of the capacity to make nonmedical decisions such as financial decisions. Under the law, adults are
presumed to be competent until a specific action of the appropriate court declares otherwise.

Ref: Sessums LL, Zembrzuska H, Jackson JL: Does this patient have medical decision-making capacity? JAMA
2011;306(4):420-427.

Item 114

ANSWER: C

This patient has a history and physical findings that are consistent with a rotator cuff tear. Most commonly
the mechanism of injury in an acute rotator cuff tear is forced abduction of the arm with significant
resistance. Often this will occur when a person attempts to break a fall with an outstretched hand. There
is usually a sudden sensation of tearing pain in the shoulder. Pain and muscle spasm will limit shoulder
motion. Patients with a large tear cannot initiate shoulder abduction and will have a discrepancy between
active and passive motion. Patients with significant tears will also have a positive drop arm test. This test
is performed by passively abducting the arm to 90° and asking the patient to hold the arm in that position
while the examiner applies pressure on the distal forearm or wrist. The test is positive if the pressure
causes the arm to drop suddenly.

Acute tears are generally managed with a splint and orthopedic referral for surgical repair. Chronic tears
may be managed with shoulder rehabilitation but may ultimately require surgical repair as well.

Bicipital tendinitis is not generally caused by acute trauma, but by irritation and microtrauma due to
repetitive elevation or abduction of the shoulder, causing an inflammatory reaction in the synovial sheath.
Patients generally present with a complaint of pain in the anterior shoulder that radiates into the upper arm.
It is more painful with activity and is worse at night. Abduction and external rotation of the arm
exacerbates the pain. On examination there should be point tenderness in the bicipital groove. Active range
of motion will be limited by pain but passive range of motion will be intact. There should not be any
weakness.

Acromioclavicular separation is usually caused by a fall or a direct blow to the point of the shoulder with
the shoulder abducted. The pain associated with this injury is over the acromioclavicular joint margin and
there may be swelling. Depending on the severity of the injury there may be full range of motion but it
may be restricted due to pain. There should not be any weakness associated with this injury.

A fracture of the humeral head generally occurs with a fall on an outstretched arm or direct blow to the
lateral side of the arm. Generally there is pain or bruising over the fracture site. Movement will be
restricted by pain, but there should not be any weakness.

A tear of the labrum can occur with acute trauma or from repetitive shoulder motion. Acute trauma may
occur from a dislocation of the shoulder, falling on an outstretched arm, or direct blows to the shoulder.
Generally, people with a tear of the labrum will have increased pain with overhead activity, popping or
grinding, loss of strength, and trouble locating a specific point of pain.

Ref: House J, Mooradian A: Evaluation and management of shoulder pain in primary care clinics. South Med J
2010;103(11):1129-1135. 2) Marx JA (ed): Rosen’s Emergency Medicine: Concepts and Clinical Practice, ed 7. Mosby
Elsevier, 2010, pp 586-587.

39
Item 115

ANSWER: E

Dermatitis herpetiformis is an immune-mediated vesicular disease that usually occurs in the young to
middle-aged. The skin lesions are extremely pruritic grouped vesicles and erosions located on the scalp,
posterior neck, and extensor surfaces of the elbows, knees, and buttocks. Most patients have a subclinical
gluten-sensitive enteropathy that is reversible with a gluten-free diet, which can sometimes control the skin
disease as well.

Erythema multiforme is an acute blistering eruption that occurs in all age groups. Porphyria cutanea tarda
patients develop erosions and bullae on sun-exposed skin. Herpes gestationis is a rare autoimmune
dermatosis of pregnancy, and bullous pemphigoid is an autoimmune blistering disorder seen in the elderly.
Tense blisters and urticarial plaques occur on the flexor surfaces of the arms, legs, axillae, groin, and
abdomen.

Ref: Goldman L, Schafer AI (eds): Goldman’s Cecil Medicine, ed 24. Elsevier Saunders, 2011, pp 2525-2529.

Item 116

ANSWER: B

A dorsiflexion injury will typically cause a scaphoid fracture in a young adult, resulting in tenderness to
palpation over the anatomic snuffbox. A plain posterior-anterior wrist radiograph is often normal.
However, a special view with the wrist prone in ulnar deviation elongates the scaphoid, often
demonstrating subtle fractures. Hook of the hamate fractures cause tenderness at the proximal hypothenar
area 1 cm distal to the flexion crease of the wrist. When this fracture is suspected, carpal tunnel and
supinated oblique view radiographs should be obtained. A scapholunate dislocation can be identified with
a “clenched-fist” view and a supinated view with the wrist in ulnar deviation.

Ref: Forman TA, Forman SK, Rose NE: A clinical approach to diagnosing wrist pain. Am Fam Physician
2005;72(9):1753-1758. 2) DeLee JC, Drez D Jr, Miller MD (eds): DeLee & Drez's Orthopaedic Sports Medicine:
Principles and Practice, ed 3. Elsevier Saunders, 2010, pp 1335-1336.

Item 117

ANSWER: B

This patient has a mallet fracture. These fractures are caused by an axial load to the tip of an extended
finger that causes forced flexion at the distal interphalangeal (DIP) joint. This leads to a fracture at the
dorsal surface of the proximal distal phalanx where the terminal finger extensor mechanism inserts. The
most appropriate treatment of a mallet fracture is to splint the DIP joint in extension for 8 weeks. The joint
should remain in full extension for optimal healing. Any flexion of the finger may affect healing and extend
the treatment time.

Surgical management has been recommended for fractures that involve more than 30% of the joint space,
but a small study showed there was no difference in outcomes compared to treatment with extension
splints. Buddy taping would not offer enough support to maintain the finger in extension at all times.

Ref: Borchers JR, Best TM: Common finger fractures and dislocations. Am Fam Physician 2012;85(8):805-810.

40
Item 118

ANSWER: E

In women 30–65 years old, screening for cervical cancer with cervical cytology and HPV testing is
recommended every 5 years. An alternative screening recommendation is to perform cervical cytology
only, at 3-year intervals. A population study of 331,818 women demonstrated a 0.016% risk of cancer in
the 5 years after having a negative result on both cervical cytology and an HPV test.

Ref: American College of Obstetricians and Gynecologists: Screening for cervical cancer. ACOG Practice Bulletin, no 131,
2012. 2) US Preventive Services Task Force: Screening for cervical cancer. April 2012.

Item 119

ANSWER: C

The most likely diagnosis for this patient’s knee pain is osteoarthritis. While he is likely to have disease
in both knees, it is common for patients to have unilateral symptoms, especially early on. Although
osteoarthritis is mainly a clinical diagnosis, plain radiography is the diagnostic study of choice if there is
concern about other diagnostic possibilities. Narrowing of the medial compartment of the knee joint is
typically the first radiographic finding; osteophytes are also commonly seen on plain films. In the scenario
presented here, there is no need for laboratory testing at this time.

Ref: Sinusas K: Osteoarthritis: Diagnosis and treatment. Am Fam Physician 2012;85(1):49-56.

Item 120

ANSWER: B

There is good evidence that psychostimulants reduce symptoms of depression within days, making
methylphenidate a good choice for this patient (SOR B). Electroconvulsive therapy is contraindicated due
to her brain lesions. Mirtazapine, fluoxetine, and nortriptyline all take at least 3–4 weeks to have any
antidepressant effects, and would not be appropriate given the patient’s life expectancy (SOR B).

Ref: Widera EW, Block JD: Managing grief and depression at the end of life. Am Fam Physician 2012;86(3):259-264.

Item 121

ANSWER: A

This patient has Bell’s palsy. Only corticosteroids have been shown to improve the outcome. Antiviral
agents have little value in the treatment of Bell’s palsy. Thrombolytic therapy may be useful for a patient
with central facial nerve weakness if it is due to a vascular event (level of evidence 3; SOR A).

Ref: Holland J, Bernstein J: Bell palsy. Am Fam Physician 2011;84(8):947-948.

41
Item 122

ANSWER: B

Marfan syndrome is an autosomal dominant disease manifested by skeletal, ophthalmologic, and


cardiovascular abnormalities. Men taller than 72 inches and women taller than 70 inches who have two
or more manifestations of Marfan syndrome should be screened by echocardiography for associated cardiac
abnormalities. These signs and symptoms include cardiac murmurs or clicks, kyphoscoliosis, anterior
thoracic deformity, arm span greater than height, upper to lower body ratio more than 1 standard deviation
below the mean, myopia, and an ectopic lens.

Athletes with a family history of Marfan syndrome should also be screened, whether they have
manifestations themselves or not. Patients with Marfan syndrome who have echocardiographic evidence
of aortic abnormalities should be placed on $-blockers and monitored with echocardiography every 6
months.

Ref: Bader RS, Goldberg L, Sahn DJ: Risk of sudden cardiac death in young athletes: Which screening strategies are
appropriate? Pediatr Clin North Am 2004;51(5):1421-1441. 2) Kliegman RM, Stanton BF, Geme JW III, et al (eds):
Nelson Textbook of Pediatrics, ed 19. Elsevier Saunders, 2011, pp 2440-2446.

Item 123

ANSWER: B

This patient most likely has an anaerobic bacterial infection. Penicillin was used to treat these infections
in the past, but because of the emergence of $-lactamase–producing organisms, clindamycin is now the
drug of choice. Clindamycin has broader coverage against both pulmonary anaerobes and facultative
aerobes such as Staphylococcus aureus and Klebsiella, which are often seen with lung abscesses.
Metronidazole has anaerobic coverage, but not for the anaerobic species often involved in pulmonary
infections, and is therefore associated with a high failure rate when used to treat lung abscesses.
Doxycycline does not cover anaerobes. Trimethoprim/sulfamethoxazole is also not considered a good
anaerobic antibiotic.

Ref: Longo DL, Fauci AS, Kasper DL, et al (eds): Harrison’s Principles of Internal Medicine, ed 18. McGraw-Hill, 2012, pp
2142-2147.

Item 124

ANSWER: A

The diagnosis of delirium is based entirely on the history and physical examination. No laboratory tests,
imaging studies, or other tests are more accurate than clinical assessment.

Ref: Marcantonio ER: In the clinic. Delirium. Ann Intern Med 2011;154(11):ITC6 1-14.

42
Item 125

ANSWER: C

While it is certainly appropriate for the nurse practitioner or physician who ordered the test to notify the
patient of mammography results, the facility performing the test is legally responsible. This is specified
by the federal Mammography Quality Standards Act, first passed by Congress in 1992.

Ref: Mammography Quality Standards Act Regulations. US Food and Drug Administration, 2009, sec 900.12(c)(2).

Item 126

ANSWER: E

Plantar fasciitis is characterized by pain that is worse with the first few steps in the morning or after a
prolonged rest. NSAIDs may help with the discomfort, but prompt relief of the pain by any modality is
not common. The pain is typically in the medial heel. While 50% of people with plantar fasciitis have heel
spurs on radiographs, this finding is not causative or diagnostic. The diagnosis is made clinically.

Ref: Tu P, Bytomski JR: Diagnosis of heel pain. Am Fam Physician 2011;84(8):909-916.

Item 127

ANSWER: A

Heat exhaustion and heatstroke are both on the continuum of heat-related illness. Heatstroke is a much
more severe condition than heat exhaustion. Evidence of central nervous system dysfunction is evidence
of heatstroke rather than heat exhaustion, even if other symptoms are not severe and point to heat
exhaustion. Heatstroke is a medical emergency.

Ref: Glazer JL: Management of heatstroke and heat exhaustion. Am Fam Physician 2005;71(11):2133-2140, 2141-2142. 2)
Marx JA (ed): Rosen’s Emergency Medicine: Concepts and Clinical Practice, ed 7. Mosby Elsevier, 2010, pp 1887-1890.

Item 128

ANSWER: E

Bupropion can lower the seizure threshold and should not be used in patients who have a history of a
seizure disorder or who drink heavily. A history of the other medical conditions listed does not
contraindicate the use of bupropion.

Ref: Fiore MC, Baker TB: Treating smokers in the health care setting. N Engl J Med 2011;365(13):1222-1231.

43
Item 129

ANSWER: C

This case illustrates the classic history and physical findings of iliotibial band syndrome. Pain occurs most
frequently at the site where the tendon crosses over the lateral femoral epicondyle. With osteoarthritis or
a meniscal tear there would be pain in the joint space with palpation. Osgood-Schlatter disease is more
common in younger adolescents and is characterized by tenderness of the tibial tubercle at the distal
insertion of the patellar ligament. Pes anserine bursitis is characterized by pain in the medial knee distal
to the joint space, at the conjoined tendon of the sartorius, gracilis, and semitendinosus.

Ref: Khaund R, Flynn SH: Iliotibial band syndrome: A common source of knee pain. Am Fam Physician 2005;71(8):1545-1550.
2) Longo DL, Fauci AS, Kasper DL, et al (eds): Harrison’s Principles of Internal Medicine, ed 18. McGraw-Hill, 2012,
pp 2860-2863.

Item 130

ANSWER: C

Metformin is contraindicated in males with creatinine levels >1.5 mg/dL (SOR C) and should be stopped
in this patient. The recent episode of diarrhea may have exacerbated renal disease associated with
long-standing hypertension and diabetes mellitus. Dose modifications for sitagliptin and simvastatin may
be needed with renal dysfunction, but these medications would not have to be stopped. Carvedilol and
clopidogrel do not require a change in dosage with renal dysfunction because their metabolism is largely
hepatic.

Ref: Ripsin CM, Kang H, Urban RJ: Management of blood glucose in type 2 diabetes mellitus. Am Fam Physician
2009;79(1):29-36, 42.

Item 131

ANSWER: D

Constipation is a very common side effect of opioids that does not resolve with time, unlike many other
adverse effects. Constipation is easier to prevent than to treat, so it is important to start an appropriate
bowel regimen with the initiation of opioid therapy. Fiber supplements and detergents (such as docusate)
are inadequate for the prevention of opioid-induced constipation. Metoclopramide is used for nausea and
increases gastric motility, but is not indicated in the treatment of constipation. Polyethylene glycol,
lactulose, magnesium hydroxide, and senna with docusate are all appropriate in this situation.

Ref: Clary PL, Lawson P: Pharmacologic pearls for end-of-life care. Am Fam Physician 2009;79(12):1059-1065.

44
Item 132

ANSWER: C

Patients diagnosed with pharyngeal gonorrhea may be asymptomatic, or they may have oropharyngeal
erythema and an exudate, along with cervical lymphadenopathy. Gonorrheal pharyngitis can coexist with
cervical gonorrhea. First-line treatment for gonorrheal pharyngitis is ceftriaxone, 250 mg intramuscularly
once (SOR C). Although chlamydial pharyngitis is uncommon, patients with pharyngitis due to gonorrhea
are often coinfected with genital Chlamydia and should therefore be treated empirically for chlamydial
infection. There is no need to perform a pelvic examination or order additional testing. All patients who
are diagnosed with gonorrhea should be retested at 3–6 months due to a high rate of reinfection (SOR C).
Partner treatment is highly recommended.

Ref: Mayor MT, Roett MA, Uduhiri KA: Diagnosis and management of gonococcal infections. Am Fam Physician
2012;86(10):931-938.

Item 133

ANSWER: E

Severe hypophosphatemia is a medical emergency. In poorly nourished patients, refeeding syndrome can
occur. Symptoms usually develop by the second or third day of improved nutrition, and are often
multisystemic. Findings may include weakness, confusion, dysrhythmias, respiratory failure, heart failure,
hypotension, ileus, metabolic acidosis, seizures, coma, and sudden death. This constellation of problems
results from decreased insulin secretion as stores of intracellular phosphate become depleted. Providing
carbohydrates through intravenous fluids or refeeding increases insulin secretion, which stimulates cells
to take up phosphate, causing severe hypophosphatemia. In this setting cells are unable to produce enough
2,3 diphosphoglycerate and adenosine triphosphate to meet metabolic demands.

While hypoglycemia is also a medical emergency, this patient’s glucose level is not low enough to cause
these symptoms. Similarly, renal failure of some type is present, as is an elevated creatine kinase
suggesting rhabdomyolysis; however, neither of these problems would be expected to cause this patient’s
symptoms. Hypocalcemia can cause multisystemic problems, including weakness and seizures, but the
patient’s calcium level is not critically low and hypocalcemia is not associated with hypotension.

Ref: Judge BS, Eisenga BH: Disorders of fuel metabolism: Medical complications associated with starvation, eating disorders,
dietary fads, and supplements. Emerg Med Clin North Am 2005;23(3):789-813. 2) Goldman L, Schafer AI (eds):
Goldman’s Cecil Medicine, ed 24. Elsevier Saunders, 2011, pp 755-756.

Item 134

ANSWER: C

There are many causes of wheezing in infants and children. Wheezing associated with feeding is most
commonly due to gastroesophageal reflux disease (level of evidence 3). Tracheoesophageal fistula and
laryngeal cleft also cause wheezing associated with feeding, but are rare. Foreign body aspiration is most
common between 8 months and 4 years of age and the child is most likely to have a history of the sudden
onset of wheezing associated with choking. The wheezing present with tracheomalacia is position related.

Ref: Weiss LN: The diagnosis of wheezing in children. Am Fam Physician 2008;77(8):1109-1114.

45
Item 135

ANSWER: E

Rhus dermatitis is an allergic phytodermatitis caused by poison ivy and other members of the
Anacardiaceae plant family, including poison oak and poison sumac. Urushiol, an oleoresin found in these
plants, is one of the most common sensitizers in the United States. Contact with these plants results in the
characteristic pruritic erythematous linear lesions with papules and edematous plaques, vesicles, and/or
bullae. The lesions typically appear 48 hours to a few days after exposure and occur on exposed areas.
However, lesions can also occur on remote sites such as the face and penis due to transfer of the oleoresin,
and may not have the characteristic linear appearance. Nickel, which is contained in jewelry and metals
in clothing, produces an eczematous eruption, often with lichenification over the affected areas. Balsam
of Peru (present in topical medications) and neomycin cause allergic contact dermatitis in the area of
application and are nonlinear in appearance. Bedbug bites appear as pruritic erythematous macules with
central hemorrhagic puncta, often in a grouped distribution (SOR C).

Ref: Habif TP: Clinical Dermatology: A Color Guide to Diagnosis and Therapy, ed 5. Mosby Elsevier, 2010, pp 12-15, 34-39.
2) Studdiford JS, Conniff KM, Trayes KP, Tully AS: Bedbug infestation. Am Fam Physician 2012;86(7):653-658. 3)
Longo DL, Fauci AS, Kasper DL, et al (eds): Harrison’s Principles of Internal Medicine, ed 18. McGraw-Hill, 2012, pp
395-404.

Item 136

ANSWER: E

Basic activities of daily living, such as dressing, eating, toileting, and grooming, are generally intact in
early dementia. In contrast, instrumental activities of daily living, such as managing money and
medications, shopping, cooking, housekeeping, and transportation, which often require calculation or
planning, are frequently impaired in early dementia.

Ref: Simmons BB, Hartmann B, Dejoseph D: Evaluation of suspected dementia. Am Fam Physician 2011;84(8):895-902.

Item 137

ANSWER: A

Intoeing, as described in this patient, is usually caused by internal tibial torsion. This problem is believed
to be caused by sleeping in the prone position and sitting on the feet. In 90% of cases internal tibial torsion
gradually resolves without intervention by the age of 8. Avoiding sleeping in a prone position enhances
resolution of the problem. Night splints, orthotics, and shoe wedges are ineffective. Surgery (osteotomy)
has been associated with a high complication rate, and is therefore not recommended in mild cases before
the age of 8.

Ref: Sass P, Hassan G: Lower extremity abnormalities in children. Am Fam Physician 2003;68(3):461-468. 2) Kliegman RM,
Stanton BF, Geme JW III, et al (eds): Nelson Textbook of Pediatrics, ed 19. Elsevier Saunders, 2011, p 2347.

46
Item 138

ANSWER: C

Studies show that few residents have an advance directive at the time of admission to a nursing home.
Studies also show that residents with advance directives are more likely to die in a nursing home with
hospice care, are less likely to have a feeding tube or ventilator in the last month of life, require fewer
resources, and are hospitalized less (SOR B).

Ref: Mollow DW, Guyatt GH, Russo R, et al: Systematic implementation of an advance directive program in nursing homes:
A randomized controlled trial. JAMA 2000;283(11):1437-1444. 2) Unwin BK, Porvaznik M, Spoelhof GD: Nursing home
care: Part I. Principles and pitfalls of practice. Am Fam Physician 2010;81(10):1219-1227.

Item 139

ANSWER: A

The major indication for joint replacement is severe joint pain. Loss of joint function and radiographic
evidence of severe destruction of the joint may also be considered in the decision. The appearance of the
joint and the status of the contralateral joint may be minor considerations. Surgical insertion of a foreign
body into an infected joint is contraindicated.

Ref: Zhang W, Moskowitz RW, Nuki G, et al: OARSI recommendations for the management of hip and knee osteoarthritis, Part
II: OARSI evidence-based, expert consensus guidelines. Osteoarthritis Cartilage 2008;16(2):137-162.

Item 140

ANSWER: C

This audiogram depicts a sensorineural loss in a notch pattern, with the greatest hearing loss at 3500 Hz.
This pattern is most consistent with a noise-induced hearing loss that typically develops gradually as a
result of chronic exposure to excessive sound levels. A vestibular schwannoma most commonly results in
a high-frequency sensorineural hearing loss without the notch. Early on, Meniere’s disease causes a
low-frequency hearing loss, but later it may cause a peaked audiogram with both low- and high-frequency
loss. Complete occlusion of the ear canal with cerumen results in a flat conductive hearing loss across all
frequencies. Tympanic membrane perforation causes a low- to mid-frequency conductive hearing loss.

Ref: Walker JJ, Cleveland LM, Davis JL, Seales JS: Audiometry screening and interpretation. Am Fam Physician
2013;87(1):41-47.

Item 141

ANSWER: E

Community-acquired pneumonia in children over the age of 5 is most commonly due to Mycoplasma
pneumoniae, Chlamydophila pneumoniae, and Streptococcus pneumoniae. Less common bacterial
infections include Haemophilus influenzae, Staphylococcus aureus, and group A Streptococcus. Initial
treatment with antibiotics is empiric, as the pathogen is usually unknown at the time of diagnosis. The
choice in children is based on age, severity of illness, and local patterns of resistance. Children age 5–16
years who can be treated as outpatients are usually treated with oral azithromycin. For patients requiring
inpatient management, intravenous cefuroxime plus either intravenous erythromycin or azithromycin is
recommended.

47
Ref: Stuckey-Schrock K, Hayes BL, George CM: Community-acquired pneumonia in children. Am Fam Physician
2012;86(7):661-667.

Item 142

ANSWER: C

In 50%–90% of patients, pityriasis rosea starts with an erythematous, scaly, oval patch a few centimeters
in diameter. This is usually followed within a few days by smaller patches on the trunk and sometimes the
proximal extremities. Pityriasis rubra pilaris is a rare disease with five types. The classic adult type begins
with a small red plaque on the face or upper body that gradually spreads to become a generalized eruption.
The other conditions listed typically begin with multiple lesions.

Ref: Habif TP: Clinical Dermatology: A Color Guide to Diagnosis and Therapy, ed 5. Mosby Elsevier, 2010, pp 309-311,
316-319, 332-333, 537-540, 771. 2) Goldman L, Schafer AI (eds): Goldman’s Cecil Medicine, ed 24. Elsevier Saunders,
2011, pp 2518-2520.

Item 143

ANSWER: B

This patient is experiencing delirium, which is common in the last weeks of life, occurring in 26%–44%
of persons hospitalized with advanced cancer and in up to 88% of persons with a terminal illness. In
studies of a palliative care population it was possible to determine a cause for delirium in less than 50%
of cases. There is a consensus based on observational evidence and experience that antipsychotic agents
such as haloperidol are effective for the management of delirium, and they are widely used. However,
there have been few randomized, controlled trials to assess their effectiveness.

While benzodiazepines are used extensively in persons with delirium who are terminally ill, there is no
evidence from well-conducted trials that they are beneficial. Trazodone is an antidepressant that is
sometimes used for insomnia. Scopolamine is an anticholinergic that is used to reduce respiratory
secretions in hospice patients, but its anticholinergic side effects would increase delirium severity.
Amitriptyline also has significant anticholinergic properties.

Ref: Keeley PW: Delirium at the end of life. Clinal Evidence Handbook, 2009, pp 595-596. 2) Keeley PW: Delirium at the end
of life. Am Fam Physician 2010;81(10):1260-1261.

Item 144

ANSWER: E

Patients with diabetes mellitus, atherosclerosis, and end-organ damage benefit from ACE inhibitors and
angiotensin receptor blockers (ARBs) equally when they are used to prevent progression of diabetic
nephropathy. Combining an ACE inhibitor with an ARB is not recommended, as it provides no additional
benefit and leads to higher creatinine levels, along with an increased likelihood that dialysis will become
necessary.

Ref: Roett MA, Liegl S, Jabbarpour Y: Diabetic nephropathy—The family physician’s role. Am Fam Physician
2012;85(9):883-889.

48
Item 145

ANSWER: E

The initial treatment of mild to moderate allergic rhinitis should be an intranasal corticosteroid alone, with
the use of second-line therapies for moderate to severe disease (SOR A). The adverse effects and higher
cost of intranasal antihistamines, as well as their decreased effectiveness compared with intranasal
corticosteroids, limit their use as first- or second-line therapy for allergic rhinitis. Moderate to severe
disease not responsive to intranasal corticosteroids should be treated with second-line therapies, including
antihistamines, decongestants, cromolyn, leukotriene receptor antagonists, and nonpharmacologic therapies
such as nasal irrigation.

Ref: Sur DK, Scandale S: Treatment of allergic rhinitis. Am Fam Physician 2010;81(12):1440-1446.

Item 146

ANSWER: B

Synthetic opioid medications are generally not discovered when screening urine for opioids using an
immunoassay method. These synthetic opioids include fentanyl, methadone, and oxymorphone. Opioid
reversal should still be considered in this patient, and a search for a fentanyl patch is indicated.

Ref: Moeller KE, Lee KC, Kissack JC: Urine drug screening: Practical guide for clinicians. Mayo Clin Proc 2008;83(1):66-76.

Item 147

ANSWER: B

Primary hyperaldosteronism is the most common cause of secondary hypertension in the middle-aged
population, and can be diagnosed from a renin/aldosterone ratio. This diagnosis is further suggested by
the finding of hypokalemia, which suggests hyperaldosteronism even though it is not present in the
majority of cases.

An echocardiogram would help make a diagnosis of coarctation of the aorta, but this is more common in
younger patients. Renal MRA may demonstrate renal artery stenosis, but this condition is more common
in older patients. Sleep apnea is increasing in prevalence along with the rise in obesity, but it is not
suggested by this case. A 24-hour urine catecholamine test is used to diagnose pheochromocytoma, which
is not suggested by this patient’s findings. Pheochromocytoma is also less common than aldosteronism
(SOR C).

Ref: Viera AJ, Neutze DM: Diagnosis of secondary hypertension: An age-based approach. Am Fam Physician
2010;82(12):1471-1478.

49
Item 148

ANSWER: A

People taking carbamazepine have a five- to eightfold increased risk of developing agranulocytosis.
Baseline values including a CBC, serum electrolytes, and liver enzymes should be obtained before the drug
is started, and the patient should be monitored with periodic hematologic testing. The other medications
listed are not associated with agranulocytosis. Aripiprazole and olanzapine carry black box warnings for
an increased risk of death in the elderly. Lithium is associated with lithium toxicity and thyroid
dysfunction. Imipramine carries a warning for cardiac toxicity, and EKG monitoring is recommended.

Ref: Stem TA, Rosenbaum JF, Fava M, et al (eds): Massachusetts General Hospital Comprehensive Clinical Psychiatry. Mosby,
2008, pp 40-42.

Item 149

ANSWER: A

Anticholinesterase inhibitors such as donepezil are considered first-line therapy for patients with mild to
moderate Alzheimer’s disease (SOR A). Memantine is an NMDA receptor antagonist and is often used in
combination with anticholinesterase inhibitors for moderate to severe Alzheimer’s disease, but it has not
been shown to be effective as a single agent for patients with mild to moderate disease. There is not enough
evidence to support the use of selegiline, a monoamine oxidase type B inhibitor, in the treatment of
Alzheimer’s disease. Risperidone and other antipsychotic medications are not approved by the Food and
Drug Administration for treatment of Alzheimer’s disease, but can sometimes be helpful in controlling
associated behavioral symptoms. Studies of ginkgo biloba extract have not shown a consistent, clinically
significant benefit in persons with Alzheimer’s disease.

Ref: Winslow BT, Onysko MK, Stob CM, Hazlewood KA: Treatment of Alzheimer disease. Am Fam Physician
2011;83(12):1403-1412.

Item 150

ANSWER: D

Acute subacromial bursitis is common and is often associated with calcific deposits in the supraspinatus
tendon, pain on abduction, and local tenderness. Bicipital tendinitis results in tenderness on palpation of
the tendon of the long head of the biceps. A rotator cuff tear usually results from an injury, and affects
range of motion. Osteoarthritis seldom causes acute, severe pain. A frozen shoulder may result from
subacromial bursitis and presents with limitation of shoulder motion.

Ref: Marx JA (ed): Rosen’s Emergency Medicine: Concepts and Clinical Practice, ed 7. Mosby Elsevier, 2010, pp 585-590.
2) Roberts JR, Hedges JR (eds): Clinical Procedures in Emergency Medicine, ed 5. Saunders Elsevier, 2010, pp 950-951.

50
Item 151

ANSWER: E

Once considered generally beneficial to the running athlete, preparticipation static stretching has been
found lacking in terms of benefit and even detrimental when subjected to scientific study. There is strong
evidence that static stretching significantly slows performance in sprints up to 100 meters. Studies have
failed to demonstrate that static stretching before running significantly decreases the likelihood of muscular
injury of the lower limbs or results in a measurable reduction of delayed-onset muscle soreness. Limited
evidence suggests that preparticipation static stretching, when performed alone, adversely affects both
strength and endurance in elite athletes but has little measurable effect on amateur and casual athletes.
Based on current understanding of sports performance, static stretching is of most benefit when performed
during the cool-down period following exercise, which has been found to increase flexibility, and is best
avoided immediately before athletic endeavors. A preparatory aerobic warm-up combined with dynamic
range-of-motion exercises may be of some benefit for runners.

Ref: Herbert RD, de Noronha M, Kamper SJ: Stretching to prevent or reduce muscle soreness after exercise. Cochrane Database
Syst Rev 2011;(7):CD004577. 2) Yeung SS, Yeung EW, Gillespie LD: Interventions for preventing lower limb soft-tissue
running injuries. Cochrane Database Syst Rev 2011;(7):CD001256.

Item 152

ANSWER: B

A positive Lachman test indicates that the anterior cruciate ligament may be torn. The posterior drawer
test evaluates posterior cruciate ligament stability. The McMurray and Thessaly assessments test for
meniscal tears. The ballottement test is for detecting intra-articular knee effusion.

Ref: Grover M: Evaluating acutely injured patients for internal derangement of the knee. Am Fam Physician
2012;85(3):247-252.

Item 153

ANSWER: B

HPV vaccine is given as a three-dose series, so this patient is due for her third dose. The recommended
interval between the first and third doses is 6 months, with approximately 4 months recommended between
the second and third doses; however, the series can safely be completed at longer intervals (SOR C). The
patient received her second dose at age 12 and she is now 14 years of age, so it has been over 4 months.
She was up to date on all immunizations at age 8, so it can be assumed that she has received her rubella,
measles, and polio vaccinations. There is currently not a vaccine approved for hepatitis C.

Ref: 2012 Recommended immunizations for children from 7 through 18 years old. Centers for Disease Control and Prevention,
American Academy of Pediatrics, American Academy of Family Physicians, 2012.

51
Item 154

ANSWER: A

Testosterone replacement can induce polycythemia, so baseline hematocrit/hemoglobin levels should be


obtained prior to treatment and repeated approximately every 6 months. FSH is not relevant in the workup
or treatment of hypogonadism. Testosterone treatment does not directly affect glucose tolerance or
electrolytes, so baseline studies and follow-up are not necessary.

Ref: Bassil N, Morley JE: Late-life onset hypogonadism: A review. Clin Geriatr Med 2010;26(2):197-222.

Item 155

ANSWER: E

Emerging data on low-carbohydrate diets is mostly encouraging, in that these diets do not seem to cause
the expected increases in blood pressure, LDL-cholesterol levels, or triglyceride levels that the medical
community had first assumed. Although low-carbohydrate diets have been shown to result in clinically
meaningful weight loss, reduced-calorie diets appear to result in similar weight loss regardless of which
macronutrients they emphasize. This patient has symptoms of metabolic syndrome and has a higher risk
of glucose intolerance or diabetes mellitus. Low-carbohydrate diets have been shown to reduce insulin
resistance at least as well as, if not better than, traditional diet plans.

Ref: Nordmann AJ, Nordmann A, Briel M, et al: Effects of low-carbohydrate vs low-fat diets on weight loss and cardiovascular
risk factors: A meta-analysis of randomized controlled trials. Arch Intern Med 2006;166(3):285-293. 2) Barker LR, Burton
JR, Zieve PD (eds): Principles of Ambulatory Medicine, ed 7. Lippincott Williams & Wilkins, 2007, p 1429. 3) Gardner
CD, Kiazand A, Alhassan S, et al: Comparison of the Atkins, Zone, Ornish, and LEARN diets for change in weight and
related risk factors among overweight premenopausal women: The A TO Z Weight Loss Study: A randomized trial. JAMA
2007;297(9):969-977. 4) Sacks FM, Bray GA, Carey VJ, et al: Comparison of weight-loss diets with different
compositions of fat, protein, and carbohydrates. N Engl J Med 2009;360(9):859-873. 5) Longo DL, Fauci AS, Kasper DL,
et al (eds): Harrison’s Principles of Internal Medicine, ed 18. McGraw-Hill, 2012, p 633.

Item 156

ANSWER: E

Certain antiepileptic drugs induce hepatic metabolism of estrogen and progestin (carbamazepine,
oxcarbazepine, phenobarbital, phenytoin, and topiramate). This can potentially lead to failure of any
contraceptive that contains estrogen and progestin. Progestin-only pills are most effective in women who
are exclusively breastfeeding. They are not as effective in pregnancy prevention in other circumstances.
Another effective option for women taking antiepileptic medications would be an intrauterine device. The
levonorgestrel (progestin only) IUD and copper IUD are acceptable choices even for a nulligravida. The
single-rod implantable progestin system also would be an acceptable choice for this patient.

Ref: Bonnema RA, McNamara MC, Spencer AL: Contraception choices in women with underlying medical conditions. Am Fam
Physician 2010;82(6):621-628.

52
Item 157

ANSWER: B

Patients receiving dual antiplatelet therapy who require bypass surgery should continue taking aspirin.
Clopidogrel or prasugrel should be stopped 5 days before the surgery due to the increased risk of major
bleeding during surgery.

Ref: Guyatt GH, Akl EA, Crowther M, et al: Executive summary: Antithrombotic therapy and prevention of thrombosis, 9th
ed: American College of Chest Physicians Evidence-Based Clinical Practice Guidelines. Chest 2012;141(2 Suppl):7S-47S.

Item 158

ANSWER: C

The EKG shows atrial fibrillation with a rapid ventricular rate. The patient is stable, so initial treatment
should focus on rate control. Intravenous $-blockers or nondihydropyridine calcium channel antagonists
are preferred for initial therapy to control the rate. Amiodarone may be used for rhythm control but would
not be the initial treatment of choice in this case. Cardioversion is not indicated unless the patient becomes
unstable. Adenosine is not a recommended treatment for atrial fibrillation.

Ref: Fuster V, Rydén LE, Cannom DS, et al: ACC/AHA/ESC 2006 Guidelines for the Management of Patients with Atrial
Fibrillation: A report of the American College of Cardiology/American Heart Association Task Force on Practice
Guidelines and the European Society of Cardiology Committee for Practice Guidelines (Writing Committee to Revise the
2001 Guidelines for the Management of Patients With Atrial Fibrillation): Developed in collaboration with the European
Heart Rhythm Association and the Heart Rhythm Society. Circulation 2006;114(7):e257-e354. 2) Gutierrez C, Blanchard
DG: Atrial fibrillation: Diagnosis and treatment. Am Fam Physician 2011;83(1):61-68.

Item 159

ANSWER: A

A decrease in serum sodium concentration does not always indicate a decrease in osmolality of body fluids.
In cases of hyperglycemia, the main cause of the hyponatremia is the glucose-related increase in osmolality
of extracellular fluid, followed by the movement of water from intracellular to extracellular fluid
compartments and a subsequent loss of excessive extracellular fluid and electrolytes. The serum sodium
concentration is also diminished in patients with hyperlipidemia or hyperproteinuria because of the volume
occupied by the lipids or proteins. If the lipids or proteins are removed, the sodium concentration in the
remaining plasma is found to be normal. No treatment is needed for these conditions.

Ref: Rakel RE, Rakel DP (eds): Textbook of Family Medicine, ed 8. Elsevier Saunders, 2011, p 200.

Item 160

ANSWER: D

Guidelines from the American Academy of Pediatrics state that stimulant medication can be prescribed for
preschool children, but only after a thorough trial of behavior modification. Foods and additives have
never been shown to cause or aggravate ADHD. Children with ADHD often have other behavioral
problems such as depression or oppositional-defiant disorder.

53
Ref: Subcommittee on Attention-Deficit/Hyperactivity Disorder; Steering Committee on Quality Improvement and Management,
Wolraich M, et al: ADHD: Clinical practice guideline for the diagnosis, evaluation, and treatment of attention-deficit/
hyperactivity disorder in children and adolescents. Pediatrics 2011;128(5):1007-1022.

Item 161

ANSWER: B

This patient is suffering from patellofemoral pain syndrome, which causes anterior knee pain that is worse
with running downhill. The examination is often normal, although there may be apprehension when the
knee is extended with pressure over the patella and the patella will sometimes track laterally.
Patellofemoral pain syndrome can be treated with exercises to strengthen the quadriceps and hips, and by
using a knee sleeve with a doughnut-type cushion that the patella fits into. Static stretching would not
address the problem. MRI would be indicated if there were joint-line pain or an unstable knee. Pes
anserine bursitis usually causes pain and tenderness medially, below the joint line.

Ref: Bope ET, Kellerman R, Rakel RE (eds): Conn’s Current Therapy 2011. Elsevier Saunders, 2011, p 1026.

Item 162

ANSWER: A

Rheumatoid arthritis is most often symmetric at presentation and particularly affects the wrists and other
extremity joints that have a high ratio of synovium to articular cartilage. Rheumatoid factor is often
negative in the early months of the disease, although it may be positive later. Radiographs and laboratory
tests are helpful, but the diagnosis is primarily clinical. Osteoarthritis of the wrists usually involves the
carpal-metacarpal joint of the thumb primarily, and the joint would be red if there were an injury.
Fibromyalgia usually involves the soft tissue of the trunk, and there is no evidence of inflammation. Lyme
disease can cause a variety of joint diseases, but not chronic symmetric arthritis.

Ref: Wasserman AM: Diagnosis and management of rheumatoid arthritis. Am Fam Physician 2011;84(11):1245-1252. 2)
Goldman L, Schafer AI (eds): Goldman’s Cecil Medicine, ed 24. Elsevier Saunders, 2011, pp 1685-1686.

Item 163

ANSWER: B

The parallelogram shape of this infant’s head is typical of positional skull deformity, also known as benign
positional molding or occipital plagiocephaly. This condition has been estimated to be present in at least
1 in 300 infants, with some studies showing milder variants in up to 48% of healthy infants. The incidence
of positional skull deformity is increased in children who sleep in the supine position, but switching to
prone sleeping is not recommended because this would increase the risk of sudden infant death syndrome.
The deformity can be prevented by routine switching of the dependent side of the infant’s head. Supervised
“tummy time” for 30–60 minutes each day can also decrease the amount of flattening and can increase the
child’s motor development. Children who have positional skull deformity should also be screened for
torticollis. This condition can prevent correct positioning and is remedied with physical therapy techniques.

Positional skull deformity should be differentiated from cranial synostosis, which is the result of abnormal
fusion of one or more of the sutures between the skull bones. Ipsilateral frontal bossing and ear
advancement are not seen, resulting in a trapezoid-shaped head.

54
Most infants with positional skull deformity improve within 2–3 months with the institution of positional
changes and tummy time. If the condition does not significantly improve after this amount of time, referral
to a pediatric neurosurgeon with expertise in craniofacial malformations would be appropriate.

Ref: Laughlin J, Luerssen TG, Dias MS; Committee on Practice and Ambulatory Medicine, Section on Neurological Surgery:
Prevention and management of positional skull deformities in infants. Pediatrics 2011;128(6):1236-1241.

Item 164

ANSWER: C

Somatoform disorders are often encountered in family medicine. Studies have documented that 5% of
patients meet the criteria for somatization disorder, while another 4% have borderline somatization
disorder. Most of these patients are female and have a low socioeconomic status. They have a high
utilization of medical services, usually reflected by a thick medical chart, and are often single parents.
Physicians tend to be less satisfied with the care rendered to these patients compared to those without the
disorder. Patients with multiple unexplained physical complaints have been described as functionally
disabled, spending an average of one week per month in bed. Many of these patients seek and ultimately
undergo surgical procedures, and it is not uncommon for them to have multiple procedures, especially
involving the pelvic area. Often there are associated psychiatric symptoms such as anxiety, depression,
suicide threats, alcohol or drug abuse, interpersonal or occupational difficulties, and antisocial behavior.
A history of a dysfunctional family unit in which one or both parents abused alcohol or drugs or were
somatically preoccupied is also quite common. These individuals often enter relationships with alcohol
abusers.

Somatization disorder should be managed by one primary physician so that an established relationship and
regular visits can help curtail the dramatic symptoms that otherwise may lead to hospitalization. The family
physician is in a position to monitor family dynamics and provide direction on such issues as alcoholism
and child abuse. Each office visit should include a physical examination, and the temptation to tell the
patient that the problem is not physical should be avoided. Knowing the patient well helps to avoid
unnecessary hospitalizations, diagnostic procedures, surgery, and laboratory tests. These measures should
be carried out only if clearly indicated. Psychotropic medications should be avoided except when clearly
indicated, as medications reinforce the sick role, may be abused, and may be used for suicide gestures.
Following these recommendations significantly decreases the cost of care for the patient.

Ref: Sadock BJ, Sadock VA, Ruiz P (eds): Kaplan & Sadock’s Comprehensive Textbook of Psychiatry, ed 9. Lippincott
Williams & Wilkins, 2009, pp 1927-1940. 2) Goldman L, Schafer AI (eds): Goldman’s Cecil Medicine, ed 24. Elsevier
Saunders, 2011, p 2244.

Item 165

ANSWER: E

Sarcoidosis is a disease of unknown cause characterized by the presence of noncaseating epithelioid


granulomas; it involves many different organ systems. The lungs are commonly involved; bilateral hilar
lymphadenopathy is often present and pulmonary infiltrates and fibrosis somewhat typical. Sarcoidosis may
also affect the skin, central nervous system, eyes, liver, heart, salivary glands, kidneys, muscles, or bones.
When the disease is limited to asymptomatic hilar adenopathy, it is termed stage I and no treatment has
been shown to be beneficial. The most appropriate management of stage I patients is routine follow-up.

Ref: Wu JJ, Schiff KR: Sarcoidosis. Am Fam Physician 2004;70(2):312-322. 2) Longo DL, Fauci AS, Kasper DL, et al (eds):
Harrison’s Principles of Internal Medicine, ed 18. McGraw-Hill, 2012, pp 2805-2813.

55
Item 166

ANSWER: E

This patient has cor pulmonale. Patients should be assessed for chronic oxygen therapy, which has been
shown to reduce hospitalization rates and mortality (SOR A). O2 saturation is <88% in most cases.
Oxygen therapy may be justified with a slightly higher O2 saturation if cor pulmonale is well documented
in a patient with COPD. Cautious diuretic therapy may be useful for symptomatic edema.

Digoxin is not thought to be beneficial in the absence of atrial fibrillation and is more likely to cause an
arrhythmia in a hypoxic patient. "-Blockers, calcium channel blockers, and ACE inhibitors are not
recommended for cor pulmonale. Calcium channel blockers and vasodilators may have some benefit in
primary pulmonary hypertension, but they have not proven beneficial in COPD-related cor pulmonale.

Ref: National Clinical Guideline Centre: Chronic Obstructive Pulmonary Disease: Management of Chronic Obstructive
Pulmonary Disease in Adults in Primary and Secondary Care. National Institute for Health and Clinical Excellence (NICE),
2010.

Item 167

ANSWER: D

Common variable immunodeficiency is the most commonly diagnosed disorder among the primary
immunodeficiencies. It is a disorder of humoral immunity associated with reduced serum levels of IgG,
IgM, and IgA, and frequently presents as late as the third or fourth decade of life. The disorder is
associated with recurrent sinus infections, otitis media, bronchiectasis, and chronic gastrointestinal
problems. Recognition of the disorder is important, as infections may be reduced when patients are treated
with intravenous immune globulin.

Abnormalities in the other test results are compatible with less common primary immunodeficiencies.
Lymphopenia suggests a disorder of cellular immunity such as severe combined immunodeficiency,
thrombocytopenia suggests the Wiskott-Aldrich syndrome, and an abnormal nitroblue tetrazolium test
suggests a phagocytic disorder.

Ref: Cooper MA, Pommering TL, Korányi K: Primary immunodeficiencies. Am Fam Physician 2003;68(10):2001-2009. 2)
Goldman L, Schafer AI (eds): Goldman’s Cecil Medicine, ed 24. Elsevier Saunders, 2011, pp 1615-1622. 3) Reust CE:
Evaluation of primary immunodeficiency disease in children. Am Fam Physician 2013;87(11):773-778.

Item 168

ANSWER: D

The U.S. Preventive Services Task Force recommends vitamin D supplementation to prevent falls in
community-dwelling adults 65 and older who are at increased risk for falls (grade B recommendation).
Some studies suggest that low vitamin D levels are associated with an increased risk of cardiovascular
disease, multiple sclerosis, colon cancer, dementia, and even diabetes mellitus, but these studies are
epidemiologic and thus are not based on high-quality evidence (SOR C).

Ref: Bordelon P, Ghetu MV, Langan RC: Recognition and management of vitamin D deficiency. Am Fam Physician
2009;80(8):841-854. 2) Kulie T, Groff A, Redmer J, et al: Vitamin D: An evidence-based review. J Am Board Fam Med
2009;22(6):698-706. 3) Ott SM: ACP Journal Club. Review: Vitamin D with calcium reduces fractures in adults. Ann
Intern Med 2012;156(12):JC6-JC7. 4) US Preventive Services Task Force: Prevention of Falls in Community-Dwelling
Older Adults: US Preventive Services Task Force Recommendation Statement. AHRQ pub no 11-05150-EF-2, 2012.

56
Item 169

ANSWER: B

For the average adult over the age of 60, the normal time required for the Timed Up and Go test is 10
seconds. A time longer than 10 seconds may indicate weakness, a balance or gait problem, and/or an
increased fall risk.

Ref: Croswell J, Young-Rok S: Prevention of falls in community-dwelling older adults. Am Fam Physician
2012;86(12):1135-1136.

Item 170

ANSWER: A

The current management of methanol intoxication, depending on its severity, includes ethanol
administration to inhibit the metabolism of methanol, hemodialysis to remove alcohol and its toxins, and
vigorous management of metabolic acidosis with bicarbonate therapy. Ethanol is a competitive inhibitor
of toxin metabolism and slows the formation of toxic metabolites, formaldehyde, and formic acid from
methanol, permitting these products to be disposed of by ordinary metabolic or excretory pathways. It has
a similar effect in ethylene glycol poisoning, slowing the formation of glycoaldehyde and glycolic,
glyoxylic, and oxalic acids.

Ref: Marx JA (ed): Rosen’s Emergency Medicine: Concepts and Clinical Practice, ed 7. Mosby Elsevier, 2010, pp 2001-2007.

Item 171

ANSWER: C

Treatment for a unilateral undescended testis should be started at 6–12 months of age to avoid testicular
damage. It was once thought that delaying descent lowered the incidence of testicular cancer, but it is now
believed that orchiopexy allows for early cancer detection. HCG treatment may promote descent into the
distal canal, but the testicle often ascends again. Ultrasonography will not show an undescended testis in
many cases and is therefore not recommended. Hormonal treatments have been used in Europe but
randomized, controlled trials have not shown them to be effective.

Ref: Docimo SG, Silver RI, Cromie W: The undescended testicle: Diagnosis and management. Am Fam Physician
2000;62(9):2037-2044. 2) Kliegman RM, Stanton BF, Geme JW III, et al (eds): Nelson Textbook of Pediatrics, ed 19.
Elsevier Saunders, 2011, pp 1859-1860.

Item 172

ANSWER: C

Head trauma is a known cause of the syndrome of inappropriate secretion of antidiuretic hormone
(SIADH). This patient’s course has been very acute, with hyponatremia developing within 48 hours. Such
a precipitous drop in serum sodium may lead to cerebral and pulmonary edema. If left untreated the patient
can have seizures, become obtunded, and die from brain herniation. These dangers require immediate
treatment with hypertonic saline to correct the falling levels of sodium. This must be done cautiously so
as to not overcorrect the sodium level too quickly, which could lead to osmotic demyelination syndrome.
An increase in serum sodium levels of about 6 mEq/L should be enough to reduce symptoms and prevent
progressive cerebral edema.

57
Ref: Longo DL, Fauci AS, Kasper DL, et al (eds): Harrison’s Principles of Internal Medicine, ed 18. McGraw-Hill, 2012, pp
341-359.

Item 173

ANSWER: D

Parvovirus B19 is associated with erythema infectiosum, or fifth disease. It is also associated with
nonspecific fever, arthropathy, chronic anemia, and transient aplastic crisis.

Ref: Sabella C, Goldfarb J: Parvovirus B19 infections. Am Fam Physician 1999;60(5):1455. 2) Longo DL, Fauci AS, Kasper
DL, et al (eds): Harrison’s Principles of Internal Medicine, ed 18. McGraw-Hill, 2012, pp 1478-1480.

Item 174

ANSWER: C

Natural family planning (NFP) is a potentially effective method for contraception and for determining the
time of ovulation for purposes of conception. While the contraceptive effectiveness of the different NFP
methods varies significantly, the success rates for typical use are as high as 92%–98% (SOR B).
Monitoring the presence and consistency of cervical mucus production allows for the determination of both
the beginning and end of a woman’s most fertile period. Some NFP methods use cervical mucus secretion
as the sole basis for determining fertility. The symptothermal method also incorporates calendar
calculations, basal body temperature measurement, and ovulation-related symptoms as a complement to
the cervical mucus component. The Marquette Model incorporates cervical mucus and basal body
temperature charting with electronic monitoring of urine estrogen and LH metabolites to provide additional
information to determine when ovulation has occurred.

Ref: Smoley BA, Robinson CM: Natural family planning. Am Fam Physician 2012;86(10):924-928.

Item 175

ANSWER: B

Stillbirth is defined as fetal death occurring at or after 20 weeks gestation, and affects approximately 1 in
160 pregnancies in the United States. A large study of stillbirths from 2006 to 2008 tried to establish a
cause in 663 cases, and a probable or possible cause was identified in approximately 75% of these. While
there were some significant ethnic differences, placental abnormalities and obstetric complications were
the largest category of causes in white women, and this was even more true after 32 weeks gestation. Other
important causes included infection and fetal defects. More than one cause was found in one-third of cases.

Ref: Stillbirth Collaborative Research Network Writing Group: Causes of death among stillbirths. JAMA
2011;306(22):2459-2468.

Item 176

ANSWER: E

Primidone and propranolol are the first-line drugs for essential tremor. Alprazolam is considered to be
possibly effective. The tremor described is not due to Parkinson’s disease and would not respond to
carbidopa/levodopa. Levetiracetam is not effective. There is not enough evidence to recommend for or
against the use of olanzapine.

58
Ref: Zesiewicz TA, Elble RJ, Louis ED, et al: Evidence-based guideline update: Treatment of essential tremor: Report of the
Quality Standards Subcommittee of the American Academy of Neurology. Neurology 2011;77(19):1752-1755.

Item 177

ANSWER: E

This patient has pneumonia, sepsis, and suspected coinfection with influenza. Although the rapid
antigen-based nasal swab was negative, false-negative rates may be as high as 70% and this test should not
be relied upon to rule out influenza. Treatment should include both antiviral and antibacterial agents that
include coverage against methicillin-resistant Staphylococcus aureus (MRSA), the most common bacterial
pathogen isolated from critically ill patients with coinfection. Oseltamivir, ceftriaxone, azithromycin, and
vancomycin should be initiated empirically for the pneumonia and sepsis. The criteria for sepsis are
satisfied by a temperature >38.3°C, a WBC count >12,000/mm3, a respiratory rate >20/min, and a
source of probable infection.

Ref: Chertow DS, Memoli MJ: Bacterial coinfection in influenza: A grand rounds review. JAMA 2013;309(3):275-282.

Item 178

ANSWER: D

Emollients are a mainstay of chronic therapy for atopic dermatitis (SOR C), but topical corticosteroids are
the first-line treatment for flare-ups (SOR A). Calcineurin inhibitors such as pimecrolimus are a
second-line treatment for moderate to severe atopic dermatitis (SOR A). Antibiotics are not useful in
reducing flare-ups of atopic dermatitis unless there is clear evidence of a secondary infection (SOR A).
Neither topical nor oral antihistamines are recommended for routine treatment of atopic dermatitis because
they are not effective in treating the associated pruritus.

Ref: Berke R, Singh A, Guralnick M: Atopic dermatitis: An overview. Am Fam Physician 2012;86(1):35-42.

Item 179

ANSWER: A

The most cost-effective intervention for type 2 diabetes mellitus is lifestyle modification. The difficulty
with this intervention, however, lies in keeping the patient motivated. Metformin is also cost-effective.

Bariatric surgery shows immediate benefits in lowering glucose levels in patients who have undergone this
treatment. Unfortunately, studies on its long-term effects are lacking. Sitagliptin, pioglitazone, and insulin
are effective secondary agents but are expensive and not as cost-effective as metformin or dietary changes
and exercise.

Ref: Ismail-Beigi F: Glycemic management of type 2 diabetes mellitus. N Engl J Med 2012;366(14):1319-1327.

59
Item 180

ANSWER: C

American Heart Association guidelines recommend treating hypertension in patients with stable heart
failure with ACE inhibitors and/or $-blockers. Other agents, such as thiazide diuretics or calcium channel
blockers, can be added if needed to achieve blood pressure goals (SOR B). $-Blockers with intrinsic
sympathomimetic activity should be avoided, as they increase myocardial oxygen demand.

While thiazide diuretics are often a first choice for uncomplicated hypertension, this is not the case for
patients with coronary artery disease. Long-acting calcium channel blockers may be used in patients who
do not tolerate $-blockers, but short-acting calcium channel blockers should be avoided because they
increase mortality. ACE inhibitors are recommended as antihypertensive agents in patients already on
$-blocker therapy (especially following myocardial infarction), in diabetics, and in patients with left
ventricular dysfunction. Although angiotensin receptor blockers have indications similar to those of ACE
inhibitors, the American Heart Association recommends using them only in patients who do not tolerate
ACE inhibitors. Long-acting nitrates are used for their anti-anginal properties and have no role in the
management of hypertension.

Ref: Pflieger M, Winslow BT, Mills K, Dauber IM: Medical management of stable coronary artery disease. Am Fam Physician
2011;83(7):819-826. 2) Fihn SD, Gardin JM, Abrams J, et al: 2012 ACCF/AHA/ACP/AATS/PCNA/SCAI/STS guideline
for the diagnosis and management of patients with stable ischemic heart disease: A report of the American College of
Cardiology Foundation/American Heart Association task force on practice guidelines, and the American College of
Physicians, American Association for Thoracic Surgery, Preventive Cardiovascular Nurses Association, Society for
Cardiovascular Angiography and Interventions, and Society of Thoracic Surgeons. Circulation 2012;126(25):e354-e471.

Item 181

ANSWER: B

This patient’s symptoms are compatible with Meniere’s disease, which is characterized by multiple
episodes of vertigo lasting for 20–120 minutes, accompanied by a fluctuating hearing loss, tinnitus, and
a sense of aural fullness. Audiograms will reveal a low-frequency hearing loss with an upsloping curve,
which can become flattened over the years. Most patients develop unilateral symptoms, and many patients
will develop bilateral disease many years after the onset of the unilateral symptoms. Multiple studies have
reported the rate of bilateral Meniere's disease to be as high as 50% many years after the initial diagnosis.

Motion sickness is a common cause of nausea, but the nausea usually does not come on suddenly and is
not as pronounced as with Meniere’s disease. A vestibular migraine can present like a sudden Meniere’s
disease attack but in this patient the audiograms, tinnitus, and aural fullness suggest Meniere’s disease.
Benign positional vertigo is very common, and hearing loss could be an incidental finding. However, the
most common form of age-related hearing loss is seen at the higher frequencies. Positional vertigo like this
patient has is common between attacks of Meniere’s disease.

There is often a family history of Meniere’s disease, and there is frequently an association with allergies.
The condition can also get worse with caffeine use. Even though the diagnosis is clinical, MRI and blood
tests are recommended to rule out other conditions that may be putting pressure on the endolymphatic
system and thus causing the symptoms.

Ref: Sajjadi H, Paparella MM: Meniere’s disease. Lancet 2008;372(9636):406-414.

60
Item 182

ANSWER: C

This patient has bipolar II disorder. She has a history of hypomanic episodes as well as major depression,
with no history of a manic or mixed episode. Among the pharmacologic options listed, only divalproex
and lithium are indicated for treating bipolar depression or acute mania, and for maintenance. They should
be given as single agents, however, not in combination with other drugs. No evidence supports
combination therapy or the addition of an antidepressant in the acute phase of depression.

In a study of patients with bipolar II disorder, initially adding paroxetine or bupropion to the mood
stabilizer was no more effective than using lithium or valproate. An SSRI or bupropion can be added if
a therapeutic dosage of a mood stabilizer does not resolve symptoms and the patient is not in a mixed state.
Tricyclic antidepressants and antidepressants with dual properties, such as venlafaxine, should be avoided
because they may induce mania. Aripiprazole is indicated for acute mania but not for bipolar depression.

Ref: Price AL, Marzani-Nissen GR: Bipolar disorders: A review. Am Fam Physician 2012;85(5):483-493.

Item 183

ANSWER: E

The Future Revascularization Evaluation in Patients with Diabetes Mellitus: Optimal Management of
Multivessel Disease (FREEDOM) trial funded by the National Heart, Lung, and Blood Institute concluded
that in patients with diabetes mellitus and advanced coronary artery disease, coronary artery bypass graft
surgery was superior to percutaneous coronary intervention (PCI) in that it significantly reduced rates of
death and myocardial infarction, although stroke rates were higher in the 30-day perioperative period. The
FREEDOM trial suggested that these outcomes are similar whether PCI is performed without stents, with
bare-metal stents, or with drug-eluting stents. These results were consistent with reports from other smaller
or retrospective studies of revascularization in patients with diabetes mellitus.

Ref: Farkouh ME, Domanski M, Sleeper LA, et al: Strategies for multivessel revascularization in patients with diabetes. N Engl
J Med 2012;367(25):2375-2384. 2) Hlatky MA: Compelling evidence for coronary-bypass surgery in patients with diabetes.
N Engl J Med 2012;367(25):2437-2438.

Item 184

ANSWER: D

In young children with bilious emesis, anorexia, and lack of fever, the most likely diagnosis is intestinal
malrotation with volvulus. Abdominal ultrasonography is less sensitive and specific for malrotation than
an upper gastrointestinal series, so an upper GI series should be ordered initially if volvulus is suspected.
If appendicitis were suspected, ultrasonography would be preferred. CT is not a good choice because of
the amount of radiation it delivers, especially given efforts to decrease the use of CT in children unless
absolutely necessary. This patient’s presentation is not typical for testicular torsion, therefore scrotal
ultrasonography should not be the initial test of choice.

Ref: Saito JM: Beyond appendicitis: Evaluation and surgical treatment of pediatric acute abdominal pain. Curr Opin Pediatr
2012;24(3):357-364.

61
Item 185

ANSWER: B

Ingestion of grapefruit juice can increase absorption and serum levels of statins, leading to an increased
risk of muscle injury. The mechanism for this is believed to be the cytochrome p-450 pathway. Starfruit
juice and pomegranate juice can have a similar effect. These juices contain an irreversible inhibitor of
intestinal CYP3A4, and increase the bioavailability of atorvastatin, lovastatin, and simvastatin.
Rosuvastatin and fluvastatin utilize the CYP2C9 system for metabolism, so the effect on these drugs is
minimal.

Grapefruit juice reduces CYP3A4 activity by 50% within 4 hours of ingestion, and activity is reduced by
30% for as long as 24 hours after ingestion. Several studies document that consuming 600 mL of
double-strength juice for 3 days produces a more than tenfold increase in the area under the curve for
simvastatin and lovastatin, but only a 250% increase in atorvastatin.

Ref: Venero CV, Thompson PD: Managing statin myopathy. Endocrinol Metabol Clin North Am 2009;38(1):121-136. 2)
Bonow RO, Mann DL, Zipes DP, Libby P (eds): Braunwald’s Heart Disease: A Textbook of Cardiovascular Medicine,
ed 9. Elsevier Saunders, 2011, p 987.

Item 186

ANSWER: A

HMG-CoA reductase inhibitors, or statins, play an important role in the management of patients with
cardiovascular disease and have an excellent safety and tolerability record. The incidence of significant
liver injury from statin drugs is about 1%, and nonalcoholic fatty liver disease or stable hepatitis B or C
infection is not a contraindication to treatment with statins. Although many patients taking statins
experience elevation of hepatic transaminases, these elevations are generally mild and asymptomatic, and
often resolve spontaneously even with no changes in treatment. Transaminase elevations up to three times
the upper limit of normal are not a contraindication to continued use of the drug at the same dosage.

Ref: Gillett RC Jr, Norrell A: Considerations for safe use of statins: Liver enzyme abnormalities and muscle toxicity. Am Fam
Physician 2011;83(6):711-716.

Item 187

ANSWER: D

A 95% confidence interval is an estimate of certainty. It means there is 95% certainty that the true value
lies within the given interval range. When a confidence interval crosses 1.00, the validity of the resulting
statistical estimate is questionable. Sensitivity is the percentage of patients with a disease who have a
positive test for the disease. Specificity is the percentage of patients without a disease who have a negative
test for the disease. Relative risk reduction is the percentage difference in risk between the treatment and
control groups. The number needed to treat is the number of patients who need to receive an intervention
instead of the alternative in order for one additional patient to benefit.

Ref: Glossary of evidence-based medicine and statistical terms. Am Fam Physician 2012;86(3):231.

62
Item 188

ANSWER: C

The Fourth Report on the Diagnosis, Evaluation, and Treatment of High Blood Pressure in Children and
Adolescents defines hypertension in children as a systolic or diastolic blood pressure above the 95th
percentile for the patient’s sex, age, and height on several different readings. Although it is appropriate
to have this finding confirmed in the outpatient setting, 130 mm Hg is still at the 99th percentile for systolic
blood pressures in this patient. Hypertension in a patient this young should prompt a search for secondary
causes, which are more common in young hypertensive patients than in adults with hypertension. The
recommended workup includes blood and urine testing, as well as renal ultrasonography. An evaluation
for end-organ damage is also recommended, including retinal evaluation and echocardiography.

Ref: Kliegman RM, Stanton BF, Geme JW III, et al (eds): Nelson Textbook of Pediatrics, ed 19. Elsevier Saunders, 2011, pp
1639-1647. 2) Riley M, Bluhm B: High blood pressure in children and adolescents. Am Fam Physician
2012;85(7):693-700.

Item 189

ANSWER: B

For a healthy nonsmoker with no chronic disease who is not in a high-risk group, pneumococcal vaccine
is recommended once at age 65, or as soon afterward as possible. Persons that should be immunized before
age 65 include patients with chronic lung disease, cardiovascular disease, diabetes mellitus, chronic liver
disease, cerebrospinal fluid leaks, cochlear implants, immunocompromising conditions, or asplenia, and
residents of nursing homes and long-term care facilities. The Advisory Committee on Immunization
Practices of the CDC updated the recommendations for pneumococcal vaccination in 2011 to include
immunization for persons age 50–64 in the following categories: Alaska Natives, Native Americans living
in areas of increased risk, persons with asthma, and smokers.

Ref: Centers for Disease Control and Prevention (CDC); Advisory Committee on Immunization Practices: Updated
recommendations for prevention of invasive pneumococcal disease among adults using the 23-valent pneumococcal
polysaccharide vaccine (PPSV23). MMWR Morb Mortal Wkly Rep 2010;59(34):1102-1105. 2) Vaughn JA, Miller RA:
Update on immunizations in adults. Am Fam Physician 2011;84(9):1015-1017.

Item 190

ANSWER: E

NSAIDs, pyridoxine, and diuretics have been shown to be no more effective than placebo in the treatment
of patients with carpal tunnel syndrome. Splinting, physical therapy, and corticosteroid injections have all
been shown to result in short-term improvement. Patients with persistent symptoms achieve the best
long-term relief with surgery.

Ref: Daniels JM, Muller MH: Hand and Wrist Injuries. FP Essentials monograph series, no 400, 2012, pp 34-38.

63
Item 191

ANSWER: A

A number of pretest probability scoring systems are available for assessing venous thromboembolism
(VTE), which includes deep vein thrombosis (DVT) and pulmonary embolism. Although the Wells clinical
prediction rule is widely used, other tools such as the Hamilton score and the AMUSE (Amsterdam
Maastricht Utrecht Study on thromboEmbolism) score are also available. The Wells rule divides patients
suspected of having a DVT into low, intermediate, and high-risk categories, with a 5%, 17%, and 53%
prevalence of DVT, respectively. This patient has a Wells score of 0 (+1 for calf circumference increase
>3 cm, +1 for pitting edema, –2 for a likely alternative diagnosis of gastrocnemius strain) and is
therefore at low risk. A negative D-dimer assay has a high negative predictive value for DVT, so the
diagnosis can be ruled out in a patient who has a low pretest probability and a negative D-dimer result. A
negative D-dimer assay does not rule out DVT in a patient with a moderate to high pretest probability
(SOR C).

Ref: Wilbur J, Shian B: Diagnosis of deep venous thrombosis and pulmonary embolism. Am Fam Physician
2012;86(10):913-919.

Item 192

ANSWER: E

This patient presents with a classic description of supraventricular tachycardia (SVT). The initial
management of SVT centers around stopping the aberrant rhythm. In the hemodynamically stable patient
initial measures should include vagal maneuvers (SOR C), intravenous adenosine or verapamil (SOR B),
intravenous diltiazem or $-blockade, intravenous antiarrhythmics, or cardioversion in refractory cases.
While digoxin is occasionally useful in atrial fibrillation with a rapid ventricular rate, it is not
recommended for SVT. Radiofrequency ablation is fast becoming the first-line therapy for all patients with
recurrent SVT, not just those refractory to suppressive drug therapies. Observational studies have shown
that this therapy results in improved quality of life and lower cost as compared to drug therapy (SOR B).

Ref: King DE, Dickerson LM, Sack JL: Acute management of atrial fibrillation: Part 1. Rate and rhythm control. Am Fam
Physician 2002;66(2):249-256. 2) Colucci RA, Silver MJ, Shubrook J: Common types of supraventricular tachycardia:
Diagnosis and management. Am Fam Physician 2010;82(8):942-952.

Item 193

ANSWER: C

Acute respiratory distress syndrome (ARDS) may be caused by pulmonary sepsis or sepsis from another
source, or it may be due to acute pulmonary injury, including inhalation of smoke or other toxins.
Inflammatory mediators are released in response to the pulmonary infection or injury. The syndrome has
an acute onset and is manifested by rapidly developing profound hypoxia with bilateral pulmonary
infiltrates. The mortality rate in patients with ARDS may be as high as 55%.

Early recognition and prompt treatment with intubation and mechanical ventilation is necessary to improve
chances for survival. Patients with ARDS should be started at lower tidal volumes (6 mL/kg) instead of
the traditional volumes (10–15 mL/kg) (SOR A). These patients also often require higher positive
end-expiratory pressure settings (SOR B).

64
Fluid management should be conservative to allow for optimal cardiorespiratory and renal function and
to avoid fluid overload. However, the routine use of central venous or pulmonary artery pressure catheters
is not recommended due to the potential complications associated with their use (SOR A). While surfactant
is commonly used in children with ARDS, it does not improve mortality in adults (SOR A).

Ref: Saguil A, Fargo M: Acute respiratory distress syndrome: Diagnosis and management. Am Fam Physician
2012;85(4):352-358.

Item 194

ANSWER: A

First-line treatment for diabetic peripheral neuropathy, according to the American Diabetes Association,
is tricyclic antidepressants. Anticonvulsants are second line and opioids are third line. Many medications
have been found to be effective, including the tricyclics, duloxetine, pregabalin, oxycodone, and tramadol
(SOR A).

Among the tricyclics, amitriptyline, imipramine, and nortriptyline have been found to be the most effective
(SOR A). For an uninsured patient, the tricyclics are also the most affordable.

Ref: Page N, Deluca J, Crowell K: What medications are best for diabetic neuropathic pain? J Fam Pract 2012;61(11):691-693.

Item 195

ANSWER: D

The child described has facial features characteristic of fetal alcohol syndrome. Fetal alcohol spectrum
disorders (FASD) are caused by the effects of maternal alcohol consumption during pregnancy. Fetal
alcohol syndrome is the most clinically recognized form of FASD and is characterized by a pattern of
minor facial anomalies, including a thin upper lip, a smooth philtrum, and a flat nasal bridge; other
physical anomalies, such as clinodactyly; prenatal and postnatal growth retardation; and functional or
structural central nervous system abnormalities.

Children with Down syndrome have hypotonia, a flat face, upward and slanted palpebral fissures and
epicanthic folds, and speckled irises (Brushfield spots); varying degrees of mental and growth retardation;
dysplasia of the pelvis; cardiac malformations; a simian crease; short, broad hands; hypoplasia of the
middle phalanx of the 5th finger; and a high, arched palate.

Marfan syndrome is characterized by pectus carinatum or pectus excavatum, an arm span to height ratio
>1.05, a positive wrist and thumb sign, limited elbow extension, pes planus, and aortic ascendens
dilatation with or without aortic regurgitation.

The bilateral renal agenesis seen with Potter syndrome leads to death shortly after birth. Other anomalies
include widely separated eyes with epicanthic folds, low-set ears, a broad and flat nose, a receding chin,
and limb anomalies.

Finally, Prader-Willi syndrome is characterized by severe hypotonia at birth, obesity, short stature
(responsive to growth hormone), small hands and feet, hypogonadism, and mental retardation.

Ref: Wattendorf DJ, Muenke M: Fetal alcohol spectrum disorders. Am Fam Physician 2005;72(2):279-282, 285. 2) Kliegman
RM, Stanton BF, Geme JW III, et al (eds): Nelson Textbook of Pediatrics, ed 19. Elsevier Saunders, 2011, pp 400-403,
412-413, 625-626, 1827, 2240-2246.

65
Item 196

ANSWER: A

While there is substantial overlap in the signs, symptoms, and physical findings for the various etiologies
of chest pain, a good history and physical examination can help determine which patients require
immediate further evaluation for a potentially serious cause. The chest pain associated with pericarditis is
typically pleuritic, and is worse with inspiration or in positions that put traction on the pleuropericardial
tissues, such as lying supine. Patients with acute pericarditis typically get relief or improvement when there
is less tension on the pericardium, such as when sitting and leaning forward. This position brings the heart
closer to the anterior chest wall, which incidentally is the best position for hearing the pericardial friction
rub associated with acute pericarditis.

Radiation of chest pain to both arms should raise concerns about myocardial ischemia or infarction.
Radiation to the right scapula is sometimes seen with cholelithiasis. Worsening pain with the Valsalva
maneuver is nonspecific and is of no particular diagnostic value.

Ref: McConaghy JR, Oza RS: Outpatient diagnosis of acute chest pain. Am Fam Physician 2013;87(3):177-182.

Item 197

ANSWER: A

Children with diabetes mellitus are at increased risk for retinopathy, nephropathy, and hypertension. They
are also more likely to have immune-mediated disorders such as celiac disease and hypothyroidism. For
all children and adolescents with type 1 diabetes mellitus, the American Diabetes Association recommends
screening for hypothyroidism, nephropathy, hypertension, celiac disease, and retinopathy. Screening for
dyslipidemia should be considered if there is a family history of hypercholesterolemia or cardiac events
before age 55.

Ref: American Diabetes Association: Standards of medical care in diabetes—2013. Diabetes Care 2013;36(Suppl 1):S40-S43.

Item 198

ANSWER: B

The most likely diagnosis is septic arthritis of the hip. Ultrasonography is highly sensitive for the effusion
seen in septic arthritis, which can be aspirated to confirm the diagnosis (SOR A). It is important to
diagnose this problem as soon as possible. Clinical features of septic arthritis include an oral temperature
>38.5°C (101.3°F), refusal to bear weight on the affected leg, an erythrocyte sedimentation rate >40
mm/hr, a peripheral WBC count >12,000/mm3, and a C-reactive protein level >20 mg/L. If
ultrasonography is negative, a bone scan should be done. CT of the hip is indicated to visualize cortical
bone. MRI is especially valuable for osteomyelitis.

Ref: Sawyer J, Kapoor M: The limping child: A systematic approach to diagnosis. Am Fam Physician 2009;79(3):215-224.

66
Item 199

ANSWER: A

Asymptomatic gallstones are not usually an indication for prophylactic cholecystectomy, as most patients
remain asymptomatic throughout their lives, and only 1%–4% develop symptoms or complications from
gallstones each year. Only 10% of patients found to have asymptomatic gallstones develop symptoms
within the first 5 years after diagnosis, and only 20% within 20 years.

In the past, cholecystectomy was recommended for diabetic patients with asymptomatic gallstones, based
on the assumption that autonomic neuropathy masked the pain and signs associated with acute cholecystitis,
and that patients would therefore develop advanced disease and more complications. More recent evidence
has shown that these patients have a lower risk of major complications than previously thought.

Prophylactic cholecystectomy is not recommended in renal transplant patients with asymptomatic


gallstones. One study found that 87% of these patients remained asymptomatic after 4 years, with only 7%
developing acute cholecystitis and requiring subsequent uncomplicated laparoscopic cholecystectomy.
Other studies have shown that the presence of gallstone disease does not negatively affect graft survival.

Patients with hemoglobinopathies are at a significantly increased risk for developing pigmented stones.
Gallstones have been reported in up to 70% of sickle cell patients, up to 85% of hereditary spherocytosis
patients, and up to 24% of thalassemia patients. In sickle cell patients, complications from asymptomatic
gallstones have been reported to be as high as 50% within 3–5 years of diagnosis. This has been attributed
largely to the diagnostic challenge associated with symptomatic cholelithiasis versus abdominal sickling
crisis. In the past these patients were managed expectantly because of the significant morbidity and
mortality associated with open operations. The operative risk for these patients (especially sickle cell
patients) has been lowered by laparoscopic cholecystectomy, along with improved understanding of
preoperative hydration and transfusion, improved anesthetic technique, and better postoperative care.
Prophylactic laparoscopic cholecystectomy in these patients prevents future diagnostic confusion, as well
as the mortality and morbidity risk associated with emergency surgery. Furthermore, cholecystectomy can
and should be performed at the time of splenectomy, whether open or laparoscopic.

Studies have shown no significant differences in progression to symptoms from silent gallstones in cirrhotic
patients compared with noncirrhotic patients. Expectant management is therefore recommended in patients
with cirrhosis.

Ref: Cameron JL, Cameron AM (eds): Current Surgical Therapy, ed 10. Elsevier Saunders, 2011, pp 335-336.

Item 200

ANSWER: A

The criteria for chronic fatigue syndrome include fatigue for 6 months and a minimum of four of the
following physical symptoms: impaired memory, postexertional malaise, muscle pain, polyarthralgia,
tender lymph nodes, sore throat, new headaches, and unrefreshing sleep. Both cognitive-behavioral therapy
and graded exercise therapy have been shown to improve fatigue levels, anxiety, work/social adjustment,
and postexertional malaise (SOR A). Treatments that have not been shown to be effective include
methylphenidate, melatonin, and galantamine. Citalopram has not been shown to be effective in the
absence of a comorbid diagnosis of depression.

67
Ref: Yancey JR, Thomas SM: Chronic fatigue syndrome: Diagnosis and treatment. Am Fam Physician 2012;86(8):741-746.

Item 201

ANSWER: B

There are many drugs that can induce a syndrome resembling systemic lupus erythematosus, but the most
common offenders are antiarrhythmics such as procainamide. Hydralazine is also a common cause. There
is a genetic predisposition for this drug-induced lupus, determined by drug acetylation rates. Polyarthritis
and pleuropericarditis occur in half of patients, but CNS or renal involvement is rare. While all patients
with this condition have positive antinuclear antibody titers and most have antibodies to histones, antibodies
to double-stranded DNA and decreased complement levels are rare, which distinguishes drug-induced
lupus from idiopathic lupus.

The best initial management for drug-induced lupus is to withdraw the drug, and most patients will improve
in a few weeks. For those with severe symptoms, a short course of corticosteroids is indicated. Once the
offending drug is discontinued, symptoms seldom last beyond 6 months.

Ref: Klippel JH, Stone JH, Crofford LJ, et al (eds): Primer on the Rheumatic Diseases, ed 13. Springer, 2008, pp 316-317.
2) Longo DL, Fauci AS, Kasper DL, et al (eds): Harrison’s Principles of Internal Medicine, ed 18. McGraw-Hill, 2012,
p 2735.

Item 202

ANSWER: E

If a metal foreign body is present on the cornea for more than 24 hours a rust ring will often be present
in the superficial layer of the cornea. This material is toxic to the cornea and should be removed as soon
as possible, but it is not an emergency. The proper removal of a rust ring requires the use of a slit lamp
and specialized ophthalmic equipment. Referral to an eye specialist within 24–48 hours is the best
management in this case.

Ref: Tintinalli JE, Kelen GD, Stapczynski JS (eds): Emergency Medicine: A Comprehensive Study Guide, ed 7. McGraw-Hill,
2011, pp 1517-1549.

Item 203

ANSWER: B

Workers and children at child care centers should receive postexposure prophylaxis if one or more cases
of hepatitis A is found in a child or worker. Hepatitis A vaccine is preferred over immunoglobulin because
of its long-lasting effect, ease of administration, and efficacy. Children younger than 1 year of age should
receive immunoglobulin. Family members should receive prophylaxis only during an outbreak and if their
child is still in diapers.

Ref: Update: Prevention of hepatitis A after exposure to hepatitis A virus and in international travelers. Updated
recommendations of the Advisory Committee on Immunization Practices (ACIP). MMWR Morb Mortal Wkly Rep
2007;56(41):1080-1084. 2) Matheny SC, Kingery JE: Hepatitis A. Am Fam Physician 2012;86(11):1027-1034.

68
Item 204

ANSWER: A

In general, recommended preoperative testing is based on the patient’s medical history and risk factors,
the risk associated with the planned surgery, and the patient’s functional capacity. In the case of cataract
surgery, however, randomized, controlled trials have established a lack of benefit from preoperative testing
for patients in their normal state of health (SOR A).

Ref: Keay L, Lindsley K, Tielsch J, et al: Routine preoperative medical testing for cataract surgery. Cochrane Database Syst
Rev 2012;(3):CD007293. 2) Feely MA, Collins CS, Daniels PR, et al: Preoperative testing before noncardiac surgery:
Guidelines and recommendations. Am Fam Physician 2013;87(6):414-418.

Item 205

ANSWER: C

Cefazolin is the recommended prophylactic antibiotic for most patients undergoing orthopedic procedures
such as total joint replacement, unless the patient has a $-lactam allergy (SOR A).

Ref: Salkind AR, Rao KC: Antibiotic prophylaxis to prevent surgical site infections. Am Fam Physician 2011;83(5):585-590.
2) Bratzler DW, Dellinger EP, Olsen KM, et al: Clinical practice guidelines for antimicrobial prophylaxis in surgery. Am
J Health Syst Pharm 2013;70(3):195-283.

Item 206

ANSWER: D

A medial meniscus tear is the most likely diagnosis in a patient older than 40 who was bearing weight when
the injury occurred, was unable to continue the activity, and has a positive Thessaly test. This test is
performed by having the patient stand on one leg and flex the knee to 20°, then internally and externally
rotate the knee. The presence of swelling immediately after the injury makes an internal derangement of
the knee more likely, so osteoarthritis is less probable. This patient is able to bear weight, so a fracture
is also not likely. Either a collateral ligament tear or an anterior cruciate ligament tear is possible, but these
are not as common in this situation.

Ref: Grover M: Evaluating acutely injured patients for internal derangement of the knee. Am Fam Physician
2012;85(3):247-252.

Item 207

ANSWER: D

Orthostatic hypotension is defined as a documented drop in blood pressure of at least 20 mm Hg systolic


or 10 mm Hg diastolic that occurs within 3 minutes of standing. When symptomatic it is often described
as lightheadedness or dizziness upon standing. Etiologies to consider include iatrogenic, neurologic,
cardiac, and environmental causes, plus many others alone or in combination. Since orthostatic
hypotension may result in syncope, leading to falls and substantial injury, identifying it and taking
corrective steps can produce a significant benefit.

Ref: Bradley JG, Davis KA: Orthostatic hypotension. Am Fam Physician 2003;68(12):2393-2398. 2) Gilhus NE, Barnes M,
Brainin M (eds): European Handbook of Neurological Management, ed 2. Blackwell Publishing Ltd, 2011, pp 469-475.

69
Item 208

ANSWER: A

Peaked T waves are most commonly associated with acute myocardial infarction and hyperkalemia (SOR
A). Other causes include intracranial bleeding, left ventricular hypertrophy, and left bundle branch block.
Hypocalcemia can cause peaked T waves on rare occasions, but hypercalcemia has no effect on T waves.
Metabolic alkalosis is not associated with a particular EKG pattern, but it is associated with hypokalemia
and can lead to the patterns resulting from the potassium deficit. Hypothermia causes an elevation at the
J point, called either a J wave or an Osborn wave.

Ref: Goldberger AL: Clinical Electrocardiography: A Simplified Approach, ed 7. Mosby Elsevier, 2006, pp 129-132. 2) Bonow
RO, Mann DL, Zipes DP, Libby P (eds): Braunwald’s Heart Disease: A Textbook of Cardiovascular Medicine, ed 9.
Elsevier Saunders, 2011, pp 159-161.

Item 209

ANSWER: A

Spironolactone is commonly used to treat hirsutism in women with polycystic ovary syndrome. One of its
side effects is hyperkalemia. Using an oral contraceptive at the same time as spironolactone has a
synergistic effect for treating hirsutism due to the low androgenic effects of oral contraceptives.
Contraceptives containing drospirenone should be used with caution in patients taking spironolactone,
however, because they can also lead to hyperkalemia. The other contraceptive options listed do not
increase the risk for hyperkalemia.

Ref: Radosh L: Drug treatments for polycystic ovary syndrome. Am Fam Physician 2009;79(8):671-676.

Item 210

ANSWER: C

Erysipelas is caused primarily by group A Streptococcus, with a rare case caused by group C or G. Most
cases of erysipelas involve the face, but the lesions can occur anywhere on the body. Penicillin is an
effective treatment.

Ref: Longo DL, Fauci AS, Kasper DL, et al (eds): Harrison’s Principles of Internal Medicine, ed 18. McGraw-Hill, 2012, p
1067.

Item 211

ANSWER: A

Some hypothyroid patients who are treated with appropriate dosages of levothyroxine and whose TSH
levels are in the appropriate range continue to have persistent symptoms such as fatigue, depressed mood,
and weight gain. If the TSH is in the appropriate range then no adjustment is necessary and annual serum
TSH testing is recommended. Patients who remain symptomatic on an appropriate dosage of levothyroxine,
as determined by a TSH <2.5 mIU/L, are not likely to benefit from combination
triiodothyronine/thyroxine therapy (SOR A). Desiccated thyroid hormone preparations are not
recommended by the American Association of Clinical Endocrinologists for the treatment of
hypothyroidism. A meta-analysis of 11 randomized, controlled trials of combination T3/T4 therapy versus
T4 monotherapy showed no improvements in pain, depression, or quality of life (SOR A).

70
Ref: Gaitonde DY, Rowley KD, Sweeney LB: Hypothyroidism: An update. Am Fam Physician 2012;86(3):244-251. 2) Garber
JR, Cobin RH, Gharib H, et al: Clinical practice guidelines for hypothyroidism in adults: Cosponsored by the American
Association of Clinical Endocrinologists and the American Thyroid Association. Endocr Pract 2012;18(6):988-1028.

Item 212

ANSWER: A

Treating basal cell carcinoma with Mohs micrographic surgery leads to the lowest recurrence rate. Because
of its cost and limited availability, however, this procedure should be limited to tumors with a higher risk
for recurrence. Risk factors include larger size, more invasive histologic subtypes (micronodular,
infiltrative, and morpheaform), and sites associated with a higher risk of recurrence.

High-risk locations include the “mask” areas of the face, which include the central face, eyelids, eyebrows,
periorbital area, nose, lips (cutaneous and vermilion), chin, mandible, preauricular and postauricular
skin/sulci, temple, and ear. Other high-risk sites include the genitalia, hands, and feet. Moderate-risk
locations include the cheeks, forehead, scalp, and neck. All other areas, including the trunk and
extremities, are low-risk areas.

Even with a low-risk location, a lesion that is ³20 mm in size has a high risk of recurrence. With a
moderate-risk location a lesion ³10 mm in size carries a higher risk of recurrence, and a lesion ³6 mm in
size is considered high risk in a high-risk location.

Ref: Firnhaber JM. Diagnosis and treatment of basal cell and squamous cell carcinoma. Am Fam Physician 2012;86(2):161-168.

Item 213

ANSWER: D

In a study of 256 asymptomatic Swedish patients with mildly elevated liver transaminase levels, alcohol
was found to be the cause in 10% of cases. An accurate history is important for making the diagnosis. A
biopsy alone cannot differentiate alcoholic liver disease from nonalcoholic fatty liver disease. An
AST/ALT ratio >2 supports a diagnosis of alcoholic liver disease. Elevated (-glutamyl transpeptidase
(GGTP) is also associated with alcohol abuse, especially in a patient with an AST/ALT ratio >2.

Ref: Oh RC, Hustead TR: Causes and evaluation of mildly elevated liver transaminase levels. Am Fam Physician
2011;84(9):1003-1008.

Item 214

ANSWER: B

Tinea capitis is an infection of the scalp caused by a variety of superficial dermatophytes. The treatment
of choice for this infection is oral griseofulvin. It has the fewest drug interactions, a good safety record,
and anti-inflammatory properties. Terbinafine has equal effectiveness and requires a significantly shorter
duration of therapy, but it is only available in tablet form. Since tinea capitis most commonly occurs in
children, tablets would have to be cut and/or crushed prior to administration. Oral itraconazole,
fluconazole, and ketoconazole have significant side effects. Topical antifungals such as ketoconazole and
miconazole are ineffective against tinea capitis. Topical acyclovir is used in the treatment of herpesvirus
infections, and oral cephalosporins are used in the treatment of bacterial skin infections.

71
Ref: Habif TP: Clinical Dermatology: A Color Guide to Diagnosis and Therapy, ed 5. Mosby Elsevier, 2010, pp 509-510.

Item 215

ANSWER: D

While it would increase the risk of urinary infection, indwelling catheter placement is most likely to
provide immediate relief of this patient’s urinary retention. It will minimize or prevent further
contamination of his decubitus ulcer with urine. Prostatectomy may relieve the urethral obstruction, but
this patient is likely to remain incontinent due to his vascular dementia. Doxazosin or finasteride would
likely be inadequate in this situation. Tolterodine is not indicated for overflow incontinence.

Ref: Unwin BK, Porvaznik M, Spoelhof GD: Nursing home care: Part II. Clinical aspects. Am Fam Physician
2010;81(10):1229-1237. 2) Gould CV, Umscheid CA, Agarwal RK, et al: Guideline for prevention of catheter-associated
urinary tract infections 2009. Infect Control Hosp Epidemiol 2010;31(4):319-326. 3) DuBeau CE, Kuchel GA, Johnson
T 2nd, et al: Incontinence in the frail elderly: Report from the 4th International Consultation on Incontinence. Neurourol
Urodyn 2010;29(1):165-178.

Item 216

ANSWER: A

This patient has a low pretest probability of pulmonary embolism based on the Wells criteria. She would
be a good candidate for a high-sensitivity D-dimer test, with a negative test indicating a low probability
of venous thromboembolism. In patients with a low pretest probability of venous thromboembolism,
ultrasonography or helical CT would not be the recommended initial evaluation. Neither troponin I nor an
ANA level would be part of the recommended initial evaluation.

Ref: Wilbur J, Shian B: Diagnosis of deep venous thrombosis and pulmonary embolism. Am Fam Physician
2012;87(12):913-919.

Item 217

ANSWER: A

Intertrigo is skin inflammation caused by skin-on-skin friction. It is facilitated by moisture trapped in deep
skinfolds where air circulation is limited. When intertrigo does not respond to usual conservative measures,
including keeping the skin clean and dry, evaluation for infection is recommended. A Wood’s light
examination, KOH preparation, and exudate culture can assist in identifying causative organisms.

The moist, damaged skin associated with intertrigo is a fertile breeding ground for various
microorganisms, and secondary cutaneous infections are commonly observed in these areas. Candida is
the organism most commonly associated with intertrigo. In the interdigital spaces dermatophytes (e.g.,
Trichophyton rubrum, Trichophyton mentagrophytes, Epidermophyton floccosum) are more common.
Staphylococcus aureus may present alone or with group A $-hemolytic Streptococcus (GABHS).
Pseudomonas aeruginosa, Proteus mirabilis, or Proteus vulgaris also may occur alone or simultaneously.

Ref: Janniger CK, Schwartz RA, Szepietowski JC, Reich A: Intertrigo and common secondary skin infections. Am Fam
Physician 2005;72(5):833-838. 2) Habif TP: Clinical Dermatology: A Color Guide to Diagnosis and Therapy, ed 5. Mosby
Elsevier, 2010, p 534.

72
Item 218

ANSWER: C

Kawasaki disease, or mucocutaneous lymph node syndrome, is a common form of vasculitis most often
seen in children under 5 years of age. It is typically self-limited, with fever and acute inflammation lasting
12 days on average without therapy. However, if untreated, this illness can result in heart failure, coronary
artery aneurysm, myocardial infarction, arrhythmias, or occlusion of peripheral arteries. This diagnosis
requires that fever be present for 5 days or more with no other explanation. In addition, at least four of
the following symptoms must be present: (1) nonexudative conjunctivitis that spares the limbus; (2)
changes in the oral membranes such as diffuse erythema, injected or fissured lips, or “strawberry tongue”;
(3) erythema of the palms and soles, and/or edema of the hands or feet followed by periungual
desquamation; (4) cervical adenopathy in the anterior cervical triangle with at least one node larger than
1.5 cm in diameter; and, (5) an erythematous polymorphous rash, which may be targetoid or purpuric in
20% of cases. The disease must be distinguished from toxic shock syndrome, streptococcal scarlet fever,
Stevens-Johnson syndrome, juvenile rheumatoid arthritis, measles, adenovirus infection, echovirus
infection, and drug reactions.

Treatment significantly diminishes the risk of complications. Current recommendations are to hospitalize
the patient for treatment with intravenous immune globulin. In addition, aspirin is used for both its
anti-inflammatory and antithrombotic effects. While prednisone is used to treat other forms of vasculitis,
it is considered unsafe in Kawasaki disease, as a study has shown an extraordinarily high rate of coronary
artery aneurysm with its use.

Ref: Scuccimarri R: Kawasaki disease. Pediatr Clin North Am 2012;59(2):425-445.

Item 219

ANSWER: D

This patient has classic findings for acute rectal fissure. Although patients often require an internal
sphincterotomy, nonsurgical measures that relax the sphincter have proven helpful. Botulinum toxin
injected into the internal sphincter has proven most beneficial, but topical preparations are not yet available
and have not been shown to be effective for this problem. Corticosteroid creams may decrease the pain
temporarily, but potent fluorinated corticosteroid creams such as clobetasol are not indicated in the
treatment of fissure. Capsaicin cream can be helpful for pruritus ani, but not for anal fissures. Drugs that
dilate the internal sphincter, including diltiazem, nifedipine, and nitroglycerin ointment, have proven to
be beneficial in healing acute fissures, but usually have to be compounded by a pharmacist.

Ref: Fargo MV, Latimer KM: Evaluation and management of common anorectal conditions. Am Fam Physician
2012;85(6):624-630.

73
Item 220

ANSWER: A

Children generally have a benign course after insect stings, but those who have a moderate to severe
systemic reaction have a high risk of future reactions. Local reactions may initially look like cellulitis, but
antibiotic therapy is not needed. The treatment of choice for anaphylaxis subsequent to an insect sting is
systemic epinephrine. Corticosteroids may be given as adjunctive treatment. Immunotherapy may induce
cellular suppression after 4 or 5 years. Patients who stop venom immunotherapy after 1 or 2 years continue
to be at moderate risk for systemic allergic reaction to future stings.

Ref: Golden DB: Stinging insect allergy. Am Fam Physician 2003;67(12):2541-2546. 2) Kliegman RM, Stanton BF, Geme JW
III, et al (eds): Nelson Textbook of Pediatrics, ed 19. Elsevier Saunders, 2011, pp 808-809.

Item 221

ANSWER: B

Spironolactone is now recommended for treating resistant hypertension, even when hyperaldosteronism
is not present. A longer-acting diuretic such as chlorthalidone is also recommended for treating
hypertension, particularly in resistant cases with normal renal function. There is no benefit to switching
from an ACE inhibitor to an ARB. Nitrates have some effect on blood pressure but are recommended only
for patients with coronary artery disease.

Ref: Nishizaka MK, Zaman MA, Calhoun DA: Efficacy of low-dose spironolactone in subjects with resistant hypertension. Am
J Hypertens 2003;16(11 Pt 1):925-930. 2) Myat A, Redwood SR, Qureshi AC, et al: Resistant hypertension. BMJ
2012;345:e7473. 3) Adams M, Bellone JM, Wright BM, Rutecki GW: Evaluation and pharmacologic approach to patients
with resistant hypertension. Postgrad Med 2012;124(1):74-82.

Item 222

ANSWER: A

Chronic excessive alcohol intake produces functional changes in neurotransmitter activity that can lead to
a net increase in excitatory neuroreceptor activity when the person stops drinking. Withdrawal can be
divided into four levels of severity: minor, major, seizures, and delirium tremens. Minor alcohol
withdrawal is characterized by tremor, anxiety, nausea, vomiting, and/or insomnia 6–24 hours after the
patient’s last drink. Major withdrawal occurs 10–72 hours after the last drink and can include the signs and
symptoms of minor withdrawal, as well as visual and auditory hallucinations, diaphoresis, tachycardia,
and elevated blood pressure. Alcoholic seizure generally occurs within 2 days of the last drink and may
be the only sign of withdrawal, although approximately one-third of these patients will progress to delirium
tremens. The onset of delirium tremens can occur anytime within 3–10 days following the last drink. The
defining clinical finding is delirium, but the findings seen in milder forms of alcohol withdrawal can also
be present, and may be more severe. Fever is most often seen with delirium tremens and is less common
with less severe forms of alcohol withdrawal.

Ref: Rakel RE, Rakel DP (eds): Textbook of Family Medicine, ed 8. Elsevier Saunders, 2011, pp 1095-1097. 2) Manasco A,
Chang S, Larriviere J, et al: Alcohol withdrawal. South Med J 2012;105(11):607-612.

74
Item 223

ANSWER: E

Cyanosis and tachypnea may be a sign of transient tachypnea of the newborn (TTN), or may be the first
signs of a more serious health condition. Many of the severe conditions may be ruled out through the
history (hyaline membrane disease is unlikely in this full-term infant) and physical examination (choanal
atresia may be ruled out at the bedside). After these initial steps are taken, a chest radiograph and
laboratory studies will help rule out many urgent and life-threatening conditions, such as pneumothorax,
pneumonia, severe anemia, sepsis, etc. An EKG is less helpful in determining causes since it cannot
differentiate defects that cause cyanotic heart disease. Infusion of prostaglandin E1 should be started only
if the condition is determined to be ductal dependent; this is not the case in this scenario. Bag-mask
ventilation is not warranted in this child, who is alert and breathing and has a normal heart rate. At this
time it is reasonable to start investigations into the cause of the child’s problems before resorting to more
extreme measures such as intubation.

Ref: Hermansen CL, Lorah KN: Respiratory distress in the newborn. Am Fam Physician 2007;76(7):987-994. 2) Tutdibi E,
Gries K, Bücheler M, et al: Impact of labor on outcomes in transient tachypnea of the newborn: Population-based study.
Pediatrics 2010;125(3):e577-e583.

Item 224

ANSWER: E

A peripheral blood smear revealing the presence of keratocytes, blister cells, and polychromatic
macrocytes is consistent with an oxidative hemolytic process. Hemolysis is seen in glucose-6-phosphate
dehydrogenase (G6PD) deficient persons, such as this patient, following an acute insult triggered by drugs,
infection, or fava bean ingestion. A blood smear suggests the diagnosis even if an assay for G6PD is
normal, which is frequently the case in heterozygous men of African ancestry. Peripheral blood smears
showing spherocytes suggest spherocytic hemolytic anemia. Sickle-shaped red blood cells indicate sickle
cell anemia, whereas hypochromic and teardrop-shaped red blood cells may be seen in thalassemia. Red
blood cell fragments are found in microangiopathic hemolytic anemia.

Ref: Bain BJ: Diagnosis from the blood smear. N Engl J Med 2005;353(5):498-507. 2) Frank JE: Diagnosis and management
of G6PD deficiency. Am Fam Physician 2005;72(7):1277-1282. 3) Longo DL, Fauci AS, Kasper DL, et al (eds):
Harrison’s Principles of Internal Medicine, ed 18. McGraw-Hill, 2012, pp 878-880.

Item 225

ANSWER: B

The combination of the vasodilators hydralazine and isosorbide dinitrate has been shown to be effective
in the treatment of heart failure when standard treatment with diuretics, $-blockers, and an ACE inhibitor
(or ARB) is insufficient to control symptoms or cannot be tolerated. This combination is particularly
effective in African-Americans with NYHA class III or IV heart failure, with advantages including reduced
mortality rates and improvement in quality-of-life measures. Digoxin, a long-time standard for the
treatment of heart failure, is useful in reducing the symptoms of heart failure but has not been shown to
improve survival. Amlodipine and other calcium channel blockers do not have a direct role in the treatment
of heart failure.

Ref: Drugs for chronic heart failure. Treat Guidel Med Lett 2012;10(121):69-72.

75
Item 226

ANSWER: D

Until recently, the number of heroin overdoses had been in decline for the past few decades. Although
heroin still only accounts for about 1% of drug overdoses, it has become more common in the past few
years. Overdose is manifested by CNS depression and hypoventilation. Clinical clues include pupillary
miosis and a decreasing respiratory rate in the presence of a semi-wakeful state. In addition to
hypoventilation, a multifactorial acute lung injury occurs within 2–4 hours of the overdose and is associated
with hypoxemia and a hypersensitivity reaction, resulting in noncardiogenic pulmonary edema. Findings
include hypoxia, crackles on lung auscultation, and pink, frothy sputum. Treatment must include
respiratory support with intubation, mechanical ventilation, and oxygen, as well as opiate reversal with
naloxone, which may require repeat doses or intravenous infusion.

Arrhythmias and myocardial ischemia/infarction do not occur as direct pharmacologic effects of heroin,
although they may occur as a consequence of the pulmonary toxicity or the presence of other drugs taken
intentionally or otherwise (i.e., heroin cut with other agents). Acute renal injury, hepatic injury, and
thromboembolic events are also not a direct result of the pharmacologic effects of heroin.

Ref: Shannon MW, Borron SW, Burns M: Haddad and Winchester’s Clinical Management of Poisoning and Drug Overdose,
ed 4. Saunders Elsevier, 2007, pp 644-645. 2) Li W, Gunja N: Illicit drug overdose: Prevalence and acute management.
Aust Fam Physician 2013;42(7):481-485.

Item 227

ANSWER: A

Acromegaly usually has an insidious onset, with a time to diagnosis on the order of 6–10 years. The classic
facial findings, which include enlargement of the supraorbital ridges and mandible and a widened nose,
are sometimes difficult to identify without a reference to the patient’s appearance from several years
earlier. Even close family members will often not notice the changes since they occur so gradually.
Patients with this condition sometimes present with nonspecific symptoms such as hyperhidrosis,
arthralgias, fatigue, and headache. More specific complaints such as visual field defects or increasing ring
and shoe sizes should prompt a physician to consider this diagnosis. The condition is caused by a growth
hormone–secreting pituitary adenoma in 98% of cases. In younger patients this disease results in gigantism.
Blood testing for growth hormone and IGF-1 is usually adequate for initial testing.

Ref: Reddy R, Hope S, Wass J: Acromegaly. BMJ 2010;341:c4189. 2) Longo DL, Fauci AS, Kasper DL, et al (eds):
Harrison’s Principles of Internal Medicine, ed 18. McGraw-Hill, 2012, pp 2876-2902.

Item 228

ANSWER: C

Psychogenic polydipsia is voluntary excessive water intake, usually in excess of 1 L/hr, which overwhelms
the capacity of the kidneys to excrete free water in the urine. This disorder is seen most often in patients
with either mental illness or a developmental disability. It is most common in individuals with chronic
schizophrenia and middle-aged women with anxiety disorders. It is characterized by hyponatremia but
seldom to the point of causing symptoms. When plasma osmolarity is <280 mOsm/kg, the secretion of
antidiuretic hormone is suppressed. The water-seeking behavior is noted by others rather than being
reported by the patient.

76
The physical examination in these patients is usually unremarkable, and evidence of volume overload such
as demonstrable pitting edema should prompt suspicion of another etiology. Plasma and urine osmolarity
should be measured, along with plasma and urinary sodium. Urinary measurements are best done by
collecting a 24-hour urine sample. If the patient’s hyponatremia is noted to be euvolemic and hypotonic,
with a urine osmolarity <100 mOsm/kg, and if serum osmolarity is also low (<280 mOsm/kg), then a
metabolic panel and urinalysis would be helpful to exclude other causes of hyponatremia. Other etiologies
should also be considered if elevated glucose, elevated protein, or abnormal renal function is noted. With
regard to urine specific gravity, the urine should be dilute in patients with psychogenic polydipsia. Specific
gravity would be a predictable way of noting osmolarity, as a lower specific gravity would indicate a lower
urine osmolarity.

Ref: Marx JA (ed): Rosen’s Emergency Medicine: Concepts and Clinical Practice, ed 7. Mosby Elsevier, 2010, p 1725.

Item 229

ANSWER: C

Regular use of inhaled long-acting $2-agonists, inhaled long-acting anticholinergic agents, or inhaled
corticosteroids has been shown to reduce the risk of COPD exacerbations, with combinations of these
agents producing additional benefit compared with monotherapy. The other agents listed are helpful for
relief of symptoms of COPD but do not reduce the incidence of exacerbations.

Ref: Niewoehner DE: Outpatient management of severe COPD. N Engl J Med 2010;362(15):1407-1416.

Item 230

ANSWER: E

Clostridium difficile infections are associated with a high rate of recurrence. Approximately 20% of
patients successfully treated for C. difficile will have a relapse. For the first relapse, a 10- to 14-day course
of oral metronidazole is recommended if symptoms are moderate. If symptoms are severe, a 10- to 14-day
course of oral vancomycin is indicated. If a second relapse is confirmed, an oral vancomycin taper over
approximately 15 weeks is recommended. The regimen is 125 mg every 6 hours for 10–14 days, then 125
mg every 12 hours for 7 days, then 125 mg daily for 7 days, then 125 mg every other day for 8 days,
followed by 125 mg every 3 days for 15 days.

Intravenous vancomycin is not effective for C. difficile infections. Clindamycin is a common cause of C.
difficile infection and has no role in its treatment.

Ref: Friedman LS, Brandt LJ (eds): Sleisenger & Fordtran’s Gastrointestinal and Liver Disease, ed 9. Saunders, 2010, p 1901.

Item 231

ANSWER: E

The family physician or a consulting psychiatrist can make recommendations regarding driving, and the
patient’s family or the person designated as having power of attorney can withhold access to a vehicle, but
the state motor vehicle department reserves final judgment in these situations, and may require a driving
test.

77
Ref: Carr DB, Ott BR: The older driver with cognitive impairment: “It’s a very frustrating life.” JAMA
2010;303(16):1632-1641.

Item 232

ANSWER: C

Signs that an asthma exacerbation may be life threatening include altered mental status, absence of
wheezing, and paradoxical chest or abdominal movement. A PaCO2 >42 mm Hg may indicate impending
respiratory failure; levels <40 mm Hg would be expected with hyperventilation of any cause. An FEV1
<40% of expected, especially after initial treatment in the emergency department, is an indication for
admission. Systemic corticosteroids are frequently used for even moderate asthma exacerbations.

Ref: Lazarus SC: Emergency treatment of asthma. N Engl J Med 2010:363(8):755-764.

Item 233

ANSWER: C

The picture shows a very localized erythematous rash that appears to be allergic dermatitis in the healing
stage. The location is very atypical of eczema, and the chronicity would rule out herpes zoster. The lesion
does not have any typical features of tinea corporis, which tends to be annular or macular with scaling.
Squamous cell cancer is exceedingly rare in children and would not be suggested by a rash that completely
disappears only to reappear again. The history and appearance are most suggestive of nickel sensitivity,
likely associated with a metal clasp or snap on the child’s pants. Placing athletic tape over this clasp usually
results in resolution of the condition.

Ref: Kliegman RM, Stanton BF, Geme JW III, et al (eds): Nelson Textbook of Pediatrics, ed 19. Elsevier Saunders, 2011, pp
2250-2252.

Item 234

ANSWER: A

This patient’s injury is commonly referred to as “jersey finger,” a flexor digitorum profundus avulsion
fracture that results from forced hyperextension of a flexed DIP joint. On examination the patient will be
unable to flex the finger at the DIP joint. Radiographs will show a bony fragment at the volar surface of
the proximal distal phalanx. Because the risk of tendon retraction is high, patients with these fractures
should be referred to a hand surgeon as soon as the diagnosis is made.

Ref: Borchers JR, Best TM: Common finger fractures and dislocations. Am Fam Physician 2012;85(8):805-810.

78
Item 235

ANSWER: C

Most persons who have come in close contact with individuals confirmed to have pertussis should receive
postexposure prophylaxis, regardless of immunization status (SOR C). The decision about whether to give
postexposure chemoprophylaxis should take into account several factors, including infectiousness, degree
of exposure, potential consequences of severe pertussis in the exposed individual, and the possibility of
secondary exposure of persons at high risk, such as infants younger than 12 months of age. If there is no
contraindication, a macrolide given to close contacts, and started within 21 days of the onset of the original
patient’s cough, can prevent symptomatic infection. Early macrolide administration can also reduce the
duration and severity of symptoms and shorten the time the patient is contagious.

The prophylactic dosage of azithromycin is the same as the treatment dosage, which is a daily dose for 5
days. A one-dose regimen is not recommended. Prophylaxis is indicated in this patient because he was in
close contact with the infected individual and also has a 4-month-old sister, who is at high risk from
possible secondary exposure. He is up to date on all vaccinations, and DTaP vaccine is not appropriate for
use as prophylaxis. Tetracycline is contraindicated in young children.

Ref: Tiwari T, Murphy TV, Moran J, et al: Recommended antimicrobial agents for the treatment and postexposure prophylaxis
of pertussis: 2005 CDC guidelines. MMWR Recomm Rep 2005;54(RR-14):1-16. 2) Graham L: CDC releases guidelines
on antimicrobial agents for the treatment and postexposure prophylaxis of pertussis. Am Fam Physician 2006;74(2):333-336.
3) Bader MS, McKinsey DS: Postexposure prophylaxis for common infectious diseases. Am Fam Physician 2013;88(1):
25-32.

Item 236

ANSWER: B

An association with vitamin B12 deficiency has been noted for metformin, proton pump inhibitors, and
H2-blockers. It is not known whether supplementation in patients taking these drugs will prevent this, but
monitoring vitamin B12 levels may be useful. Prednisone, insulin glargine, gabapentin, and risperidone
have not been associated with vitamin B12 deficiency.

Ref: Langan RC, Zawistoski KJ: Update on vitamin B12 deficiency. Am Fam Physician 2011;83(12):1425-1430.

Item 237

ANSWER: A

A pregnancy test is an appropriate first step in the evaluation of a woman with amenorrhea and
galactorrhea who is not taking medications. If the test is negative, hyperprolactinemia should be suspected.
Provided that the sample is obtained without excessive venipuncture stress, measurement of a single
prolactin level above the upper limits of normal confirms the diagnosis of hyperprolactinemia. In the
absence of other breast signs or symptoms, breast imaging is not necessary. A brain MRI focusing on the
pituitary sella is not needed until the diagnosis of hyperprolactinemia is confirmed. Dynamic tests of the
hypothalamic-pituitary-adrenal axis, such as a dexamethasone suppression test, are generally not indicated
in the evaluation of amenorrhea/galactorrhea.

Ref: Melmed S, Casanueva FF, Hoffman AR, et al: Diagnosis and treatment of hyperprolactinemia: An Endocrine Society
clinical practice guideline. J Clin Endocrinol Metab 2011;96(2):273-288. 2) Huang W, Molitch ME: Evaluation and
management of galactorrhea. Am Fam Physician 2012;85(11):1073-1080.

79
Item 238

ANSWER: D

SSRIs are the first-line medications in the treatment of general anxiety disorder (SOR A). SSRIs can be
used long-term without concerns about abuse or tolerance. They are both effective and well tolerated, and
help treat the concomitant depression that exists in many patients with generalized anxiety disorder (GAD).
Cognitive-behavioral therapy has also been shown to be an effective treatment for GAD and has lower
attrition rates and more durable effects compared to medications (SOR A). SNRIs have also been shown
to have some benefit. Buspirone is effective in the treatment of anxiety, but does not help with depression.
The same is true for benzodiazepines, which are effective for treating GAD but do not help with depression
that might be present. Pregabalin has been used in Europe for treatment of GAD, but the FDA has not
approved this use.

Ref: Kavan MG, Elsasser G, Barone EJ: Generalized anxiety disorder: Practical assessment and management. Am Fam Physician
2009;79(9):785-791.

Item 239

ANSWER: C

Antimitochondrial antibodies are positive in 90% of patients with primary biliary cirrhosis and this test is
the first step for ruling out the disease. A liver biopsy is the most important test in antimitochondrial
antibody–negative primary biliary cirrhosis, but it is not the initial test of choice. Antinuclear antibody
testing, anti–smooth muscle antibody testing, and magnetic resonance cholangiopancreatography are not
appropriate.

Ref: Goldman L, Schafer AI (eds): Goldman’s Cecil Medicine, ed 24. Elsevier Saunders, 2011, pp 1014-1015.

Item 240

ANSWER: A

Cold medications, including analgesic/decongestant combinations, provide modest symptom relief in adults
and older children, but are not effective and are associated with an increased risk of adverse effects in
those younger than 2 years of age (SOR A). The number needed to harm is 14.

Ref: Salisbury-Afshar E: Oral antihistamine/decongestant/analgesic combinations for the common cold. Cochrane for Clinicians
2012;86(9):812.

80

S-ar putea să vă placă și